Questions Flashcards Preview

Year 3 > Questions > Flashcards

Flashcards in Questions Deck (500)
Loading flashcards...
0
Q

This drug lowers heart rate, cardiac output and mean arterial blood pressure during exercise. It can also be used for migraine prophylaxis and glaucoma. This drug also causes a decrease in endogenous renin release.

A

Beta blocker. Lower heart rate, CO and MABP during exercise. Reduce renin release and NA release. Use: migraine prophylaxis, anxiety, hypotension, post- MI, CCF. Also useful in arrhythmias as they act to increase the refractory period of the AVN.

1
Q

This drug is a vasoconstrictor which can be used as a nasal decongestant. It is also mydriatic when used as an eye drop.

A

Alpha 1 agonist. Alpha 1 agonists such as phenylephrine are vasoconstrictors and also have a use as a mydriatic. They are used as nasal decongestants as a result of their vasoconstrictor effect.

2
Q

Beta blocker contraindications

A

Absolute: - asthma - severe bradycardia - high AV block - sick sinus syndrome - severe unstable LVF Relative - peripheral vascular disease - depression

3
Q

CCB contraindications

A

De compensated severe heart failure Bradycardia AV block Sick sinus syndrome

4
Q

A 55 year old obese female complains of an occasional burning pain behind the sternum. The pain is worse after large meals and when drinking hot liquids.

A

GORD This patient has GORD characterised by heartburn and regurgitation of acid. It is more severe at night when the patient is lying flat and also when the patient is bending over. Risk factors include obesity and hiatus hernia. Diagnosis is generally clinical and cans also be achieved by a diagnostic trial of a PPI. Normally an upper GI endoscopy is reserved for complications such as strictures, Barrett’s or cancer or for atypical features. An OGD may show oesophagitis, or Barrett’s (red velvety). However it may be normal. Manometry or pH monitoring may also be performed but in this case this patient will probably just have a therapeutic and diagnostic trial of a PPI.

5
Q

Mr L, an investment banker, who is as wide as he is tall, has been brought in to A&E at 2am on Sunday after collapsing at the kebab shop on Shepherds Bush Road. He is dressed in a schoolboy outfit, which only makes sense when a similarly dressed friend arrives to tell you that the office went clubbing in Po Na Na together. Apparently Mr L had been complaining of tight chest pain and anxiety earlier in the evening but a cigarette calmed him down.

A

Acute coronary syndrome. ACS refers to acute myocardial ischaemia and encompasses STEMI, NSTEMI and unstable angina. This man is clearly overweight and is seeming munching a kebab so there are clear cardiovascular risk factors. The right chest pain followed by a collapse make this sounds like an MI. Those with identified STEMI should be considered for immediate reperfusion therapy by thrombolytics or PCI. Those with NSTEMI or unstable angina do not benefit from this. Most complain of chest pain which is described as substernal pressure, heaviness, squeezing, burning or tightness. It may either localise or radiate to the arms, shoulders, back, neck or jaw and is usually reproduced by exertion, eating, cold or emotional stress. ECG and bio markers like troponin will be useful in confirming the diagnosis.

6
Q

Miss A, a 52 year old attends a Rapid Diagnostic Clinic for Breast Cancer at CX. She is concerned over an area of tenderness on her left breast. O/E you find that the lower medial quadrant of the left breast feels sore when you palate deeply. There is no palpable lump and both mammography and US are normal.

A

Costochondritis. This is costochondritis or Tietze’s syndrome (costochondritis & swelling), which presents with insidious onset of anterior chest wall pain which is made worse by certain movements of the chest and deep inspiration. The key sign is that there is pain when palpating the costochondral joints, particularly 2-5 and the diagnosis is clinical. Tests are done to exclude other diagnoses such as breast pathology. First line treatment is NSAIDs. Oral NSAIDs are preferred in the primary care setting and a beneficial response confirms the diagnosis. If NSAIDs or local corticosteroid injection fail to make the symptoms better then you should seek further investigations and consider a wider differential diagnosis including pleuritis, PE, rib fracture and GORD.

7
Q

Pain on breathing in & out, dyspnoea, haemoptysis, stony dull to percuss.

A

PE Patients with a high clinical suspicion of PE should be anti coagulated while waiting a definitive diagnosis unless CI. PE can cause atelectasis which can result in a dull percussion note and a pleural effusion which is exudative in nature, causing a stony dull percussion note. SOB and chest pain are common symptoms and there may also be haemoptysis. Strong risk factors include DVT, obesity, surgery in the past 2 months, prolonged bed rest, malignancy, previous VTE, pregnancy and the thrombophilias such as factor V Leiden. ECG may be normal, or may show tachycardia, new RAD, new RBBB or the classical S wave in I, Q wave with TWI in III.

8
Q

A 32 year old woman has a 2 day history of intermittent attacks of a sharp pain over the lower left side of her chest. The pain is exacerbated by movements of the rib cage and the patients tells you it becomes difficult to breathe. She has also felt feverish.

A

Bornholm disease. Bornholm disease is caused Coxsackie B virus and symptoms include the fever seen as well as the characteristic attacks of severe pain in the lower chest, which is exacerbated by small movements of the rib cage, which make it difficult for the patient to breathe.

9
Q

A 25 year old basketball player, with very long arms, describes sudden onset chest pain over the left side of his chest. He is very short of breath. Choose the single most appropriate investigation.

A

CXR This patient has a primary pneumothorax. Marfan’s syndrome is a risk factor and there are many reports of primary spontaneous pneumothoraces attributes to pulmonary tissue fragility due to defective fibrillin. Primary pneumothoraces occur in young people without known lung conditions. The main investigation is a CXR and are classified by the amount of visible rim: small 2cm. If the patient is clinically stable, they can be observed and given O2 - an invasive approach is not necessary and the O2 will increase the rate of pneumothorax reabsorption. A large pneumothorax may require per cutaneous needle aspiration. If this fails a chest drain should be inserted.

10
Q

A 23 year old student comes into hospital as part of the regular treatment for his condition. He was first brought to see you at a young age after his GP noticed he was not gaining weight adequately. In addition he was noted to be an ill child and was always developing colds. The mother distinctly remembers that he had difficulty passing his first motions after he was born? What is the approximate carrier frequency of this disease? When 18 he presented with severe pneumonia productive of green sputum, which indicated the acquisition of a pathogen the doctors had been worried about for a number of years. What is the most likely pathogen? What is the most suitable antibiotic?

A

The features are consistent with a diagnosis of cystic fibrosis, a common AR genetic disorder. The gene affected is for the CFTR, which is a protein that functions as a Cl channel. Abnormalities in chloride secretion lead to viscous fluids and result in excess inflammation and infections due to impaired clearance of secretions. The carrier frequency in the white population is approximately 1/20-1/25. The actual incidence is 1/2500 live births. Pseudomonas aeruginosa is a common pathogen found in CF patients. It is an opportunistic organism, infection is usually occurring in immunocompromised and severely ill patients. The green sputum is highly characteristic and occurs due to a blue-green pigment produced by the bacteria. Ciprofloxaxin.

11
Q

Which ONE of the following statements is true? A) paracetamol poisoning is the 2nd most commonly encountered drug overdose in the UK B) The hepatotoxic dose of paracetamol is taken as 150mg/kg for high risk patients. C) Paracetamol overdose leads to approx. 2000 deaths per year D) Activated charcoal is better than gastric lavage in certain situations. E) N-acetylcysteine is approx 75% useful in preventing significant hepatic impairment if given within 12 hours.

A

D) A) Paracetamol (acetaminophen) is the most common drug overdose. B) The hepatotoxic dose is 150mg/kg but those at high risk (2• to reduced glutathione reserves) can develop at much lower doses. C) 200 deaths per year E) N-acetylcysteine is 100% successful in preventing significant hepatic impairment if used within 8 hours.

12
Q

Which 1 is true? A) High risk patients are those in whom hepatic glutathione reserves are increased. B) patients with anorexia nervosa should be treated using the low risk treatment line. C) 1% of patients develop an allergic reaction to N-acetylcysteine D) methionine is an alternative to N-acetylcysteine E) serum albumin is the best indicator for severity of liver damage.

A

D). A) High risk are those in whom glutathione reserves are diminished. B) High risk patients include those with anorexia C) 10% develop an allergic reaction E) PT time best indicator of severity of liver damage in paracetamol overdose.

13
Q

A diabetic lady on metformin with suspected lung cancer is booked to have a CT scan of her chest. She has a microcytic anaemia with high urea and creatinine. Which 1 of the following should be undertaken? A) 0.9% saline for 12h prior to procedure B) increase metformin to optimise her blood sugar control. C) check for metal implants. D) ECG E) V/Q scan prior to procedure

A

A) This lady has pre-existing renal impairment as shown by the raised urea and creatinine and is also on metformin for her diabetes. She is therefore at great risk of developing contrast induced nephropathy. this is often not taken into consideration when people have CT scans with contrast. It presents as acute deterioration in renal function secondary to contrast medium used and can lead to significant morbidity and mortality. Good hydration prior to and following the procedure helps minimise the risk as does stopping metformin 48 hours before the scan. A number of agents are used in an attempt to prevent contrast induced nephropathy including N-acetylcysteine, aminophylline and statins. Contrast induced nephropathy is also of concern people undergoing interventional radiological procedures and coronary angiography. Renal function should be monitored in those people following the investigation.

14
Q

Which one of the following statements is true about pancreatitis? A) serum amylase is preferred over lipase the diagnosis of the above condition B) the Glasgow School is valid six hours after presentation C) enteral nutrition is of benefit to patients with the above condition D) patients with persisting organ failure and severe disease will require serial abdominal x-rays to aid and management E) if gallstones are suspected ERCP should be delayed until clinical improvement because.

A

C) Patients with pancreatitis are usually very ill and will require nutritional support to aid the recovery. This is most needed in those with severe disease with the enteral route being preferred you to lower costs and side-effects If pancreatitis is thought to be secondary to gallstones then urgent ERCP should be performed ideally less than 72 hours after presentation.

15
Q

Which one of the following statements is false? A) subdural haematoma is are more common in alcoholics B) delirium tremens occurs after 2 to 3 days of alcohol abstinence C) Wernicke syndrome is the triad of ophthalmoplegia, nystagmus and amnesia. D) Acamprosate works by binding to NMDA receptors to increase abstinence to alcohol E) mortality of untreated delirium tremens is in the order of 30%

A

C) Wernicke Syndrome is actually the triad of ophthalmoplegia and nystagmus, ataxia and encephalopathy. Amnesia would be more in keeping with Korsakoff syndrome - a progression of Wernicke encephalopathy

16
Q

Which one of the following is most useful in a patient with constipation secondary to sigmoid volvulus? A) laparotomy B) phosphate enema C) gastrgrafin meal D) flatus tube insertion E) conservative management

A

D) Conservative management of flatus tube insertion or flexible sigmoidoscopy can be all that is required to relieve the obstruction and untwist the bowel. Occasionally more aggressive surgical treatment is required.

17
Q

Which 1 of the following is true? A) DKA only occurs in patients with T1DM. B) The mortality rate of DKA is greater than that of HHS. C) DKA typically has an onset of less than 24 hours. D) A raised WCC in a patient with DKA implies co-existent infection. E) Under-dosing insulin is the commonest factor precipitating DKA.

A

C) A) Occurs in both but more common in T1DM. B) DKA mortality ~5%; HHS mortality 15-25% D) Raised WCC can be independent of infection in DKA. E) infection is the most common precipitating factor.

18
Q

A 3 year old girl is febrile and has been unwell for 12 hours. She complains of a headache and is drowsy but otherwise neurologically intact. What is the most discriminating investigation in the acute management?

A

Lumbar Puncture Meningitis commonly affect the extremes of age (60) due to impaired immunity. An LP to obtain CSF is the most important investigation when this diagnosis is considered. This should not however delay the starting of empirical anti microbial therapy. When the specific organism is identified, treatment can be modified accordingly. Fever, headache and drowsiness should make you suspicious here of this diagnosis. Atypical presentations can also occur, particularly in the very young, old or immunocompromised. In older patients, frequently, the only presenting sign of meningitis is confusion or an altered mental state. In infants, S&S can be very non-specific and may include lethargy, poor feeding, irritability and fever. A rash is noted in 80-90%, commonly 4-18h after initial symptoms and is associated with meningococcal aetiology. In bacterial meningitis, the CSF pressure is usually raised and the WBC count is elevated. Glucose level is decreased compared to the serum value and the protein level is increased. In those who are untreated, Gram stain and culture of CSF are usually positive for the causative organism. If an LP is delayed or regarded as clinically unsafe (in raised ICP) then blood samples should be obtained for culture. Of course, a heat CT should be considered before LP if there is focal neurology, new onset seizures, papilloedema, altered consciousness or any sign that May indicate raised ICP.

19
Q

2 weeks after a holiday on the Far East, a 30 year old lady presented with anorexia, fever and joint pains. Jaundice appeared a week later and on examination her liver and spleen were both enlarged and tender. What is the most likely pathogen?

A

Viral hepatitis. This is likely HAV, which is primarily transmitted via the faeco-oral route. After the virus is consumed absorbed, it replicates in the liver and is excreted in the bile (to be re-transmitted). Transmission usually precedes symptoms by about 2 weeks and patients are non-infectious 1 week after the onset of jaundice. The history can reveal risk factors such as living in an endemic area, contact with an infected person, MSM or a known food borne outbreak. (Classically associated with shellfish harvesters from sewage contaminated water) The clinical course of HAV consists of a pre-icteric phase, lasting 5-7 days, consisting characteristically of N&V, abdo pain, fever, malaise and headache. Rarer symptoms may be present such as arthralgias and even severe thrombocytopenia and signs that maybe found include splenomegaly, RUQ pain and tender hepatomegaly as well as bradycardia. The icteric phase is characterised by dark urine, pale stools, jaundice and pruritis. Jaundice peaks at 2 weeks. A fulminant course in 10.000 units although this doesn’t correlate with severity. ALT is normally > than AST.

20
Q

A 12 year old boy presents with a flu-like illness, fever, headache, vomiting, tremor on the left side if his body and weakness of his left leg. What is the most likely pathogen?

A

Polio Poliovirus infection is usually asymptomatic and when symptomatic the most common presentation is with a minor GI illness. There is no cure for poliovirus infection and treatment is primarily supportive. This patient had acute flaccid paralysis or paralytic poliomyelitis, which is the hallmark of major illness. This can rarely progress to bulbar paralysis and respiratory compromise. Paralytic poliomyelitis presents with decreased tone and motor function as well as reduced tendon reflexes and muscle atrophy of the affected limb. Lack of vaccination is a strong risk factor. Remember that there are 2 main types of polio vaccine- Sabin (oral weakened strain used in endemic regions) & Salk (inactivated poliovirus used in the rest of the world).

21
Q

These organisms are an increasing problem as nosocomial infections. They are commensalism of the GIT. Many are vancomycin resistant. What is the organism?

A

Enterococcus faecium Vancomycin resistance enterococci (VRE) - most are Enterococcus faecium. Enterococcus faecalis are mostly not VRE but are more prevalent than Enterococcus faecium. These are one of the big causes of nosocomial infections in the UK. Enterococcus faecium is a Gram positive bacteria and is a GIT commensal. It is commonly implicated in hospital acquired line and UTI. The UK top 5 hospital-acquired infections are: MRSA, VRE, ESBL (E.coli & Klebsiella), Pseudomonas & Acinetobacter.

22
Q

What is the Tensilon test?

A

The Tensilon test is used to distinguish between myasthenia gravis and lambert eaton syndrome. Edrophonium (aka Tensilon) is an AChEi that is given. It improves MG muscle weakness but has no effect to Lambert-Eaton.

23
Q

A 62-year-old man presents to the emergency department with acute onset shortness of breath and haemoptysis. He had a hip replacement operation two weeks ago has been finding the exercises to get back on its feet very difficult. On examination his tachypnoeic at rest tachycardic with the pulse of 130 minute has a pyrexia of 37.4°C and oxygen saturation of 92% on room air. Which one of the following is the most appropriate investigation in this patient? A) bronchoscopy B) DD diner C) ECG D) lateral chest x-ray E) blood cultures

A

ECG The patient has a diagnosis of a pulmonary embolism given the sudden onset of shortness of breath haemoptysis and hypoxia caused by a recent hip operation and prolonged inability. Given the height pre-test probability of a pulmonary embolism a d dimer test should not be performed because regardless of the result you investigate with a CT pulmonary angiogram or a V/Q scan to confirm your clinical suspicion. The place of the d dimer testing is in those patients with a pre-test probability of a PE and to help in its exclusion. D dimers should not be used for diagnostic reasons because although the sensitivity of the test is high the specificity is low A bronchoscopy is not useful as the haemoptysis does not reflect underlying suspicion of malignancy or infection. A low-grade pyrexia is a common finding in patience with a PE and blood cultures are therefore not the most appropriate investigation. In the acute setting a PA or AP chest x-ray is vital in the acute assessment of a patient short of breath but a lateral x-ray would not be required. The most appropriate investigation is therefore an ECG

24
Q

A 24 year old university student presents to her GP with a worsening tiredness and bruising. On examination she is extremely pale and has multiple bruises over her body. She is referred urgently to the local hospital where a blood count shows: Hb: 7.2g/dl WCC: 2.08x10(9)/l PMN: 0.74x10(9)/l Platelets: 48x10(9)/l She has no past medical history of note, takes no regular medications and there is no significant family history apart from a distant aunt who has sickle-cell disease. She does drink a few glasses of wine a week and smoke cigarettes occasionally but denies recreational drug use. She is reviewed by the haematologists who perform a bone marrow biopsy which reveals a hypoplastic marrow Which one of the following is the most likely diagnosis? A) myelodysplastic syndrome B) aplastic anaemia C) Evans syndrome D) myelofibrosis E) TTP

A

B) Aplastic anaemia The patient presented severe anaemia, leucopenia and thrombocytopenia with a hypoplastic marrow. The most likely diagnosis is aplastic anaemia. Myelodysplastic syndrome is a disease of the elderly with decreasing production and quality of press conference. Evans syndrome is the presence of autoimmune haemolysis and autoimmune thrombocytopenia together in the same patient.

25
Q

A 22-year-old patient has a history of a regular menstrual cycles over the past few years. She is well known to you and has seen you regularly with regard to her weight problem, oily skin and acne. She presents to you on this occasion with a five-month history of amenorrhoea and weight gain. Which one of the following is the most appropriate initial investigation in the above scenario? A) pelvic US B) sex hormone binding globulin free androgen levels. C) LH:FSH levels D) OGTT E) BhCG estimation

A

E) BhCG estimation Although the history for this patient is suggestive of PCOS, it is extremely important to exclude pregnancy in any patient presenting with amenorrhea before considering alternative diagnoses.

26
Q

Which 1 of the following statements is true? A) haemolytic disease of the newborn is caused by IgM Abs. B) Goodpasture’s disease is an example of a type III HS reaction. C) Anaphylactic reactions are due to IgA D) RBCs sensitised in vivo to autoAb can be detected using the direct Coombs reaction. E) HIV uses CXCR4 and CCR5 co-receptors to gain entry into CD8 T cells.

A

D) A) This occurs in Rhesus disease. Maternal IgG crosses the placenta leading to haemolysis. B) Goodpasture’s is a type II HS reaction where Ab cause damage by binding to Ag in tissues. Type III is caused by IC deposition. C) anaphylaxis involves IgE E) HIV targets CD4 cells.

27
Q

A 34-year-old man has attacks of sudden severe pain waking up the last fortnight. The pain is on the right side of his face and makes eyes water. What is the diagnosis?

A

Cluster headache Cluster headache is characterised by attacks of severe pain localised the unilateral orbital, supraorbital and or temporal areas which lasts from 15 minutes to 3 hours, and occurs with a frequency ranging from once every other day to 8 times a day. These attacks can occur at the same time period of many weeks (known as the cluster period) accompanied by ipsilateral autonomic signs. The cause is hypothalamic activation with secondary trigeminal autonomic activation (for instance lacrimation, rhinorrhoea, nasal congestion conjunctival infection and partial Horner’s i.e. ptosis and miosis). Cluster period attacks can be triggered by things like alcohol. Greater occipital nerve blockade often provide immediate relief until preventative medication take effect.

28
Q

A 40 year-old man presents with an instantaneous onset of a severe headache but by drowsiness and vomiting. What is the diagnosis?

A

Subarachnoid haemorrhage SAH (bleeding into subarachnoid space) presents with sudden severe headache patients often described as the worst headache of their life, and can often be so bad that they feel like they have been kicked in the back of the head. Half of all patients lose consciousness and eye pain with exposure to light can also be seen. Altered mental status is common. SAH occurs most commonly in the 50 to 55 age group and affects women and black people more than men and white people. The most common cause of the non-traumatic SAH is an aneurysm which ruptures. Conditions which predisposed to aneurysm formation and SAH include adult PKD, Marfan’s, NF1 and Ehlers-Danlos. Cerebral aneurysms arise around the circle of Willis. A CT scan is indicated, and if unrevealing, this should be followed by an LP. Cerebral angiography can confirm the presence of aneurysms. The patient should be stabilised and is followed by surgical clipping or endovascular coil embolisation, the choice is subject to much current controversy sparked by relatively recent research. Complications can commonly occur and include rebleeding, hydrocephalus and vasospasm.

29
Q

830-year-old woman experienced a strange feeling in the stomach, followed by stiffness and jerking in the left arm. Afterwards, she felt drowsy but remembers everything. What is the diagnosis?

A

Partial seizure. The feeling this woman felt in her stomach before (Aura), the description of the arm, incontinence and feeling “worn out” afterwards make this a seizure. Seizures can happen in anyone but epilepsy is the tendency to have repeated, unprovoked seizures. Epileptic seizures can be split into focal (partial or localised) and generalised (involving both hemispheres). Generalised seizures can be further divided into absence (typical or atypical), myoclonic, chronic, tonic, tonic-clonic and atonic types. Be sure to exclude other causes, such as a space occupying lesion with a CT or MRI scan. There may also be tongue biting. Always think of “before, during and after” when taking history. History taking is the most important aspect of diagnosis. Antiepileptic monotherapy is the preferred treatment.

30
Q

A 42-year-old man who lives alone presented with confusion and ataxia. Both his hands had a Dupytren’s contracture. What is the likely diagnosis?

A

Thiamine/B1 deficiency This patient has Wernicke’s encephalopathy. Wernicke’s is due to a acute thiamine deficiency, which is a problem in alcoholics (the Dupytren’s contracture here is a clue to alcohol use). Others at risk include those with AIDS, cancer and treatment with chemotherapy, malnutrition and GIT surgery, especially bariatric procedures. It is a clinically underdiagnosed condition. The classic triad is a mental change, ophthalmoplegia and gait dysfunction which is actually only seen in 10% of cases. In reality, the manifestations are varied and a high index of suspicion is needed. This is an emergency and treatment is with parenteral replacement of thiamine this avoids permanent neurological damage including later development of Korsakoff psychosis, which is irreversible. Note that thiamine should be given before dextrose! Magnesium deficiency also need to be corrected as it is a cofactor in the functioning of thiamine dependent enzymes.

31
Q

Step 1 in a 40 year old Caucasian male commencing anti-hypertensive therapy?

A

ACEi

32
Q

Step 2 in anti-hypertensive treatment of a 40 year old Caucasian male.

A

CCB or Thiazides diuretic

33
Q

Step 3 in anti-hypertensive management of a 40 year old Caucasian male.

A

CCB & diuretic

34
Q

Step 4 antihypertensive management of a 40 year old Caucasian male

A

Alpha blocker or spironolactone

35
Q

A 55 year old Afro-Caribbean needs to commence on an anti-hypertensive agent.

A

CCB or diuretic

36
Q

Hypertension due to increased renin secretion is common in men or women?

A

Women

37
Q

For each patient choose the most appropriate treatment: A 40 year old man with diabetes proteinuria and hypertension of 148/98mmHg

A

ACEi This man has concomitant diabetes and chronic renal disease and stage one hypertension. The first line treatment is monotherapy with an ACE inhibitor. ACEi are renoprotective and decreases the progression of proteinuria in diabetics.

38
Q

For each patient choose the most appropriate treatment: A 30-year-old woman who has developed hypertension early in her first pregnancy blood pressure is 150/95. She does not have proteinuria or oedema.

A

Methyldopa The first-line agent for hypertension in pregnancy is methyldopa. This drug acts by stimulating alpha-2 receptors which have a sympatholytic effect on the vasculature. The lowest dose should be titrated upwards into a therapeutic effect is achieved or an adverse event prevent upping the dose further. The second-line agent is labetalol.

39
Q

For each patient choose the most appropriate treatment: A 60-year-old woman has hypertension with no comorbidities. Mean blood pressure is 170/110mmHg.

A

Thiazide & ACEi This patient has stage to hypertension and will require the initiation of two concurrent antihypertensive medications at the same time. The first line is it thiazides diuretic plus ace inhibitor. A combination of non-dihydropyridine CCBs with beta-blockers should be avoided due to the risk of AV block. However as second line options, ACEi plus CCB/beta-blockers can be used thiazides plus beta-blockers.

40
Q

For each patient choose the most appropriate treatment: A 60 year-old man with atrial fibrillation and hypertension of 155/95

A

Beta-blocker This is stage one hypertension complicated by AF. The first choice is a beta-blocker and the second choice is a non-DHP CCB. If stopping treatment, beta-blockers should be weaned off as abruptly stopping beta-blockers has been associated with exacerbations of angina and heart attacks have also occurred. If the patient has COPD or asthma then CCBs may be preferred.

41
Q

For each patient choose the most appropriate treatment: A 45-year-old man has collapsed. He has papilloedema and multiple haemorrhages on funduscopy. He has proteinuria. CT brain is normal. BP is 250/140.

A

Labetalol. This is a hypertensive emergency with BP >210/130. Treatment should be started without waiting for further tests as this is a medical emergency and BP must be lowered over the minutes to hours with IV medications in ITU setting. Oral medications can be given afterwards to win the patient off IV. Guidelines indicate the goal is to reduce MABP by no more than 25% in the first hour, then if stable to 160/100 or less within the next 2 to 6 hours. This is malignant hypertension associated here with retinopathy. Head CT denies evidence of infarct or haemorrhage. The proteinuria here is a sign of real failure. The first line treatment in malignant hypertension is IV labetalol. Onset of action 5 to 10 minutes, duration of action to 3-8 hours.

42
Q

True or False The association between blood pressure and stroke is stronger than between blood pressure and coronary artery disease.

A

True

43
Q

The alternative name for Conn’s syndrome is? And the specific treatment is?

A

Primary hyperaldosteronism. Spironolactone. (K sparing diuretic = competitive antagonist of aldosterone [mineralocorticoid] receptor

45
Q

An 18 year old man presents with a night time cough and SOB while playing football. This has got progressively worse over the previous 2 months. Most likely diagnosis? A.Oesophageal reflux B.COPD C.Asthma D.Foreign body E.Sarcoidosis F.Angiotensin converting enzyme inhibitor G.Postnasal drip H.Bronchiectasis I.Carcinoma of bronchus J.Tuberculosis

A

Asthma SOB and the cough, which may wake the patient from sleep combined with the patient’s age and progressive course suggest asthma. Examination can show an expiratory wheeze but may be normal and treatment is step wise based on BTS guidelines. It is worth noting that in severe exacerbations, the chest may be silent. Night symptoms occur in more severe asthma and symptoms can be exacerbated by exercise. Diagnosis is supported by PEFR variation of at least 20% over 3 days in a week over several weeks or an increase of at least 20% to treatment.

46
Q

A 30 year old man, a lifelong non-smoker, presents with a history of at least 6 months of purulent sputum. He has had regular chest infections since an attach of measles aged 14. What is the most likely diagnosis? A.Oesophageal reflux B.COPD C.Asthma D.Foreign body E.Sarcoidosis F.Angiotensin converting enzyme inhibitor G.Postnasal drip H.Bronchiectasis I.Carcinoma of bronchus J.Tuberculosis

A

Bronchiectasis. Bronchiectasis is permanent bronchi dilation due to bronchial wall damage and loss of elasticity. It is often as a consequence of recurrent/severe infections and most present with chronic productive mucopurulent cough. The most common identifiable cause is CF. Chest CT is the diagnostic test. Diagnosis is aided by sputum analysis.

47
Q

A 70 year old man with a history of 10kg weight loss over the previous 3 months. More recently, he developed acute lower back pain. He presents to A&E with coughing and sputum. CXR shoes left lower lobe pneumonia. Most likely diagnosis? A.Renal failure B.Malignancy C.Anorexia nervosa D.Malabsorption E.Addison’s disease F.Liver failure G.HIV H.Depression I.Tuberculosis J.Infestation with helminths K.Cardiac failure L.Diabetes mellitus M.Hyperthyroidism

A

Malignancy This person likely has lung cancer which has results in the significant weight loss of 10kg. This is post-obstructive pneumonia which is common in lung cancer patients and is caused, most of the time, by a large and centrally obstructing tumour. It is essential to relieve this obstruction in this case and many techniques can be tried. There is also bone pain here in the lower spinal column which is due to mets. The prognosis here is not good.

48
Q

A 40 year old lady with multiple painful lumps in her breast, which are painful & tender premenstrually Most likely diagnosis? A.Breast Abscess B.Carcinoma of the breast CFibroadenosis D.Fibroadenoma E.Duct Ectasia F.Lipoma G.Gynaecomastia H.Sebaceous cyst I.Breast Cyst

A

Fibroadenosis Fibrocystic breasts are characterised by ‘lumpy’ breasts associated with pain which fluctuates with the menstrual cycle (it is worse during the luteal phase of menses). Risk factors include obesity, nulliparity, HRT and late onset menopause and first childbirth. It is a diagnosis of exclusion, and is considered to be an exaggerated physiological phenomenon rather than a disease (54% of clinically normal breasts are found on autopsy to have fibrocystic changes). Symptoms typically arise between the 3rd and 4th decases of life. There may also be a nipple discharge, which can be suspicious if bloody or profuse etc and may indicate the presence of an intraductal papilloma, cancer, or duct ectasia. Cysts can be aspirated if symptomatic (asymptomatic or small ones do not require intervention). If the aspirate is straw coloured and completely aspirated, there is no need for cytology, but if the aspirate is bloody, cytology or biopsy is needed to exclude cancer. There is improvement of mastalgia and cysts at menopause and until then it runs a chronic relapsing course.

48
Q

A 25 year old lady returning for Australia presents with an acutely painful left calf. US confirms a DVT extending above the popliteal veins. She has recently missed a period. Most appropriate management? A.Embolectomy B.Fondaparinux (FXa inhibitor) C.Start warfarin therapy D.Intravenous heparin E.Vena cava filter F.Check INR and continue warfarin G.Subcutaneous low molecular weight heparin H.Subcutaneous low molecular weight heparin I.Observation in hospital J.Antiembolism stocking K.Reassure and discharge L.Reassure and discharge

A

SC LMWH Women developing a DVT during pregnancy can be treated with heparin or LMWH. However, LMWH is preferred due to more dependable pharmacokinetics.

49
Q

A woman who is on warfarin for a confirmed right calf DVT develops increasing pain and swelling of that leg. This is the 3rd time this has happened. Repeat imaging shows thrombus limited to the calf. Most appropriate management? A.Embolectomy B.Fondaparinux (FXa inhibitor) C.Start warfarin therapy D.Intravenous heparin E.Vena cava filter F.Check INR and continue warfarin G.Subcutaneous low molecular weight heparin H.Subcutaneous low molecular weight heparin I.Observation in hospital J.Antiembolism stocking K.Reassure and discharge L.Reassure and discharge

A

SC LMWH Patients with recurrent thromboembolism despite on warfarin should be given heparin or primarily LMWH. They should be given for at least 5 days until INR is between 2-3. Fondaparinux (fX inhibitor) would be used instead if there was a high chance of delayed HIT. Warfarin is continued and efforts must be made to work out how this patient has developed a DVT despite warfarin therapy. There may be sub therapeutic dosing, the presence of malignancy or anti phospholipid syndrome. If there is documented thrombosis progression despite adequate anti coagulation, without HIT and other causes excluded, an IVC filter may be indicated but evidence of its efficacy have been debated.

50
Q

5 indications for dialysis in acute kidney injury

A

AEIOU Acidemia due to metabolic acidosis Electrolyte imbalance - hyperkalemia Intoxication - SLIME poisons: Salicylic acid, Lithium, Isopropanol, Mg-containing laxatives, Ethylene Glycol Overload of fluid refractory to diuretics Uraemic complications: pericarditis, encephalopathy, GI bleed

51
Q

Miss K is a 45 year old retired kitchen lady who presents to A+E with abdominal discomfort which has been getting worse over the past few weeks. She says she feels tired, bloated and has lost her appetite. Findings show antimitochrondrial antibodies are present and serum alkaline phosphatase is raised Most likely diagnosis? Gilbert’s Syndrome Hepatitis C Primary biliary cirrhosis Hepatocellular carcinoma Hepatorenal syndrome Hepatitis E Hepatitis D Hepatitis A Hepatitis B Portal hypertension Gall stones Pyogenic abscess Alcoholic liver disease GORD

A

PBC Primary biliary cirrhosis (PBC) is a chronic condition where the intrahepatic small bile ducts are progressively damaged (and eventually lost) occuring on a background of portal tract inflammation. Fibrosis develops, ultimately leading to cirrhosis (which is defined as fibrosis with nodular regeneration). It is widely believed to be autoimmune in aetiology as almost all patients have antimitochrondrial antibodies. This is the give away point in this question which is otherwise a non-specific presentation. Elevated ALP suggests cholestasis.

52
Q

A 22 year old man walks into A&E, he was involved in a fight in a bar. As he left the bar he complained of drowsiness and double vision worst when walked down stairs. Which nerve is affected? A.Cranial nerve Vc B.Cranial nerve Vb C.Cranial nerve III D.Optic tract E.Cranial nerve X F.Cranial nerve I G.Cranial nerve IX H.Cranial nerve II I.Cranial nerve Va J.Cranial nerve XI K.Cranial nerve VI L.Cerebellum MCranial nerve VIII N.Cranial nerve IV OCranial nerve XII P.Cranial nerve VII

A

CN IV This man is complaining of double vision. You should already be thinking III, IV or VI which are the cranial nerves responsible for eye movements. CN IV innervates the superior oblique muscle which controls depression, intorsion and adduction of the eye. This is vertical diplopia (worse when walking down the stairs) hence the cause is a lesion in CN IV. The trochlear nucleus is located in the midbrain at the level of the inferior colliculus and fasciles decussate at the medullary velum to exit at the dorsal midbrain. The nerve enters the orbit through the superior orbital fissure. Testing is done at the bedside by asking the patient to follow the examiner’s finger with the eyes, while observing eye movements and asking the patient to report any diplopia. Remember that diplopia is maximal in the direction of action of the paralysed muscle (which makes sense if you think about it) and the outer image is the image which is false, and disappears when the ipsilateral eye is covered up.

53
Q

A 79 year old lady was brought to A&E after a fall. On examination, when asked to close her eyes, her eyeballs rolled upwards and she was unable to close her eyes. Which nerve is affected? A.Cranial nerve Vc B.Cranial nerve Vb C.Cranial nerve III D.Optic tract E.Cranial nerve X F.Cranial nerve I G.Cranial nerve IX H.Cranial nerve II I.Cranial nerve Va J.Cranial nerve XI K.Cranial nerve VI L.Cerebellum MCranial nerve VIII N.Cranial nerve IV OCranial nerve XII P.Cranial nerve VII

A

CN VII This is someone who cannot close their eyes. Eye closure relies on orbicularis oculi which is innervated by the facial nerve (VII). It is the most common CN mononeuropathy and the most common cause of a peripheral facial palsy is Bell’s palsy. This is why when testing CN VII you ask a patient to screw their eyes up tightly and resist you trying to open them as well as asking them to do other seemingly odd things like raising their eyebrows against resistance, showing you their teeth and pursing their lips. The facial nerve is both motor and sensory (via the nervus intermedius).The nerve exits the cranium through the stylomastoid foramen and enters the parotid, where it splits into 5 branches – temporal, zygomatic, buccal, mandibular and cervical.

54
Q

A 7 year old girl presents with at 39.3oC fever, rash and neck stiffness. You suspect raised intracranial pressure. You are in a DGH late at night and the radiographers are not on call. What is the best diagnostic investigation to order? A.Laparoscopy B.ABGs C.Transvaginal ultrasound D.FBC E.PET scan F.Compression USS G.Helical CT scan H.CXR I.Lumbar puncture J.MRI Head K.Urinary beta hCG L.AXR M.CT head N.Blood cultures

A

Blood cultures. You suspect that this little girl has meningitis with fever, nuchal rigidity and presumably a petechial/purpuric rash, typically associated with meningococcal meningitis. If there is evidence of raised intracranial pressure, then a CT head should be considered before LP. There is this suspicion in the question, but the radiographers have gone home. That’s not to say that if you suspect raised ICP and feel an LP would not be safe to do, that you should go ahead and perform a lumbar puncture anyway. In this case, as a responsible doctor, you would order blood cultures as LP is clearly clinically unsafe. This is less accurate as the result may be influenced by previous antimicrobials and positive blood cultures have been reported in only 40-70% of clinically suspected meningococcal infection. But, unless you want to get struck off by the GMC and imprisoned for causing uncal herniation… it’s the best choice you have. The main diagnostic investigation for meningitis is still to do is an LP except you simply cannot do it in this case. This would show low CSF glucose, raised CSF protein and positive CSF culture or gram stain or meningococcal antigen. Given the signs and symptoms here are very striking, you would not delay treatment. Until you know which organism is causing the symptoms and its sensitivities, empirical therapy can be started with, for example, vancomycin and ceftriaxone. Once the organism is confirmed and you know sensitivities, you can begin targeted antibiotic therapy. For example, if the infection is confirmed to be Neisseria and is penicillin sensitive, you can start benzylpenicillin or ampicillin intravenously. All patients should receive adequate supportive care throughout.

55
Q

A 40 year old man presents with acute colicky pain in his back radiating to his left flank. He has dysuria. What is the best diagnostic test? A.Laparoscopy B.ABGs C.Transvaginal ultrasound D.FBC E.PET scan F.Compression USS G.Helical CT scan H.CXR I.Lumbar puncture J.MRI Head K.Urinary beta hCG L.AXR M.CT head N.Blood cultures

A

Helical CT scan This patient has renal colic which classically presents with severe flank pain radiating to the groin. Microscopic haematuria is present in up to 90% of cases. Up to 85% of stones are visible on a plain KUB although urate stones are radiolucent. If the stone is radio-opaque, calcification will be seen within the urinary tract. In pregnancy, a renal USS is first line. The IVP has now been replaced by the CT scan which is the new diagnostic standard. A non-contrast helical (or spiral) CT is preferred due to high sensitivity and specificity and accurately determines presence, site and size of stones. Stones are analysed after they are extracted or when they are expelled to check their composition. It is worth noting that in all females of child bearing age, a urine pregnancy test is necessary to exclude an ectopic pregnancy.

56
Q

68 year old male. Complains of fever, night sweats and weight loss. On examination has hepatosplenomegaly and peripheral lymphadenopathy. On further investigation, he was also found to have intra-abdominal and pelvic lymphadenopathy. Most appropriate answer: B12 deficiency Iron deficiency anaemia Acute myeloblastic leukaemia Hodgkin’s lymphoma Folate deficiency Infectious mononucleosis Acute lymphocytic leukaemia Myeloma Hepatocellular carcinoma

A

Hodgkin’s lymphoma This is a lymphoma and the main differential is between Hodgkin’s and Non-Hodgkin’s. It is actually very difficult to differentiate the two clinically but there is only one option here and the question is testing your ability to tell that is is lymphoma, at this stage. Possible differentiating signs or symptoms include the bimodal age distribution of Hodgkin’s and that Hodgkin’s is associated with pruritis and alcohol-triggered pain. But again, the two are clinically similar. Lymphoma may like this case present with systemic symptoms of fever, drenching night sweats (which classically require patients to change their clothes) and weight loss due to the hypermetabolic state. There may also be SOB, cough, general malaise and abdominal discomfort (hepatomegaly, splenomegaly, lymphadenopathy, bowel involvement), headache, change in mental status, ataxia (if the CNS is involved), fatigue and chest pain (from anaemia), bone pain (from bone involvement) etc. There are many risk factors such as EBV, which is associated with Hodgkin’s lymphoma, Burkitt’s lymphoma, AIDS-related primary CNS lymphoma and nasal NK/T-cell lymphoma. Treatment depends on the histological subtype and the severity of symptoms at presentation.

57
Q

A teenage boy presents with haematuria. It has been noted that his hearing has been worsening over recent years. He has a family history of early death. Most likely diagnosis: A.Nephrocalcinosis B.Fanconi Syndrome C.Chronic Pyelonephritis D.Acute Nephritic Syndrome E.Acute Pyelonephritis F.Nephrotic Syndrome G.Alport’s syndrome H.Nephrogenic Diabetes Inspidus I.Goodpasture’s syndrome J.Urinary tract Obstruction

A

Alport’s syndrome Alport’s syndrome is a rare familial nephropathy due to abnormal type IV collagen and may either be X-linked, autosomal recessive or autosomal dominant. Gross haematuria is a common presenting feature, often precipitated in a child by an infection. The age of onset of chronic renal failure is variable in this condition and it is frequently associated with sensorineural hearing loss, like in this case here. Complications such as renal failure and deafness typically occur more often in young males with the X-linked form of disease. The FH of early death may be linked to progression to end-stage renal failure. The mainstay of treatment is to monitor and treat renal disease. Female carriers of X-linked mutations have a lifetime risk of developing hypertension and renal disease which is significant. In EMQs if a child, especially male, has renal disease associated with deafness, then it is likely to be Alport’s. There is also a characteristic central retinopathy present in 40-70% of males which is said to be pathognomic of the disease.

58
Q

A 33 year old man presents with increased frequency of micturition and dysuria. He thinks his urine has a more unpleasant odour than usual. In his past history, he mentions that he was diagnosed with “reflux” as a child. On examination, the pain is identified as being in the suprapubic region; he was found to be hypertensive, and haematuria was confirmed on investigation. Most appropriate answer: A.Nephrocalcinosis B.Fanconi Syndrome C.Chronic Pyelonephritis D.Acute Nephritic Syndrome E.Acute Pyelonephritis F.Nephrotic Syndrome G.Alport’s syndrome H.Nephrogenic Diabetes Inspidus I.Goodpasture’s syndrome J.Urinary tract Obstruction

A

Chronic pyelonephritis Chronic pyelonephritis is most commonly caused by chronic vesicoureteric reflux which this man gives a PMH of. Other risk factors include a history of acute pyelonephritis, which can often develop into the chronic type if inadequately treated or if acute cases are recurrent, renal obstruction (which may be due to urinary tract abnormalities, BPH or stones like a staghorn calculus) and DM. The damage is irreversible, unfortunately, and there is no specific treatment, and this condition may result in ESRF. Diagnosis is clinical and confirmed with imaging studies. The history of unpleasant smelling urine (which may be cloudy) along with urinary symptoms (suggesting UTI) and suprapubic pain (though there may be flank/back pain and tenderness) is also indicative. The patient’s condition may be complicated here by urinary tract infections and associated symptoms.

59
Q

A 56 year old vagrant presents with perifollicular skin bleeding & prolonged bleeding time Most likely diagnosis? A.Iatrogenic B.Sepsis C.Paroxysmal nocturnal haemoglobinuria (PNH) D.Pre-eclampsia E.Amyloidosis F.Malignant hypertension GDisseminated intravascular coagulation (DIC) H.Scurvy I.Thrombotic thrombocytopenic purpura (TTP) J.Haemorrhagic fever K.Cushing’s Disease L.Idiopathic thrombocytopenic purpura (ITP) M.Henoch-Schonlein syndrome N.Haemophilia O.Ehlers- Danlos syndrome

A

Scurvy Scurvy is basically vitamin C deficiency and is now a rare disorder and really only affects people who are subject to famine or wartime displacement in places like Africa. Vitamin C is needed for collagen synthesis so lack of this vitamin leads to impaired synthesis of collagen which can lead to complications. These complications include bleeding complications like spontaneous petechiae and bruising, friable gingiva and loose teeth, bone pain and joint effusions. Complications include heart failure, encephalopathy and entrapment neuropathies. To aid diagnosis, serum, leukocyte and whole blood ascorbic acid levels can be measured. This condition can be fatal if it is left untreated but complete recovery is expected on going vitamin C replacement in the form of ascorbic acid.

60
Q

A 35 year old woman presents with purpura, fever, haemolytic anaemia, microvascular thrombosis, renal failure & mild stroke. Blood film shows fragmented red cells & clotting screen is normal. Most likely diagnosis: A.Iatrogenic B.Sepsis C.Paroxysmal nocturnal haemoglobinuria (PNH) D.Pre-eclampsia E.Amyloidosis F.Malignant hypertension GDisseminated intravascular coagulation (DIC) H.Scurvy I.Thrombotic thrombocytopenic purpura (TTP) J.Haemorrhagic fever K.Cushing’s Disease L.Idiopathic thrombocytopenic purpura (ITP) M.Henoch-Schonlein syndrome N.Haemophilia O.Ehlers- Danlos syndrome

A

TTP (thrombotic thrombocytopenic purpura) The title really tells you what this condition is. TTP is a diagnosis you should consider in any patient with anaemia and thrombocytopenia as 95% of cases are fatal if left untreated. The symptoms are usually non-specific though before plasma exchange was available as a treatment option, most patients developed 5 characteristic features (a pentad) of: microangiopathic haemolytic anaemia, thrombocytopenia with purpura, acute renal insufficiency, neurological abnormalities and fever. It is rare however for all of these to be present now. Most patients present aged 30-50 and there is usually a non-specific prodrome followed on the components of the pentad. The peripheral blood film here shows evidence of microangiopathic HA with schistocutes being present. Even though not reported, thrombocytopenia would also be seen on this film. The mainstay of treatment, as hinted before, is plasma-exchange therapy

61
Q

A 20 year old man presents with colicky abdominal pain, arthritis & polyarthritis following a chest infection. He also has purpura over his buttocks & legs Most likely diagnosis A.Iatrogenic B.Sepsis C.Paroxysmal nocturnal haemoglobinuria (PNH) D.Pre-eclampsia E.Amyloidosis F.Malignant hypertension GDisseminated intravascular coagulation (DIC) H.Scurvy I.Thrombotic thrombocytopenic purpura (TTP) J.Haemorrhagic fever K.Cushing’s Disease L.Idiopathic thrombocytopenic purpura (ITP) M.Henoch-Schonlein syndrome N.Haemophilia O.Ehlers- Danlos syndrome

A

Henoch-Schonlein syndrome Henoch-Schonlein purpura is the most common vasculitis in childhood and in all cases there is a rash of palpable purpura which are typically non-blanching. If there is no rash, then it is not HSP. They are normally 2-10mm in diameter and arethe extravasation of blood into the skin. They can occur anywhere on the body but are usually concentrated on the lower extremities. Half of all patients have abdominal pain and arthralgias are commonly present (found in about 80%) and often associated with oedema. The joints most often affected are the knees and ankles. About half will show signs of renal disease such as proteinuria or haematuria. Risk factors for this condition include being male, age 3-15 and history of prior UTI. Complications can occur and the most common cause of death is renal failure.

62
Q

A 65 year old man presents with a 7 month history of vague epigastric pain. The pain is made worse immediately after eating and is relieved by vomiting. The patient avoids spicy foods and has lost 9kg in weight. On examination there is midline epigastric tenderness. Most likely diagnosis A.Pancreatitis B.Pancreatic carcinoma C.Gastritis D.Gastric ulcer E.Duodenal ulcer F.GORD G.Acute cholecystitis H.Oesophagitis I.Gastric carcinoma

A

Gastric carcinoma Weight loss is one of the most common presenting symptoms in patients with gastric cancer. Epigastric pain is present in about 80% and may resemble that of a gastric ulcer. Although commonly mentioned in EMQs, lymphadenopathy is an uncommon presentation. On examination there may be the presence of a left supraclavicular node (Virchow’s node), a periumbilical nodule (Sister Mary Joseph’s nodule) or a left axillary nodule (Irish node). These are rare findings. A mass may be palpable in advanced disease. More proximal tumours can present with dysphagia. Acanthosis nigricans, a black velvety rash, may also be present. Strong risk factors include pernicious anaemia, Helicobacter pylori and the consumption of N-nitroso compounds found in cured meats. The peak incidence occurs between 50-70 and men are twice as likely to have gastric cancer. Most are adenocarcinomas. The first test to order for suspected gastric malignancy is an urgent upper GI endoscopy with biopsy of the lesion. The mainstay of treatment is surgical resection unless there is evidence of metastatic disease.

63
Q

A 45 year old man has a 6 month history of recurrent attacks of severe epigastric pain lasting from 2 to 6 weeks. The pain occurs 2 hours after food and is relieved by drinking milk. The pain often awakes the patient at night. On examination there is a localised deep tenderness to the right of the epigastrum. Most likely diagnosis? A.Pancreatitis B.Pancreatic carcinoma C.Gastritis D.Gastric ulcer E.Duodenal ulcer F.GORD G.Acute cholecystitis H.Oesophagitis I.Gastric carcinoma

A

Duodenal ulcer Epigastric pain and tenderness related to eating a meal is typical of a peptic ulcer. The pain is generally relieved by drinking milk. 80% are duodenal and 20% are gastric. Ulcers may cause iron deficiency anaemia and associated symptoms may feature. Key risk factors are NSAID use, H. pylori infection, smoking and a family history of PUD. Zollinger-Ellison syndrome should be considered if there are multiple ulcers or ulcers refractory to treatment. Gastric ulcers classically cause pain which is exacerbated by eating and immediately relieved on vomiting. There is usually also weight loss due to a fear of food and its association with pain. Duodenal ulcers are classically made worse by hunger and are relieved by eating and the patient may wake at night with the pain. As a result, weight gain is typically a feature. In reality, it is difficult to differentiate the site of the ulcer based on these features. The most specific and sensitive test is an upper GI endoscopy which is initially ordered if the patient has ‘red flag’ symptoms, is >55 years of age or fails to respond to treatment. Duodenal ulcers rarely undergo malignant transformation so do not require a compulsory biopsy but gastric ulcers require biopsies to rule this out. In patients who are 55 or younger without ‘red flags’, testing for Helicobacter pylori (breath testing with radiolabelled urea or stool antigen testing) is necessary. Management is aimed at correcting the underlying cause such as discontinuing NSAIDs. H. pylori eradication should be started if the organism is present with triple therapy. Otherwise, a PPI is indicated.

64
Q

A 25 year old woman who complains of heat intolerance, anxiety & weight loss, despite a good appetite. You find a fine tremor in her hands & a pulse of 120. There is a smooth swelling, 5cms by 3cms, centred on the midline of her neck. You hear a bruit when you place the stethoscope diaphragm over it. Most appropriate management? A.Amoxicillin B.Excise for biopsy C.Sialogram D.Carbimazole E.Reassure & explain why no active management necessary F.Full blood count & Paul Bunnell G.Thyroxine H.Upper GI endoscopy I. Technetium thyroid scan

A

Carbimazole This woman has hyperthyroidism. The smooth midline swelling and the audible bruit point towards a diagnosis of Graves’ disease. In countries where sufficient iodine intake is not an issue, Graves’ disease is the most common cause of hyperthyroidism. Peripheral manifestations such as ophthalmopathy, pretibial myxoedema and hyperthyroid acropachy do not occur with other causes of hyperthyroidism. Acropachy manifests as clubbing with soft tissue swelling. Pretibial myxoedema is almost always associated with ophthalmopathy. A thyroid isotope scan such as Tc-99 pertechnetate is used to differentiate the diffuse enlargement of Graves’ with the patchy uptake of a multinodular goitre. This is not necessary based on the examination findings. Treatment aims to normalise thyroid function and is achieved by radioactive iodine, antithyroid medications or with surgery. They are all effective and relatively safe options. Symptomatic therapy is given with beta blockers such as propranolol.

65
Q

A woman of 20 with a week’s history of sore throat & fever. You find large smooth tender sub-mandibular bilateral lymph glands. Most appropriate management A.Amoxicillin B.Excise for biopsy C.Sialogram D.Carbimazole E.Reassure & explain why no active management necessary F.Full blood count & Paul Bunnell G.Thyroxine H.Upper GI endoscopy I. Technetium thyroid scan

A

FBC + Paul Bunnell Infectious mononucleosis is caused by EBV and is characterised by fever, pharyngitis and lymphadenopathy. A FBC will show an atypical lymphocytosis. Confirmation of IM involves detection of the existence of heterophile antibodies using the Paul Bunnell monospot. A more accurate test is a serological test detecting EBV specific antibodies. Treatment is usually symptomatic but IM carries rare but potentially life threatening complications.

66
Q

70 year old woman who complains of palpitations, intolerance to heat, diarrhoea & weight loss. You find an irregular goitre in her neck that moves up and down when the patient swallows. There is no stigmata of Graves’ disease Most appropriate management? A.Amoxicillin B.Excise for biopsy C.Sialogram D.Carbimazole E.Reassure & explain why no active management necessary F.Full blood count & Paul Bunnell G.Thyroxine H.Upper GI endoscopy I. Technetium thyroid scan

A

Technetium thyroid scan. This is a case of toxic multinodular goitre. It is most common in older patients and is associated with head and neck irradiation and iodine deficiency. TSH is the initial screening test and if supressed, T4/T3 levels are measured. As the peripheral stigmata of Graves’ disease is absent, thyroid scan and uptake is indicated. I-123 is the preferred isotope but as this option is not given, Tc-99 can be used although there is a risk of false positive images. The scan will show multiple hot and cold areas consistent with areas of autonomy and areas of suppression. Definitive treatment is commonly given in the form of radioactive iodine.

67
Q

55 year old woman who complains of dysphagia is found to have a 10cms in width & 5cms in height bilateral symmetrical non-tender nodular mass in the front of her neck. Investigations show that she is euthyroid. Most appropriate management? A.Amoxicillin B.Excise for biopsy C.Sialogram D.Carbimazole E.Reassure & explain why no active management necessary F.Full blood count & Paul Bunnell G.Thyroxine H.Upper GI endoscopy I. Technetium thyroid scan

A

Upper GI endoscopy. The bilateral mass in the last part of this question is a multinodular goitre which does not need a biopsy. The patient is euthyroid. If the nodule was unilateral, then a biopsy is essential to establish or exclude malignancy. However, her dysphagia needs to be investigated with an upper GI endoscopy.

68
Q

A 63 year old man who has recently had chemotherapy for bowel cancer is suffering from diarrhoea. Most appropriate management? A.Codeine phosphate B.Antispasmodics C.Senna D.Amoxicillin E.Doxycycline F. Oral rehydrating fluids G.Mesalazine H.Lactulose I.Intravenous dextrose J.Ciprofloxacin K.Paracetamol L.Intravenous saline M. Hemicolectomy

A

Codeine phosphate. Codeine phosphate is an opiate and will kill two birds with one stone by offering effective pain relief and treating the diarrhoea. Codeine is used for mild to moderate pain, diarrhoea and as an antitussive.

69
Q

54 year old Asian woman with type 2 diabetes for 15 years. She comes to the clinic complaining of ankle swelling. On examination, BP 170/95, JVP not raised & bilateral oedema to the knees. Albumin is low. Most appropriate investigation? A.Coagulation screen B.Venous doppler studies C.Full blood count D.24 hr urine protein E.Lymphangiogram F.Plasma creatinine G. Arterial doppler studies H.Chest X-Ray I.Pelvic ultrasound J.Liver function tests

A

24hr urine protein The most common cause of nephrotic syndrome in adults with long standing diabetes is diabetic nephropathy. However, non-diabetic renal disease cannot be excluded. Nephrotic syndrome is defined by the presence of proteinuria (>3.5g/24h), oedema and hypoalbuminaemia. Some definitions add hyperlipidaemia. Do not confuse this with nephritic syndrome. Diagnosis is made by quantification of proteinuria with a 24 hour urine collection, although now it is common to do a spot urine protein-to-creatinine ratio for practical reasons.

70
Q

66 year old man presents with swelling of his right leg to the knee. He had had a right hip replacement 5 weeks previously. Most useful investigation? A.Coagulation screen B.Venous doppler studies C.Full blood count D.24 hr urine protein E.Lymphangiogram F.Plasma creatinine G. Arterial doppler studies H.Chest X-Ray I.Pelvic ultrasound J.Liver function tests

A

Venous doppler studies Recent surgery, especially orthopaedic surgery, is a strong risk factor for developing a DVT. Other strong risks include active malignancy, pregnancy, obesity and coagulopathies such as factor V Leiden. A Wells score is determined in all patients with a suspected DVT with the condition being likely if the score is 2 or more. If the Wells score is

71
Q

25 year old female with acute onset of chills, fever, cough with brown phlegm for three days. On examination she appears toxic, temperture 40degrees C, reduced breath sounds, bronchial breathing and stony dullness left lung base. Most likely diagnosis A.COPD B.Pneumonia C.Pleural mesothelioma D.Sarcoidosis E.Pneumothorax F.Carcinoma of Bronchus G.Pulmonary embolus H.Rheumatoid arthritis I.Aspiration pneumonia J.Lung metastases K.Acute asthma L.Pulmonary oedema M.Chest injury with rib fractures N.Pulmonary tuberculosis

A

Pneumonia The rusty coloured phlegm is hinting at a pneumococcal pneumonia.The patient has presented with common symptoms of fever, chills and a cough. There may also be SOB, rigors and pleuritic chest pain. The most specific and sensitive test is a CXR (PA and lateral) and initial treatment of a CAP is empirical with antibiotics. Often diagnosis is made solely on history and examination findings. Bronchial breathing, reduced breath sounds and the presence of a left sided parapneumonic effusion all indicate a pneumonia. Management is guided by the patient’s CURB-65 score.

72
Q

A 24 year old woman following a viral infection was diagnosed as having idiopathic thrombocytopaenia. She presents to A&E & complains of multiple bruising & rectal bleeding. She is on oral prednisolone 30mg/day. Her Hb is 12.5g/dl. Most likely management A.Anterior resection B.Colostomy C.IV corticosteroids D.Loperamide (Imodium) E.Topical GTN F.IV immunoglobulin G.High fibre diet H.Haemorrhoidectomy I.Hemicolectomy

A

IV immunoglobulins ITP is thought to be due to an autoimmune phenomenon. Treatment is based on the patient’s platelet count and bleeding symptoms. This patient has severe active bleeding and myst be started on IVIG plus corticosteroids, which she is already on. Platelet transfusions should be considered with tranexamic acid as an adjunct.

73
Q

An 85 year old who is known to be hypertensive & has mild impaired renal function presents with signs of dehydration & undergoes a laparotomy for small bowel obstruction. Most likely analgesic NOT to be used in the post-operative period: Diclofenac Paracetamol Tramadol Codydramol Morphine Epidural bupivacaine fentanyl

A

Diclofenac NSAIDS may impair renal function and provoke renal failure, especially in patients with pre-existing impairment. NSAIDs should be avoided if possible in these patients or used with caution at the lowest effective dose for the shortest possible time. The mechanism of damage involves reducing creatinine clearance. NSAIDs are also contraindicated in asthmatics as it causes bronchospasm due to the accumulation of leukotrienes.

74
Q

A 60 year old man with diarrhoea is transferred from another hospital for urgent femoral-distal bypass surgery & arrives with a heparin infusion in situ. His APTT is 2.4. Most likely analgesic NOT to be used in the post-operative period: Diclofenac Paracetamol Tramadol Codydramol Morphine Epidural bupivacaine fentanyl

A

Epidural bupivacaine fentanyl Epidurals are relatively contraindicated in anticoagulated patients. Insertion of the epidural needle may lead traumatic bleeding into the epidural space and with clotting abnormalities, the development of a haematoma which can lead to spinal cord compression. Coagulopathy, raised ICP and infection at the injection site are absolute contraindications. Relative contraindications include anticoagulated patients and those with anatomical abnormalities of the vertebral column. NSAIDs do not increase the risk of epidural haematoma

75
Q

A 62 year old man who requires a knee replacement gives a history of allergy to dihydrocodeine. Most likely analgesic NOT to be used in the post-operative period: Diclofenac Paracetamol Tramadol Codydramol Morphine Epidural bupivacaine fentanyl

A

Codydramol Co-dydramol is a combination of dihydrocodeine and paracetamol and the patient is known to be allergic to dihydrocodeine.

76
Q

A 65 year old man with a history of peptic ulceration requires an aortic aneurysm repair electively. Most likely analgesic NOT to be used in the post-operative period: Diclofenac Paracetamol Tramadol Codydramol Morphine Epidural bupivacaine fentanyl

A

Diclofenac NSAIDs inhibit COX which has the effect of reducing PGE2 levels. PGE2 plays a role in gastric cytoprotection by downregulating HCl production and increasing mucus and the production of bicarbonate. This leads to gastric irritation and ulceration. A PPI can be prescribed alongside NSAIDs or misoprostol can be used, which is a stable PGE1 analogue which mimics local PG to maintain the gastroduodenal mucosal barrier.

77
Q

A 52 year old woman, with recurrent episodes of severe anxiety, when her family note that she becomes very pale. BP is up to 220/124mmHg during an attack, 150/90mmHg otherwise. Her pulse rate does not increase significantly during attacks. Most likely diagnosis A.Cushing’s syndrome B.‘Essential’ hypertension C.Coarctation of the aorta D.Metabolic syndrome (Insulin resistance/Syndrome X) E.Renovascular disease F.Phaeochromocytoma G.Primary hyperaldosteronism (Conn’s syndrome) H.Isolated systolic hypertension

A

Phaechromocytoma Phaeochromocytomas presents with paroxysmal episodes of palpitations, anxiety, excessive sweating, pallor and hypertension. It can be inherited in MEN2, von Hippel-Lindau syndrome and NF1. Diagnosis is based on raised urinary and serum catecholamines, metanephrines and normetanephrines. 24 hour urinary VMA will be elevated. CT is used to localise the tumour. Treatment includes medical with the use of phenoxybenzamine, phentolamine and surgical options. Surgical excision is carried out under alpha and beta blockade to protect against the release of catecholamines into circulation when the tumour is being manipulated. The 10% rule is often quoted: 10% are bilateral, 10% malignant, 10% extraadrenal and 10% hereditary.

78
Q

A 25 year old patient presents with 1 year history of painful scrotal swelling. On examination there is a hard smooth swelling of the right testis. It did not transilluminate. There was no cough impulse. Most likely diagnosis A.Retractile testis B.Epididymo-orchitis C.Trauma D.Hydrocele E.Strangulated indirect inguinal hernia F.Varicocele G.Metastasis H.Cryptoorchidism I.Teratoma J.Torsion of testis K.Mumps orchitis

A

Teratoma Testicular cancer commonly presents as a hard and painless lump on one testis although the lump can be painful and 10% present with acute pain associated with haemorrhage or infection. Key risk factors include cryptorchidism and FH. White men have the highest incidence. The principal investigation is an ultrasound of the testis and testicular examination is vital in detecting this condition early on. Beta-hCG is raised in seminomas and teratomas however only AFP is raised in teratomas. Placental ALP can be raised in advanced disease. It is diagnostic if AFP, beta hCG and LDH are elevated. Teratomas are more common in the 20-30 age group whereas seminomas are more common after 30. Radical orchidectomy and histology is the initial treatment in most cases.

79
Q

A 24 year old political refugee presents with a tender swollen right testicle of 3 days duration. Examination reveals tender mandibular swelling. Temperature 380C. Swollen tender testicle which does not transilluminate Most likely diagnosis A.Retractile testis B.Epididymo-orchitis C.Trauma D.Hydrocele E.Strangulated indirect inguinal hernia F.Varicocele G.Metastasis H.Cryptoorchidism I.Teratoma J.Torsion of testis K.Mumps orchitis

A

Mumps orchitis The tender mandibular swelling indicates mumps parotitis. Epidydimo-orchitis is the most common extra-salivary complication of mumps in males and presents as pain and swelling in one or both testicles. Those unvaccinated and international travellers are at increased risk. The mumps vaccine is incoporated as part of MMR. Mumps is caused by an RNA paramyxovirus and spread by respiratory droplets. If the diagnosis needs to be confirmed, then serological or saliva testing is possible looking for IgM. Treatment is supportive with symptom relief mainly with analgesia and antipyretics.

80
Q

A 52 year old company director presents with fever. He has recently returned from a conference in SE Asia. Examination – Temp 37.50C. Few tender inguinal lymph node. Tenderness of spermatic cord & testis. Most likely diagnosis A.Retractile testis B.Epididymo-orchitis C.Trauma D.Hydrocele E.Strangulated indirect inguinal hernia F.Varicocele G.Metastasis H.Cryptoorchidism I.Teratoma J.Torsion of testis K.Mumps orchitis

A

Epididymo-orchitis Generally speaking, younger males are more likely to have an STI whereas older men are more likely to have an infection with enteric organisms such as ESBL. This patient is however likely to have an STI due to his foreign exploits. Risk factors include unprotected sexual intercourse. He is febrile with tenderness and has reactive lymphadenopathy which points towards epididymo-orchitis. A urethral swab should be sent for Gram stain and culture of secretions. A urine dipstick is also necessary combined with urine MC+S. Treatment involves bed rest, scrotal elevation, analgesia and antibiotics if indicated. As this is likely to be sexually transmitted, a combination of ceftriaxone and doxycycline is indicated to cover for Chlamydia trachomatis and Neisseria gonorrhoeae.

81
Q

64 yr old woman complaining of severe back pain for some weeks. For the last few days she has been very constipated and has been vomiting for 24 hours. She has been a smoker for many years and has had 3 courses of antibiotics for chest infections over the last 3 months. Most likely diagnosis A.Hypocalcaemia B.Hypothyroidism C.Hypernatraemia D.Hypercapnia E.Metabolic acidosis F.Hypokalaemia G.Hypoglycaemia H.Hypercalcaemia I.Hyperkalaemia J.Hyponatraemia K.Hypermagnesaemia L.Addison’s disease

A

Hypercalcemia 90% of hypercalcaemia is caused by primary hyperparathyroidism or cancer. Cancer is the likely cause in this woman. Malignancy can cause hypercalcaemia either by direct bony involvement leading to osteolytic lesions or paraneoplastic syndromes involving PTHrp release. The tumour is typically very advanced if hypercalcaemia is a feature. Less common causes include vitamin D overdose, hyperthyroidism, immobilisation, Paget’s and milk-alkali syndrome. The classic bone disease in hyperparathyroidism is osteitis fibrosa cystica which causes pain. Symptoms of high calcium include confusion, constipation, polyuria, polydipsia, depression, kidney stones and lethargy. This can be remembered by ‘stones, bones, abdominal groans and psychiatric moans’. The serum PTH level is elevated in primary hyperparathyroidism whereas it may be very low in malignancy due to negative feedback.

82
Q

An 80 year old man presents with severely painful feet with mottled and purple toes with black areas. He tells you that he has also had constant severe back pain for a few days. Most appropriate investigation A.Cold provocation test B.Lumbar puncture C.Blood sugar level D.Venous duplex scan E.Arteriogram. F.FBC G.CT scan H.ANCA (anti-neutrophil cytoplasmic antibody) I.Blood cultures

A

CT scan This patient has a dissecting aortic aneurysm which can be diagnosed with a CT scan showing the presence of an intimal flap. The CT scan should include chest, abdomen and pelvis to visualise the extent of the aneurysm. Dissecting aneurysms are either type A, which involves the ascending aorta, or type B. Type A dissections require urgent surgery whereas type B can be managed medically if it is not complicated by end organ ischaemia. The aortic dissection has led to a cholesterol embolism. This can be diagnosed histopathologically with the finding of cholesterol crystals. The phenomenon where cholesterol is released from an atherosclerotic plaque is called ‘trash foot’. A highly technical medical term. This results in the mottled appearance of distal embolism associated with livedo reticularis. You can search the internet for some case reports of this phenomenon.

83
Q

A 25 year old university student presents with a high fever. He has petechial rash, black areas on his toes and fingers. His BP is 70/50. Most appropriate investigation A.Cold provocation test B.Lumbar puncture C.Blood sugar level D.Venous duplex scan E.Arteriogram. F.FBC G.CT scan H.ANCA (anti-neutrophil cytoplasmic antibody) I.Blood cultures

A

Blood cultures. This patient has sepsis. Sepsis is the presence of SIRS with a likely infectious cause. This patient’s profound arterial hypotension means he has severe sepsis (dysfunction of one or more organ systems). The patient being a young university student most likely has meningitis even though symptoms of headache, photophobia and neck stiffness are not mentioned. Hence, a LP would be performed but it is not the 1st test to order in a patient who presents with sepsis. It is important in the first instance to obtain a blood culture immediately, and preferably before antibiotics are started. If this is bacterial meningitis, you would expect to see a raised WCC on the LP with elevated protein, normal/reduced glucose and predominantly neutrophils in the white cell differential. Early blood cultures allows you to either broaden your empirical antibiotic spectrum or narrow it in those with sensitive organisms. It is worth noting that in sepsis, the patient may have a low temperature

84
Q

A 55 year old woman presents with painful joints, a purpuric rash on her arms and legs. Systems review reveals heamoptysis and ear pain. On examination you find black patches on her toes. Most appropriate investigation A.Cold provocation test B.Lumbar puncture C.Blood sugar level D.Venous duplex scan E.Arteriogram. F.FBC G.CT scan H.ANCA (anti-neutrophil cytoplasmic antibody) I.Blood cultures

A

ANCA This patient has Wegener’s granulomatosis, a systemic vasculitis affecting small and medium sized vessels. The classic triad includes upper and lower respiratory tract involvement and GN. Musculoskeletal manifestations such as arthralgia and signs of thromboembolism are commonly seen. A positive cANCA (antigen being proteinase 3) in the setting of the classic triad is sufficient to diagnose Wegener’s. Urinalysis and microscopy is also indicated to reveal renal involvement and a CT chest may reveal lung involvement, particularly in those who are asymptomatic for pulmonary involvement. This may show nodules or infiltrates.

85
Q

A 58 year old male has had increasing difficulty swallowing. He has lost 10kg in the past 2 months. Upper GI endoscopy reveals a nearly circumferential irregular & ulcerated mass in the mid oesophagus. Most likely diagnosis A.Hyperplasia B.Squamous cell carcinoma C.Myeloma D.Autonomous E.Sarcoma F.Lymphoma G.Carcinoma H.Adenocarcinoma

A

Squamous cell carcinoma Dysphagia coupled with weight loss and the irregular shaped mass points to malignancy. Dysphagia occurs when there is obstruction of more than 2/3 of the lumen and presence indicates locally advanced disease. There may additionally be odynophagia. Men are twice as likely to develop oesophageal cancer. GORD, Barrett’s oesophagus, FH, tobacco and alcohol are all risk factors. The two main types are squamous cell carcinoma and adenocarcinoma. Tumours in the upper 2/3 of the oesophagus are SCC whereas those that lie in the lower 1/3 are adenocarcinomas. The main test to order is an OGD with biopsy. Treatment is either surgical resection or with chemo or radiotherapy alongside endoscopic ablation with or without stenting and brachytherapy.

86
Q

A 75 year old man who presents with frank haematuria. He tells you that 3 years ago he had a similar episode & was diagnosed as having ‘warts in the bladder’. After treatment, he moved & was lost to follow-up but had been symptom free since then. Which investigation would confirm diagnosis? A.MSU: microscopy and cultures B.X-ray lumbar spine C.Prostate specific antigen blood test D.Biopsy of prostate E.Retrograde pyelogram F. Abdominal ultrasound G.ASO titre blood test H.Helical CT I.24 hour urine monitoring J.Cytoscopy

A

Cystoscopy “Warts in the bladder” is the way of some sneaky urologist avoiding telling this man he had cancer. He was treated, and like all cancer patients, was followed up. Unfortunately, this sounds like a recurrence. Gross haematuria is the primary symptom of bladder cancer. Cystoscopy and urinary cytology are key in diagnosis. Low grade tumours are papillary and easy to see on cystoscopy whereas high grade tumours and carcinoma in situ are often difficult to visualise. Resection provides diagnosis and primary treatment in one step.

87
Q

45 year old man wakes in the night with severe pain in his right flank radiating round to the front & into his groin. He can’t get comfortable, but on examination his abdomen is soft with no masses. His urine shows a trace of blood but no other abnormality. Which investigation would confirm diagnosis? A.MSU: microscopy and cultures B.X-ray lumbar spine C.Prostate specific antigen blood test D.Biopsy of prostate E.Retrograde pyelogram F. Abdominal ultrasound G.ASO titre blood test H.Helical CT I.24 hour urine monitoring J.Cytoscopy

A

Helical CT This patient has renal colic which classically presents with severe flank pain radiating to the groin. Microscopic haematuria is present in up to 90% of cases. Up to 85% of stones are visible on a plain KUB although urate stones are radiolucent. If the stone is radio-opaque, calcification will be seen within the urinary tract. In pregnancy, a renal USS is first line. The IV Pyelogram has now been replaced by the CT scan which is the new diagnostic standard. A non-contrast helical (or spiral) CT is preferred due to high sensitivity and specificity and acurately determines presence, site and size of stones. Stones are analysed after they are extracted or when they are expelled to check their composition. It is worth noting that in all females of child bearing age, a urine pregnancy test is necessary to exclude an ectopic pregnancy.

88
Q

A 55 year old known epileptic arrives in A&E having suffered a fit whilst shopping. She is “post ictal” on arrival in A&E & breathing in an obstructed manner with O2 saturation of 92% (on air). Most appropriate method to maintain patient oxygenation Tracheostomy None Facemask with oxygen Laryngeal mask airway (LMA) Oropharyngeal airway with oxygen Single-lumen cuffed ET tube Cricothyroidotomy Suction Laryngoscopy Uncuffed ET tube Double-lumen cuffed ET tube

A

Oropharyngeal airway with oxygen An oropharyngeal (or Guedel) airway is sized from the angle of the mandible to the level of the incisors. It is a non-definitive airway adjunct. The patient is breathing in an obstructed manner indicating some degree of partial upper airway obstruction. The Guedel will keep the airway patent and prevent the tongue obstructing the airway by depressing it. The Guedel can only be used if the patient has a reduced GCS as it can initiate a gag reflex. If the patient was not unconscious, then a nasopharyngeal airway can be used (usually inserted in the right nostril). Additionally, a Guedel is contraindicated if the patient has injuries to the face or a condition that prevents the mouth from opening. Airway manoeuvres can also be used such as a jaw thrust or head tilt chin lift in addition to maintain a patent airway. A jaw thrust can only be done if the patient is unconscious. Think about where your fingers are digging in.

89
Q

A 19 year old motorcyclist suffers a head injury after colliding with a lorry. On examination he is found to have a GCS of 4 & requires an urgent CT scan Most appropriate method to maintain patient oxygenation Tracheostomy None Facemask with oxygen Laryngeal mask airway (LMA) Oropharyngeal airway with oxygen Single-lumen cuffed ET tube Cricothyroidotomy Suction Laryngoscopy Uncuffed ET tube Double-lumen cuffed ET tube

A

Single lumen cuffed ET tube. This patient is unconscious and is about to undergo a CT scan. He needs a definitive airway. The cuffed end creates a seal the prevent the aspiration of stomach contents. Reduced consciousness is a major risk factor for aspiration due to an inadequate cough reflex and impaired closure of the glottis.

90
Q

A 78 year old patient is due to undergo surgery on her short saphenous veins in the prone position under general anaesthesia. She is obese and has known GORD. Most appropriate method to maintain patient oxygenation Tracheostomy None Facemask with oxygen Laryngeal mask airway (LMA) Oropharyngeal airway with oxygen Single-lumen cuffed ET tube Cricothyroidotomy Suction Laryngoscopy Uncuffed ET tube Double-lumen cuffed ET tube

A

Single lumen cuffed ET tube This patient is elderly, having surgery in a recumbent position and has known aspiration risk factors of obesity and GORD. She will need a definitive airway with a cuffed ET tube to create a seal to prevent aspiration of stomach contents. The ease of intubation can be determined by an airway assessment including a Mallampatti score (I to IV). A double lumen ET tube is designed for some intrathoracic operations. An uncuffed ET tube is preferred in children as the trachea is not as strong. If it is an emergency setting and ET intubation fails, cricothyroidotomy can be performed which allows approximately 30 minutes of ventilation. Cricothyroidotomy will only work with at least a partially patent larynx.

91
Q

A 30 year old lady, who is breast feeding, has developed an extremely painful, hard, red 4 cm lump at the edge of the nipple. Most likely diagnosis A.Radial scar B.Pagets disease C.Duct ectasia D.Fibrocystic breast E.Benign cyst F.Cyclical breast pain G.Fat necrosis H.Breast abcess I.Breast carcinoma J.Fibroadenoma

A

Breast abscess Breast abscess presents with mastalgia and fever. Breast infection typically affects women who are lactating and the most commonly implicated pathogen is staphylococcus aureus. The painful, hard and red lump indicates the development of an abscess. Antibiotic therapy is indicated with surgical intervention such as aspiration and drainage with possible duct excision. Prompt management of mastitis when it presents will usually lead to a good timely resolution and prevent the development of complications such as an abscess. An USS can help to identify the underlying abscess which usually forms a hypoechoic lesion. Needle aspiration can be used both therapeutically and diagnostically and can be guided by ultrasound.

92
Q

A 65 year old lady has noticed a 3cm lump behind the right nipple. It is hard, non-tender, mobile and has an irregular edge. Most likely diagnosis A.Radial scar B.Pagets disease C.Duct ectasia D.Fibrocystic breast E.Benign cyst F.Cyclical breast pain G.Fat necrosis H.Breast abcess I.Breast carcinoma J.Fibroadenoma

A

Breast carcinoma This lady’s advanced age and the fact the lump is hard, irregular and non-tender all point towards malignancy. The mass is typically fixed but can be mobile in early stages. There may be associated changes such as nipple inversion, peau d’orange, lymphadenopathy and skin retraction. Skin changes are always associated with locally advanced cancer. Triple assessment is indicated with a mammogram and biopsy. Risk factors include FH, prolonged oestrogen exposure and high levels of alcohol consumption.

93
Q

A 60 year old man has suprapubic pain on standing and difficulty emptying his bladder with blood at the end of the stream. Most likely diagnosis A.Bacterial cystitis B.Gram negative septicaemia C.Advanced prostate cancer D.Localised prostate cancer E.Hydronephrosis F.Benign prostatic enlargement G.Acute prostatitis H.Bladder calculus

A

Bladder calculus There is pain on standing and difficulty urinating because the calculus falls and blocks the bladder outflow. Blood comes at the end of the stream from urine that has passed the damaged area of urothelium. Patients tend to present with suprapubic pain, haematuria and obstructive symptoms. Examination may reveal suprapubic tenderness. The first test to order is urinalysis. A non-contrast CT abdomen is also indicated to look for the stone.

94
Q

A 65 year old man presents with a large painless bladder and overflow incontinence at night and a raised creatinine level. Most likely diagnosis A.Bacterial cystitis B.Gram negative septicaemia C.Advanced prostate cancer D.Localised prostate cancer E.Hydronephrosis F.Benign prostatic enlargement G.Acute prostatitis H.Bladder calculus

A

Hydronephrosis This patient has BPH which has caused hydronephrosis. This is an example of bilateral obstructive uropathy. Acute presentations are often painful whereas chronic presentations are more insidious in onset. Blockage of urinary flow by the enlarged prostate has led to urinary retention and overflow incontinence. Initial treatment aims to relieve the pressure on the kidneys. This involves catheterisation as the first line treatment. The patient should be started on alpha blockers at the time of catheterisation.

95
Q

An 18 year old Caucasian shop assistant presents with fever & a sore throat. She is found to have enlarged but soft cervical lymph nodes & a soft spleen palpable 3cm below the costal margin. Blood film shows atypical lymphocytes. Most likely diagnosis A.Sarcoidosis B.Bacterial endocarditis C.Malaria D.CML E.Metastatic carcinoma F.IDA G.Idiopathic myelofibrosis H.Hodgkin’s disease I.Portal hypertension J.Gaucher’s disease K.Polycythaemia L.Infectious mononucleosis

A

Infectious mononucleosis This is caused by EBV and characterised by fever, pharyngitis and lymphadenopathy with atypical lymphocytosis. Positive heterophile antibody test and serological testing for EBV antibodies are diagnostic. Splenomegaly is common and enlargement occurs in the first week, lasting 3-4 weeks. It is worth remembering that splenomegaly is always an abnormal examination finding. IM is commonly named the ‘kissing’ disease as EBV is most commonly transmitted by saliva. Penetrative sex and general promiscuity in young women also increases the risk.

96
Q

A 65 year old man presents with angina & claudication. He is found to have a firm spleen extending 20cm below the costal margin. His Hb is 7.5g/dl & his blood film is leuco-erythroblastic Most likely diagnosis A.Sarcoidosis B.Bacterial endocarditis C.Malaria D.CML E.Metastatic carcinoma F.IDA G.Idiopathic myelofibrosis H.Hodgkin’s disease I.Portal hypertension J.Gaucher’s disease K.Polycythaemia L.Infectious mononucleosis

A

Idiopathic myelofibrosis This is a case of myelofibrosis. Leucoerythroblastosis and splenomegaly are common findings. Strong risk factors include exposure to radiation and industrial solvents. BM biopsy is essential for diagnosis. Extramedullary haematopoiesis leads to dacrocytes in the peripheral blood smear. Those without symptoms can be managed with folate and pyridoxine supplements. Otherwise options such as a BM transplant and hydroxycarbamide can be considered.

97
Q

Beta-blockers e.g atenolol Frequent side effect? A.Indigestion and GI bleed B.Dry cough C.Cold toes and fingers D.Gynaecomastia E.Goitre F.Swelling of feet and ankles G.Nausea and vomiting H.Peripheral neuropathy I.Tremor J.Constipation and dry mouth

A

Cold fingers and toes Beta blockers can cause bronchoconstriction (asthmatics), heart failure in those with heart disease who rely on a degree of sympathetic drive to the heart to maintain CO, hypoglycaemia (diabetics can lose warning signs), fatigue (reduced CO and muscle perfusion via beta 2 receptions) and cold extremeties (beta receptor mediated vasodilation of cutaneous vessels). Additionally there may be bad dreams which is more pronounced in lipophilic beta blockers such as propranolol.

98
Q

Erythromycin Frequent side effect A.Indigestion and GI bleed B.Dry cough C.Cold toes and fingers D.Gynaecomastia E.Goitre F.Swelling of feet and ankles G.Nausea and vomiting H.Peripheral neuropathy I.Tremor J.Constipation and dry mouth

A

Nausea and vomiting Erythromycin is a motilin agonist which causes N&V. It is a macrolide antibiotic and tends not be first line unless the patient has a penicillin allergy. It has a slightly wider spectrum of activity than penicillin. The macrolides can also cause reversible deafness although the most important group to note in drug induced deafness are the aminoglycosides such as gentamicin.

99
Q

A 78 year old woman attends complaining of recent onset of tiredness. She is pale, has hepatosplenomegaly and generalised painless lymphadenopathy in the neck, axillae and groin. Coombs’ (DAT) test is positive. Most likely diagnosis A.Portal vein thrombosis B.Systemic amyloidosis C.Malaria D.Severe emphysema E.Cirrhosis with hepatoma F.Haemachromatosis G.Toxoplasmosis H.Polycythaemia rubra vera I.CML J.RA K.Malignant melanoma L.CLL M.Right heart failure

A

CLL This elderly woman has CLL. CLL presents in older adults (generally >60) and is often asymptomatic. Smear cells can be seen in peripheral blood smear and it is associated with a warm type AIHA accounting for her pallor and fatigue (hence the Coombs’ test is positive). Painless lymphadenopathy may be present and splenomegaly is a common finding. A WCC with differential is required to make a diagnosis. An absolute lymphocytosis will be seen. CML is not associated with an AIHA and tends to present at a younger age.

100
Q

A 78 year old woman attends complaining of widespread itching. Examination reveals hepatosplenomegaly. The patient appears plethoric with no lymphadenopathy Most likely diagnosis A.Portal vein thrombosis B.Systemic amyloidosis C.Malaria D.Severe emphysema E.Cirrhosis with hepatoma F.Haemachromatosis G.Toxoplasmosis H.Polycythaemia rubra vera I.CML J.RA K.Malignant melanoma L.CLL M.Right heart failure

A

Polycythemia rubra vera PRV is a disease of middle and older age and is strongly associated with the JAK2V617 mutation. Pruritis is a common feature and is often severe and evoked by contact with water. Facial redness and fullness is commonly observed and splenomegaly is a common finding. It is a myeloproliferative disorder with raised Hct, Hb and RBC count. Blood hence becomes very viscous. There is a clear link between Budd-Chiari syndrome and subsequent PRV. Treatment is with venesection. Around 30% will go on to develop myelofibrosis.

101
Q

A 52 year old patient, 24hours post laparotomy for perforated duodenal ulcer is found to have a pulse of 120bpm and BP 95/40mmHg. He is apyrexial and is WCC is 1.5units Most likely cause of hypotension Hypovolaemia Pulmonary oedema MI Cardiac dysrhythmia Drugs Sepsis ARDS Aortic stenosis DKA SIADH

A

Hypovolemia This patient is apyrexial and there are no signs of SIRS or sepsis. The raised HR and low BP is likely due to hypovolaemia. This patient needs fluids.

102
Q

An 85 year old man presented with perforated diverticular disease in the sigmoid colon. After resuscitation he had an emergency procedure. What is the most likely procedure? A.Parastomal hernia B.Hartmann’s procedure: End colostomy C.Defunctioning loop ileostomy D.Prolapsed colostomy E.Ileal conduit F.Loop colostomy G.End ileostomy H.Subtotal colectomy and end ileostomy I.Loop ileostomy

A

Hartmann’s procedure: End colostomy A Hartmann’s procedure is usually done as an emergency. The non-viable part of bowel (sigmoid colon) is excised and the proximal part of bowel is brought out as a colostomy (end colostomy because there is no distal part of bowel connected). The rectal stump is sewn up, with a view to re-anastomosis at a later date under more optimal conditions.

103
Q

A 65 year old man undergoes elective excision of a rectal carcinoma and primary anastomosis. He is left with a stoma. What is the most likely procedure? A.Parastomal hernia B.Hartmann’s procedure: End colostomy C.Defunctioning loop ileostomy D.Prolapsed colostomy E.Ileal conduit F.Loop colostomy G.End ileostomy H.Subtotal colectomy and end ileostomy I.Loop ileostomy

A

Loop ileostomy This man has undergone an anterior resection of the rectum with a primary anastomosis. A loop ileostomy as this will allow distal bowel to be rested by diverting faecal flow so that faecal matter does not continue to aggravate the new anastomosis that has been created. Once the anastomosis is well established, the loop ileostomy can be easily reversed.

104
Q

A 25 year old man with a history of ulcerative colitis, deteriorates and requires and emergency operation to treat the disease. What is the most likely procedure? A.Parastomal hernia B.Hartmann’s procedure: End colostomy C.Defunctioning loop ileostomy D.Prolapsed colostomy E.Ileal conduit F.Loop colostomy G.End ileostomy H.Subtotal colectomy and end ileostomy I.Loop ileostomy

A

Subtotal colectomy and end ileostomy Subtotal colectectomy can be done in emergency extreme cases of UC. The ileum needs to be brought out as a stoma as the only parts of the colon that remain are the rectum and anus, and anastomosing this with the terminal ileum can cause ileal pouchitis and incontience. As you are aware, ulcerative colitis patients who have panproctocolectomies do not have this option and end up having end-ileostomies.

105
Q

A 72 year old woman has a disseminated malignancy of the colon. She is suffering from abdominal distention and increasing constipation. She has a procedure before going on to palliative care. What is the most likely procedure? A.Parastomal hernia B.Hartmann’s procedure: End colostomy C.Defunctioning loop ileostomy D.Prolapsed colostomy E.Ileal conduit F.Loop colostomy G.End ileostomy H.Subtotal colectomy and end ileostomy I.Loop ileostomy

A

Loop colostomy A loop colostomy has the same aim as a loop ileostomy. The faecal stream is diverted to rest distal bowel. Here, the primary objective is to somehow offload all the faeces that have made it past the ileum and are piling up in the colon, so that the colon does not have to keep working to push faeces against an obstruction (which will create colicky pain). Remember also that your palliative patient will lose more water if you do a loop ileostomy.

106
Q

Low calcium, Low phosphate, High PTH, High ALP Most likely diagnosis? A.Osteomalacia B.Paget’s disease C.Sarcoidosis D.Osteoporosis E.Hypercalcaemia of malignancy F.Immobility G.Hypoparathyroidism H.Primary hyperparathyroidism I.Myeloma

A

Osteomalacia

107
Q

High calcium, Low phosphate, Normal PTH, High ALP Most likely diagnosis A.Osteomalacia B.Paget’s disease C.Sarcoidosis D.Osteoporosis E.Hypercalcaemia of malignancy F.Immobility G.Hypoparathyroidism H.Primary hyperparathyroidism I.Myeloma

A

Primary hyperparathyroidism It is worth noting that in primary hyperparathyroidism, serum PTH may be normal, which is in fact abnormal in light of high calcium which should suppress PTH levels.

108
Q

Normal calcium, Normal phosphate, High PTH, Very high ALP Most likely diagnosis? A.Osteomalacia B.Paget’s disease C.Sarcoidosis D.Osteoporosis E.Hypercalcaemia of malignancy F.Immobility G.Hypoparathyroidism H.Primary hyperparathyroidism I.Myeloma

A

Paget’s disease

109
Q

High calcium, Normal phosphate, Slightly low PTH , High ALP Most likely diagnosis? A.Osteomalacia B.Paget’s disease C.Sarcoidosis D.Osteoporosis E.Hypercalcaemia of malignancy F.Immobility G.Hypoparathyroidism H.Primary hyperparathyroidism I.Myeloma

A

Hypercalcemia of malignancy

110
Q

Normal calcium, Normal phosphate, Normal PTH, Normal ALP Most likely diagnosis A.Osteomalacia B.Paget’s disease C.Sarcoidosis D.Osteoporosis E.Hypercalcaemia of malignancy F.Immobility G.Hypoparathyroidism H.Primary hyperparathyroidism I.Myeloma

A

Osteoporosis

111
Q

A 66 year old alcoholic man has a firm irregular liver, testicular atrophy, splenomegaly, a normocytic normochromic anaemia & thrombocytopaenia. Most likely diagnosis? A.Glucose-6-phosphate dehydrogenase deficiency B.Haemolytic uraemic syndrome C.Hereditary spherocytosis D.Thalassaemia major E.Sickle cell trait F.Sickle cell anaemia G.Dietary folic acid deficiency H.Blood loss I.Iron deficiency J.Anaemia of chronic disease K.Pernicious anaemia L.Coeliac disease

A

ACD Whilst alcoholism may prompt you to consider alternative causes for this man’s anaemia, a positive alcohol history may be present in those with ACD and the normocytic normochromic anemia gives this away. ACD is caused by inflammation, which can result from various disease processes. The release of pro-inflammatory cytokines leads to a cascade producing anaemia due to decreased RBC production and shortened survival. The anaemia of ACD can also be microcytic.

112
Q

A 51 year old manic depressive man, who has taken an unknown number of extra lithium tablets & has a plasma lithium level of 8 mmol/l. There are marked neurological features. Most appropriate management A.Hyperbaric oxygen B.Sodium calcium edetate C.Intravenous atropine D.None E.Haemodialysis F.IV N-acetylcysteine G.Intravenous naloxone H.Oral desferrioxamine I.Alkaline diuresis J.Ethanol K.Dicobalt edetate

A

Haemodialysis Haemodialysis is the treatment of choice for severe lithium poisoning. Note that activated charcoal does not adsorb lithium.

113
Q

A 14 year old girl who has taken at least 30 aspirin tablets (300mg each). Most appropriate management A.Hyperbaric oxygen B.Sodium calcium edetate C.Intravenous atropine D.None E.Haemodialysis F.IV N-acetylcysteine G.Intravenous naloxone H.Oral desferrioxamine I.Alkaline diuresis J.Ethanol K.Dicobalt edetate

A

Alkaline diuresis This patient has ingested at least 9000mg of aspirin. GIT decontamination should be considered as an adjunct on arrival to A&E and activated charcoal can be given. The mainstay of treatment is alkaline diuresis induced by an infusion of sodium bicarbonate. In cases of severe poisoning, it is still started as a bridge to haemodialysis.

114
Q

A 38 year old agricultural worker who has been accidentally exposed to an organophosphorous insecticide & has symptoms of cholinergic overactivity. Most appropriate management A.Hyperbaric oxygen B.Sodium calcium edetate C.Intravenous atropine D.None E.Haemodialysis F.IV N-acetylcysteine G.Intravenous naloxone H.Oral desferrioxamine I.Alkaline diuresis J.Ethanol K.Dicobalt edetate

A

IV atropine Organophosphate poisoning can occur due to occupational or accidental exposure, or if you were being attacked by Saddam Hussein’s regime. Treatment is with resuscitation and supportive care, removal of contaminated clothing, washing the skin and starting full atropinisation IV. Over treament is very much preferred to under treatment and the skin should feel dry when the patient has received adequate atropine. Pupils will also be dilated. Pralidoxime is often given as an adjunct in very severe cases but evidence does not support its routine use, unless you’ve just been attacked by a nerve agent by terrorists, which is unlikely in this agricultural worker.

115
Q

A 70 year old man after a laparotomy has been given 4L of dextrose/saline intravenously in 24 hours. He is tachypnoeic, with BP 130/90, pulse 120/min & has bilateral basal crepitations. Most appropriate initial management Intravenous saline Administer diuretics Blood transfusion Measure blood gases Intravenous sodium bicarbonate Intravenous dextrose Measure urea & electrolytes Intravenous plasma Intravenous colloid

A

Administer diuretics This patient has developed pulmonary oedema. This patient should be positioned upright and given diuretics (frusemide).

116
Q

A 20 year old man has been involved in a road traffic accident & the ambulance has just arrived. He has severe left upper abdominal tenderness, blood pressure 80/60 & pulse 140/min. Most appropriate initial management Intravenous saline Administer diuretics Blood transfusion Measure blood gases Intravenous sodium bicarbonate Intravenous dextrose Measure urea & electrolytes Intravenous plasma Intravenous colloid

A

IV saline Fluid replacement therapy aims to maintain physiological parameters. Systolic BP >90 is required for adequate organ perfusion. IV 0.9% saline is the first line treatment for volume depletion in almost all situations. It is worth noting that normal saline in large amounts carries a risk of inducing a metabolic acidosis due to the high chloride content. 5% dextrose is equivalent to water when given, and is not approriate for volume resuscitation since it will distribute throughout the total body water. Only 1/12 will remain in the intravascular space. 5% dextrose may be used, however, in resuscitation or replacement in diabetics on an insulin drip to prevent hypoglycaemia. The insulin and dextrose infusion should go in the same cannula so there is no risk of giving unopposed insulin. For example, if the arm you are giving the dextrose infusion were to clog up, or more likely, if the patient bends their arm if the cannula is sited in the antecubital fossa.

117
Q

Which 1 of the following about ECG is correct? A) ST depression and tall R waves in leads V1 + V2 is consistent with a diagnosis of a posterior myocardial infarction. B) The corrected QT interval = QT/RR interval C) Obesity causes low-voltage QRS complexes D) 2 mm ST elevation in leads II, III, aVF, V4 + V5 is consistent with an inferior MI E) S1Q3T3 is a common finding in patients diagnosed with a PE

A

A) B - corrected QT = QT/√RR interval C - Low voltage QRS complexes are caused by things such as hypothyroidism, COPD and increased haematocrit, but not obesity. D - This suggests an inferior-lateral MI E - This can occur with a PE but is not common.

118
Q

Which is true? A) The long saphenous vein runs from the sapheno-femoral junction to the medial malleolus. B) The sapheno-femoral junction is 1cm lateral and 1cm inferior to the pubic tubercle. C) The profunda femurs can be harvested and used as a replacement artery for CABG D) A saphena varix classically transmits a cough impulse and disappears when the patient is asked to stand up. E) The Trendelenburg test is useful for assessing the competence of the SFJ

A

E A - Long saphenous being runs distal to proximal. B - SFJ is 4cm inferior and lateral to the pubic tubercle C - Long saphenous vein can be used to for CABG D - Saphena varix classically transmits a cough impulse and disappears when the patient is asked to lie down.

119
Q

You are undertaking a placement in a family planning clinic during your GP rotation. Which 1 is true? A) Condoms have a success rate of only 90%, even when used to their maximal effectiveness. B) Failure rates of male surgical sterilisation are worse than female surgical sterilisation. C) The combined OCP can be used to treat acne vulgaris. D)Breast cancer is not a contraindication for the used of the combined OCP E) Mrs Smith, a 35 year old who smokes 20 cigarettes a day, can be safely started on the combined OCP.

A

C) A - Condom success rate approaching 98-99% B - The rates of failure of female tubal ligation are considerably worse than the failure rate of vasectomy in men. D + E - Combined OCP contraindications are breast cancer, >35 whilst smoking >15 cigarettes a day.

120
Q

A 56 year old lady presents with progressive tiredness, malaise and general apathy. She also states that her motions have been much looser recent and it is painful for her to eat. O/E she has difficulty appreciating sensation distally and FBC shows an Hb of 9g/dL and an MCV of 125fl. Underlying diagnosis? a) hypothyroidism b) hyperthyroidism c) pernicious anaemia d) Addison’s disease e) Stomach carcinoma

A

C) The patient has megaloblastic anaemia due to B12 deficiency and subsequent neuropathy. These features combined with the sore mouth and looser stool is suggestive of pernicious anaemia.

121
Q

You are an F1 reviewing patients in the Derm clinic. You are reviewing Mrs Jones, a 32 year old patient originally from Australia who has had long-standing pompholyx affecting her fingers. During this attendance she says that her husband has noticed a new mole on her back which has been bleeding on contact. Which is true about melanoma? A) Nodular melanoma is a highly aggressive form of the disease B) Superficial spreading melanoma, despite its name, exhibits vertical spread before lateral spread. C) Clark’s thickness represents thickness of the tumour depth D) Patients with metastatic disease have a 5 year survival of >90% E) Colour variegation should not raise suspicion as it is present in moles as well as melanoma.

A

A) Melanoma is a malignancy of melanocytes, with a major causative factor being exposure to UV radiation during childhood and adolescence. Nodular melanoma is the most aggressive form of the disease and invades vertically early and grows rapidly. Superficial spreading melanoma usually exhibits lateral spread followed by vertical spread. Breslow thickness is the depth of the tumour invasion and is measured from the surface to the area of deepest penetration of the skin and is a more reliable prognostic marker than Clark’s level, which describes the layer the melanoma cells have infiltrated to. Metastatic disease has a 5 year survival of 5-15% ABCDE criteria for worrying signs Asymmetry Border irregularity Colour variegation Diameter >6mm Elevation or enlarging mole.

122
Q

A worried parent brings her 15 year old daughter to see you in GP. The mother is concerned that her daughter is becoming increasingly thin and gaunt and has lost a considerable amount of weight after going on a crash diet. The patient feels that she is overweight and no amounts if reasoning by her parents change her beliefs. She has restricted the types and amounts of food she eats and exercises excessively. Her weight is 80% predicted for her age, height and sex. Which is true of the underlying diagnosis? a) 50:1 female:male affected b) Amenorrhoea often precedes weight loss c) Commoner in lower socio-economic groups d) Marked increase in sexual activity e) In adults a BMI of

A

B) This is anorexia nervosa A - 1:10 C - This affects higher socio-economic groups more. D - ? E - BMI

123
Q

A 40 year old woman has been diagnosed with grand map epilepsy was started on treatment after her second episode. You are an F2 in the Neurology clinic and she meets you for the first time. She complains that her old shoes no longer fit and that her hair seems to have thinned over the pst few months. She also complains that she is feeling more tired and has to take afternoon naps to keep going. O/E you note that she is slightly overweight and on direct questioning she admits her weight sees to have increased, although denies a change of diet. She also appears to have a mild tremor. Which 1 is most likely in this patient? A) Has underlying pituitary disorder B) Features should raise suspicions of underlying depression C) Features consistent with carbamazepine D) Features consistent with sodium valproate treatment E) Features consistent with lamotrigine treatment.

A

D) Sodium valproate side effects: WHAAT Weight again Hyperammonaemia (including N+V & malaise) Ankle swelling Ataxia + tremor Thinning hair

124
Q

A 32 year old housewife is referred to the GI clinic with a 2 year history of GI complaints. Which of the following is most useful in her history in establishing an organic diagnosis? A) Tenesmus B) Mucus per rectum C) Dyspareunia D) Abdo bloating E) Weight loss

A

E IBS is a complex disease that has a range of symptoms, all of which are listed in the question, apart from weight loss, which is most suggestive of an underlying organic cause.

125
Q

Which disease has the highest incidence of positive rheumatoid factor? A) Sjogren syndrome B) Acute RA C) Systemic sclerosis D) SLE E) PBC

A

A The presence of RF, an IgM (rarely IgG) Ab whose Fab portion binds the Fc portion of Ig occurs in a range of AI disease. The incidence in RA increases with increasing disease duration, being present in approx 70-80% on individuals. The presence of RF is much higher in Felty syndrome (RA + splenomegaly + neutropenia) and in Sjogren syndrome.

126
Q

A patient presents acutely with a red eye and is in a significant amount of pain, with a marked degree of photophobia. The patient also blurred vision and floaters. Which is the most likely in the above patient? A)Viral conjunctivitis B) Bacterial conjunctivitis C) Open angle glaucoma D) Closed angle glaucoma E) Uveitis

A

E A red eye is a medical emergency as the diseases responsible are able to lead to long term damage and blindness. The main 3 to consider are closed angle glaucoma, acute conjunctivitis and uveitis. In this case the significant pain and photophobia suggest uveitis, which requires urgent ophthalmological assessment and steroids.

127
Q

A female patient attends A+E because of worsening SOB which is associated with worsening rhinitis and sinusitis. O/E she has a lesion of the facial nerve on the right. Blood tests showed a markedly raised eosinophil count with raised ESR. She says that her GP sent her for a CXR in the past for similar symptoms and it was reported as abnormal. Most likely? A) Allergic asthma B) Polyarteritis nodosa C) infective exacerbation of asthma D) Churg-strauss syndrome E) SLE

A

D Churg-Strauss is a rare vasculitis affecting small and medium sized vessels especially in the lungs. Diagnosis is based on the presence of 4 or more of the following criteria 1) Asthma 2) Eosinophilia 3) Sinusitis 4) Lung infiltrates 5) Histological diagnosis of vasculitis with eosinophils 6) Mononeuritis multiplex or polyneuropathy Treatment is with immunosuppression, commonly high dose steroids, but occasionally cytotoxic drugs such as cyclophosphamide are needed.

128
Q

Which is most likely to hear wide fixed splitting of S2 on auscultation? A) RBBB B) Atrial septal defect C) VSD D) LBBB E) Patent ductus arteriosus

A

B RBBB + VSD cause wide but not classically fixed splitting. LBBB and PDA cause reverse splitting of S2

129
Q

Which is true? A) Loop diuretics exert their effect by acting on Na/K/HCO3 channels in the thick ascending limb of the loop of Henle B) Amiloride promotes K reabsorption in the collecting duct, accounting for its K sparing effects. C) Metolazone is a powerful thiazide diuretic used in severe and resistant HF D) Flecainide is the agent of choice in patients with AF and a background of coronary heart disease E) Patients on amiodarone should have regular check ups on their thyroid and renal function.

A

C Metolazone is an extremely potent diuretic commonly used synergistically with loop diuretics in resistant fluid overload, although great care has to be taken with monitoring electrolytes and renal function. A - Na/K/2Cl channels in thick ascending limb B - Amiloride is a weak K sparing diuretic, commonly combined with thiazide diuretics and inhibit Na reabsorption in the collecting duct, this limiting K excretion. D - Flecainide is a class I anti-arrhythmic agent that increases mortality in patients with significant coronary heart disease, LV dysfunction or structural heart disease. E - Amiodarone side effects require monitoring of TFTs + LFTs, although patients have to be counselled with regard to increased photosensitivity, lung fibrosis and corneal micro deposits.

130
Q

Which is true about delirium? A) Confusion tends to get better as the day progresses B) Sensory deprivation is a risk factor for development of delirium C) Visual hallucinations are suggestive of an alternative diagnosis D) Haloperidol is absolutely contraindicated in the management of delirium E) Vast majority of patients with delirium have underlying dementia

A

B Delerium is a commonly encountered problem requiring admission among the elderly population and is frequently encountered on the ward, in both medical and surgical patients. The core features include: 1) Fluctuating course 2) Onset over hours or days 3) Clouding of consciousness 4) Impaired cognition Confusion tends to get worse as the day progresses and leads to a reversal of the sleep cycle, leading to disturbances on the ward at night. Although sedatives should be avoided, occasionally it is in the patient’s best interest to calm their behaviour slightly to prevent injuries and allow them to be looked after properly. Visual hallucinations occur commonly but care has to be taken to ascertain that they do not represent alcohol withdrawal hallucinations. Patients with dementia are more likely to develop delirium and the 2 conditions often co-exist in hospitalised patients but the majority of patients who have delirium do not have underlying dementia.

131
Q

How many weeks post-operatively does abdominal wound dehiscence occur?

A

2

132
Q

A hernia where only part of the bowel wall is in the sac. What type of hernia is it? A.Obstructed hernia B.Sliding hernia C.Spigelian hernia D.Strangulated hernia E.Incarcerated hernia F.Incisional hernia G.Saphena varix H.Richter’s hernia I.Femoral artery aneurysm

A

Sliding hernia

133
Q

A hernia where the sac does not envelope all of the hernial contents What type of hernia is it? A.Obstructed hernia B.Sliding hernia C.Spigelian hernia D.Strangulated hernia E.Incarcerated hernia F.Incisional hernia G.Saphena varix H.Richter’s hernia I.Femoral artery aneurysm

A

Richter’s hernia

134
Q

Lymph nodes in the neck are described in relation to the major veins in the neck True or False

A

True

135
Q

Rodent ulcer and basal cell carcinoma are two different names for the same condition True or False

A

True

136
Q

A pilonidal abscess typically occurs in the umbilicus of hairy individuals True or False

A

False It occurs in the natal cleft

137
Q

Malignant melanoma can be difficult to distinguish from a bruise True or False

A

True. Both are characterised by blotchy discolouration of the skin.

138
Q

Thyroid goitre typically has a fine, diffuse irregularity throughout True or False

A

True

139
Q

A lump on the dorsum of the wrist that is fluctuant is typical of a…

A

Ganglion

140
Q

A rare lump not present at birth but appearing early in life, containing glary fluid and cholesterol crystals. A) Pleomorphic adenoma B) Chemodectoma C) Dermoid cyst D) Cystic hygroma E) Branchial cyst

A

Branchial cyst

141
Q

This lump presents in early infancy. It consists of many lymph-filled spaces and is brilliantly translucent. It increases in size upon coughing. A) Pleomorphic adenoma B) Chemodectoma C) Dermoid cyst D) Cystic hygroma E) Branchial cyst

A

Cystic hygroma

142
Q

A carotid body tumour occurring at the carotid bifurcation. A) Pleomorphic adenoma B) Chemodectoma C) Dermoid cyst D) Cystic hygroma E) Branchial cyst

A

Chemodectoma

143
Q

The commonest salivary tumour that is benign. A) Pleomorphic adenoma B) Chemodectoma C) Dermoid cyst D) Cystic hygroma E) Branchial cyst

A

Pleomorphic adenoma

144
Q

A 28-year-old man presents with a lump in the posterior triangle of his neck. He mentions that he has had difficulty finishing his meals and sometimes regurgitates his food at night. On palpation of the lump you hear a gurgling sound. Diagnosis A.Cervical rib B.Pharyngeal pouch C.Cystic hygroma D.Carotid body tumour E.Sternocleidomastoid tumour F.Dermoid cyst G.Torticollosis H.Goitre I.Thyroglossal cyst

A

Pharyngeal pouch A pharyngeal pouch develops from backwards protrusion of the pharyngeal mucosa through a weak area of the pharyngeal wall (known as Killian’s dehiscence). The pouch initially develops posteriorly, but then protrudes to one side, usually the left, displacing the oesophagus laterally. They occur most often in elderly men who present with a characteristic history of dysphagia, regurgitation of food and sensations of gurgling in the neck. Inhalation of regurgitated food may lead to fits of coughing and aspiration at night.

145
Q

A 32-year-old basketball player presents with pain and paraesthesia down the medial aspect of his left arm. He claims his symptoms worsen when he raises her arms for a slam dunk. On examination, there is a firm lump at the base of his neck on the left side. Diagnosis A.Cervical rib B.Pharyngeal pouch C.Cystic hygroma D.Carotid body tumour E.Sternocleidomastoid tumour F.Dermoid cyst G.Torticollosis H.Goitre I.Thyroglossal cyst

A

Cervical rib A cervical rib is a congenital overdevelopment of the transverse process of the seventh cervical verebrae. This ‘rib’ may interfere with the subclavian artery and the lower roots of the brachial plexus leading to a form of thoracic outlet syndrome. If the T1 root is affected (as in this patient), there is pain and paraesthesia along the medial aspect of the arm as well as wasting of the small muscles of the hand. Symptoms may worsen when the arms are elevated, such as during a slam dunk.

146
Q

A 3-month year old boy is brought into the clinic by his parents. He has a lump at the base of the neck, posterior to the left sternocleidomastoid muscle. On examination, it is compressible and transilluminates brilliantly. Most likely diagnosis A.Cervical rib B.Pharyngeal pouch C.Cystic hygroma D.Carotid body tumour E.Sternocleidomastoid tumour F.Dermoid cyst G.Torticollosis H.Goitre I.Thyroglossal cyst

A

Cystic hygroma A cystic hygroma is a congenital benign collection of lymphatic sacs. The cysts contain clear fluid and characteristically transilluminate brightly. It presents as a soft, fluctuant lump just beneath the skin and usually occurs in the posterior triangle of the neck. Cystic hygromas are large singular or multilocular fluid-filled cavities that arise from the jugular lymph sacs. Lymph sacs are embryonic lymph node precursors: there are also iliac and retroperitoneal lymph sacs. The jugular lymph sacs lie near the junction of the subclavian veins with the anterior cardinal veins (the future internal jugular veins). Lymphatic vessels join the sacs and pass along the head, neck and upper limbs from the jugular lymph sacs. Most cystic hygromas appear to be derived from abnormal transformation of the jugular lymph sacs. As you can see, therefore, the cystic hygroma could technically develop anywhere that the jugular lymph sacs spread to. Over 60% (according to GP notebook - different papers report different statistics) occur in the neck region, but they have also been reported on the chest wall and in the axilla and arm. Classically, cystic hygromas appear in the inferolateral part of the neck. The Oxford Handbook of Clinical Specialties puts cystic hygromas in the anterior triangle as opposed to the posterior (i.e. below digastric and in front of sternocleidomastoid).

147
Q

A 2-month-year-old boy is brought in to see you by his parents as they are worried about a lump in the right side of his neck. On examination you notice that the lump is hard and fixed to the underlying muscle and the baby’s head is tilted towards it. Diagnosis A.Sternocleidomastoid tumour B.Sialolithiasis C.Goitre D.Salivary duct carcinoma E.Dermoid cyst F.Lymphoma G.Thyroglossal cyst H.Branchial cyst

A

Sternocleidomastoid tumour The term ‘tumour’ is actually a misnomer as it is a fibrous mass in one of the sternocleidomastoid muscles. This condition is often associated with torticollis (tilting of the head) due to contraction of the sternocleidomastoid muscle itself. The etiology of this condition is uncertain, however it is thought to result from intrauterine or birth trauma leading to secondary pressure necrosis and fibrosis of the muscle.

148
Q

A 25-year-old man presents with a painful swelling which bulges from beneath the anterior border of his left sternomastoid muscle. On examination, it is soft and fluctuant, but tender. He says it has been present since childhood and only started causing him trouble following a respiratory infection. Diagnosis A.Sternocleidomastoid tumour B.Sialolithiasis C.Goitre D.Salivary duct carcinoma E.Dermoid cyst F.Lymphoma G.Thyroglossal cyst H.Branchial cyst

A

Branchial cyst A branchial cyst is a remnant of the second branchial cleft, which normally involutes during embryonic development. It commonly presents in young adults as a smooth, non-tender, fluctuant swelling in the anterior triangle anterior to the border of the sternocleidomastoid at the junction of its upper and middle thirds (this position is characteristic). It may become enlarged and inflamed with upper respiratory tract infections.

149
Q

A 60-year-old man complains of a lump in the groin. He says it enlarges if he coughs or laughs. On examination, he has an expansile mass just lateral to the femoral vein, which transmits a pulse. The mass is irreducible. Diagnosis A.Femoral hernia B.Femoral artery aneurysm C.Psoas abscess D.Direct inguinal hernia E.Indirect inguinal hernia F.Saphena varix G.False femoral artery aneurysm H.Inguinal lymphadenopathy

A

Femoral artery aneurysm. A swelling that is both pulsatile and expansile is an aneurysm. True aneurysms of the superficial femoral artery are rare, with most peripheral aneurysms occuring in the politeal artery. A femoral aneurysm may occasionally spontaneously rupture; whereas a popliteal aneurysm may thrombose and present as an acutely ischaemic leg.

150
Q

A 35-year-old Indian man presents with a swelling lateral to the femoral artery. On examination it is tender and fluctuant, but cannot be reduced. He also describes a recent history of weight loss and night sweats. Diagnosis A.Femoral hernia B.Femoral artery aneurysm C.Psoas abscess D.Direct inguinal hernia E.Indirect inguinal hernia F.Saphena varix G.False femoral artery aneurysm H.Inguinal lymphadenopathy

A

Psoas abscess This man’s ethnicity and clinical symptoms point towards a diagnosis of TB. A known complication of intraabdominal TB is abscess formation in the lumbar vertebrae (Pott’s disease). Such an abscess may track down the psoas sheath and into the groin, forming a psoas abscess. Psoas abscesses are described as ‘cold’, as they are painless and not warm to touch.

151
Q

A 63-year-old man with Von Recklinghausen’s disease presents with pain in his left thigh. On examination, there is a hard well-defined swelling lying lateral to the femoral artery. There is no cough impulse and it is irreducable. Pressure on the swelling reproduces the pain in the thigh. Diagnosis A.Caecal carcinoma BDirect inguinal hernia C.Crohn’s disease D.Femoral aneurysm E.Indirect inguinal hernia F.Strangulated femoral hernia G. Ectopic testis H.Femoral neuroma

A

Femoral neuroma. Femoral neuromas can arise due to previous trauma or as a feature of neurofibromatosis (Von Recklinghausen’s disease). The femoral nerve lies lateral to the femoral artery and compression of the neuroma results in pain along the nerve’s distribution.

152
Q

A 16-year-old boy presents with sudden onset severe pain in his right scrotum and vomiting. On examination, the right testis appears swollen and is hanging higher than the left. Diagnosis A.Torsion of the hyatid of Morgagni B.Epididymo-orchitis C.Testicular torsion D.Hydrocele E.Teratoma F.Varicocele G.Mumps orchitis H.Seminoma

A

Testicular torsion The presentation of sudden-onset pain and swelling in the testicle in a male of this age strongly suggests torsion. This is a surgical emergency and commonly occurs between the age of 10 and 15 years (rarely occuring over the age of 21). The incidence of torsion is higher for undescended testes. Pain may be referred to the abdomen as the testis retains its embryological nerve supply from the T10 sympathetic pathway. Irreversible adamage occurs within 6-12 hours, so urgent surgical intervention is required. It is sometimes difficult to differentiate testicular torsion from acute epididymitis (which presents similarly), however in EMQs you can tell the difference between the two from two key features: 1) in testicular torsion, pain is not relieved by elevating the twisted testis (negative Prehn’s sign), but pain is relieved in acute epidiymitis and 2) in testicular torsion, the affected testis usually lies high in the scrotum, but does not in epididymitis

153
Q

A 49-year-old man complains of grandually enlarging painless swelling of his left scrotum. HE says it has been present for at least 2 years. On examination, the testis is impalpable, but it is possible to get above the swelling. The swelling transilluminates brightly. Diagnosis A.Torsion of the hyatid of Morgagni B.Epididymo-orchitis C.Testicular torsion D.Hydrocele E.Teratoma F.Varicocele G.Mumps orchitis H.Seminoma

A

Hydrocoele A hydrocele is an abnormal collection of serious fluid in the tunica vaginalis surrounding the testis (hence the impalpable testis). Primary hydroceles usually occur in older men, develop slowly and are painless. Secondary hydroceles can occur secondary to tumours or inflammation of the underlying testes and epididymis. Characteristically, hydroceles transilluminate brightly.

154
Q

A 28-year-old man complains of a ‘dragging’ sensation and slight discomfort in his scrotum. On examination, the swelling is located above the testis, but is limited to the scrotum. It disappears when the patient lies flat, but there is no cough impulse. Diagnosis A.Torsion of the hyatid of Morgagni B.Epididymo-orchitis C.Testicular torsion D.Hydrocele E.Teratoma F.Varicocele G.Mumps orchitis H.Seminoma

A

Varicocoele The description in this case is characteristic of a varicocele. A varicocele is a collection of varicose veins in the pampiniform plexus (the venous network that drains the testicle). Patients present with a scrotal swelling that is visible on standing and characteristically feels like a ‘bag of worms’. It is more common on the left and may occur secondary to obstruction of the left testicular vein by a renal adenocarcinoma.

155
Q

A 35-year-old man with a history of undescended testis presents with a hard painless testicular lump. Blood tests shown an elevated β-human chorionic gonadotrophin. A.Torsion of the hyatid of Morgagni B.Epididymo-orchitis C.Testicular torsion D.Hydrocele E.Teratoma F.Varicocele G.Mumps orchitis H.Seminoma

A

Seminoma There are two main types of malignant testicular tumours: seminomas (60%) and teratomas (40%). Seminomas arise from the seminiferous tubules and occur in the 30-40 age group whereas teratomas are germ-cell tumours which occur in 20-30 year olds. They are associated with undescended testis (even after orchidopexy) and may both present as a hard, irregular, painless mass, or as a secondary hydrocele. They may spread to the lungs and liver (via the blood stream) and to the para-aortic lymph nodes (not inguinal!) Teratomas produce both alpha-fetoprotein (α-FP) and beta-human chorionic gonadotrophin (β-hCG), but seminomas only produce the latter.

156
Q

A 20-year-old man complains of mild pain in his left testicle after playing cricket in the park. On examination the left testicle is irregular in shape compared to the right. He reports having an orchidopexy when he was younger. Diagnosis A.Torsion of the hyatid of Morgagni B.Epididymo-orchitis C.Testicular torsion D.Hydrocele E.Teratoma F.Varicocele G.Mumps orchitis H.Seminoma

A

Teratoma There are two main types of malignant testicular tumours: seminomas (60%) and teratomas (40%). Seminomas arise from the seminiferous tubules and occur in the 30-40 age group whereas teratomas are germ-cell tumours which occur in 20-30 year olds. They are associated with undescended testis (even after orchidopexy) and may both present as a hard, irregular, painless mass, or as a secondary hydrocele. They may spread to the lungs and liver (via the blood stream) and to the para-aortic lymph nodes (not inguinal!) Teratomas produce both alpha-fetoprotein (α-FP) and beta-human chorionic gonadotrophin (β-hCG), but seminomas only produce the latter.

157
Q

A 53-year-old woman presents with mass below the angle of the jaw. She sayd it has been gradually increasing in size for the past 6 months. On examination, it is mobile and firm to the touch. There is no associated pain or facial weakness. Investigation Ultrasound Carotid angiography Fine-needle aspiration Excision biopsy Doppler ultrasound Sialogram Iodine uptake scan Technetium scan

A

Excision biopsy The symptoms here are suggestive of a parotid tumour, probably a pleomorphic adenoma (carcinoma would be painful, rapidly growing and may cause facial nerve palsy). Excision biopsy is required to make the diagnosis and treatment.

158
Q

A 16-year-old boy presents with a painless lump on the anterior border of the upper sternocleidomastoid on the left side. On examination, it is smooth and fluctuant, but does not transilluminate. He says that it previously became painful during a throat infection, but hasn’t bothered him since. There are no other abnormalities. Investigation Ultrasound Carotid angiography Fine-needle aspiration Excision biopsy Doppler ultrasound Sialogram Iodine uptake scan Technetium scan

A

Fine needle aspiration This boy has a branchial cyst. The clues to the diagnosis are the position and the fact that it became inflamed during a respiratory infection. Diagnosis is by fine-needle aspiration again, which demonstrates a creamy yellow turbid fluid containing cholesterol crystals. Treatment is by excision.

159
Q

A 70 year old male life long smoker complains of painless haematuria to his GP Diagnosis A.Urinary tract infection B.Ureteric colic C.Prostate cancer D.Renal cell carcinoma E.Prostatic varices F.Bladder cancer G.Trauma H.Acute pyelonephritis

A

Bladder cancer Gross haematuria is the primary symptom of bladder cancer. Risk factors include smoking, exposure to carcinogens such as the aromatic amines used in rubber and dye industries, age >55, pelvic radiation and Schistosomiasis resulting in SCC (related to chronic inflammation – so other risks also include UTI, stones etc). Bladder cancer is the most common cancer in Egypt, for the latter reason. Cystoscopy and urinary cytology are key in diagnosis. Low grade tumours are papillary and easy to see on cystoscopy whereas high grade tumours and carcinoma in situ are often difficult to visualise. Resection provides diagnosis and primary treatment in one step.

160
Q

An 81-year-old male complains of haematuria. He has been an inpatient for 3 weeks following admission for a left hip replacement due to osteoarthritis. His catheter was removed two days ago as he is becoming increasingly mobile. Diagnosis A.Urinary tract infection B.Ureteric colic C.Prostate cancer D.Renal cell carcinoma E.Prostatic varices F.Bladder cancer G.Trauma H.Acute pyelonephritis

A

Trauma This patient has had a catheter removed two days ago, which accounts for his gross haematuria (instrumentation of the urinary tract causing damage). The diagnosis is clinical based on the presence of the catheter.

161
Q

A 56 year old smoker complains of seeing bright red blood when he passes water. He mentions no pain but did have a UTI 5 years ago. Diagnosis A.Henoch-Schonlein purpura B.Waldenstrom’s macroglobinaemia C.Bladder cancer D.Goodpasture’s disease E.Nephrotic syndrome F.Post infectious glomerulonephritis G.Ureteric colic H.UTI I.Pseudo-haematuria

A

Bladder Cancer Gross (classically painless) haematuria is the primary symptom of bladder cancer. Risk factors include smoking, exposure to carcinogens such as the aromatic amines used in rubber and dye industries, age >55, pelvic radiation and Schistosomiasis resulting in SCC (related to chronic inflammation – so other risks also include UTI, stones etc). Bladder cancer is the most common cancer in Egypt, for the latter reason. Cystoscopy and urinary cytology are key in diagnosis. Low grade tumours are papillary and easy to see on cystoscopy whereas high grade tumours and carcinoma in situ are often difficult to visualise. Resection provides diagnosis and primary treatment in one step.

162
Q

A 5 year old boy has haematuria and a non-blanching rash over his upper thighs Diagnosis A.Henoch-Schonlein purpura B.Waldenstrom’s macroglobinaemia C.Bladder cancer D.Goodpasture’s disease E.Nephrotic syndrome F.Post infectious glomerulonephritis G.Ureteric colic H.UTI I.Pseudo-haematuria

A

Henoch-Schonlein purpura Henoch-Schonlein purpura is the most common vasculitis in childhood and in all cases there is a rash of palpable purpura which are typically non-blanching. If there is no rash, then it is not HSP. They are normally 2-10mm in diameter and are due to the extravasation of blood into the skin. They can occur anywhere on the body but are usually concentrated on the lower extremities. Half of all patients have abdominal pain and arthralgias are commonly present (found in about 80%) and often associated with oedema. The joints most often affected are the knees and ankles. About half will show signs of renal disease such as proteinuria or haematuria. Risk factors for this condition include being male, age 3-15 and history of prior UTI. Complications can occur and the most common cause of death is renal failure. Serum IgA levels may also be elevated.

163
Q

A 60 year old obese man presents to A&E with a history suggesting biliary colic. His medical history includes hypertension (treated with an ACE inhibitor) and dyslipidaemia. She smokes regularly and drinks alcohol socially. Abdominal ultrasound demonstrates gallstones as well as a 6cm left-sided renal mass. On further questioning, there has been haematuria. Diagnosis A.HIV B.Hyperkalaemia C.Benign renal cyst D.Polycystic kidney disease E.Pyelonephritis F.Chronic kidney disease G.Bladder cancer H.Rhabdomyolysis I.Ureteric cancer J.UTI K.Renal artery stenosis L.Renal cell carcinoma M.Renal tuberculosis

A

Renal cell carcinoma Renal cancer arising from the parenchyma/cortex is known as renal cell carcinoma. Clear cell renal cell carcinoma accounts for most primary renal cancers. They are often asymptomatic and diagnosed incidentally like on imaging when localised malignant looking renal masses are seen. Surgery for early local disease (which is diagnosed in more than half) can be curative in up to 90%. Renal masses are usually only symptomatic in late disease. The classic triad is of haematuria, flank pain and an abdominal mass – this is only seen in 10%. Uncommonly, a patient may present with symptoms of metastatic disease such as bone pain or respiratory symptoms. Symptoms, if present, also include abdominal pain, oedema/ascites from IVC disruption and scrotal varicocele in males. Risk factors include: smoking, male gender, living in developed countries, obesity, hypertension, FH, high parity and ionising radiation.

164
Q

A man comes to clinic complaining of weight gain and a cut on his leg that just won’t heal. He has various blood tests performed including a CRH (corticotrophin releasing hormone) stimulation test. His ACTH and cortisol are both shown to rise. Diagnosis A.Adrenal adenoma B.Tuberculosis C.Diabetes Insipidus D.Addison’s disease E.Addisonian crisis F.pituitary apoplexy G.Nelson’s syndrome H.Cushing’s disease I.Ectopic ACTH producing tumour J.Sheehan’s syndrome K.Drug withdrawal L.Conns Adenoma

A

Cushing’s Disease This man has weight gain and impaired wound healing, already you should be thinking cushings or diabetes…The blood test used here is being performed to differentiate between cushings disease and cushings syndrome. 10ug ovine or human CRH is administered to a patient who has fasted at least 4 hrs. pre test and serial post test ACTH and cortisol measurements are taken. In pituitary driven cushings disease the cortisol can be manipulated, that is to say exogenous CRH will cause a recordable increase in serum cortisol by the 2hr mark (and ACTH). However if the source of the excess cortisol is an ectopic ACTH producing tumour for example, this is not subject to any feedback mechanisms and so no change in serum cortisol will be seen on giving CRH.

165
Q

A Woman who has been complaining of worsening loss of peripheral vision for months suddenly takes a turn for the worse. She has an intense headache come on suddenly at home, by the time she is driven to hospital she has vomited a number of times and now complains of double vision. Her blood pressure also falls drastically. She is managed initially with hydrocortisone iv. and N. saline. Thyroxine follows. A Lumbar puncture is requested, it shows Xanthochromia. Diagnosis? A.Adrenal adenoma B.Tuberculosis C.Diabetes Insipidus D.Addison’s disease E.Addisonian crisis F.pituitary apoplexy G.Nelson’s syndrome H.Cushing’s disease I.Ectopic ACTH producing tumour J.Sheehan’s syndrome K.Drug withdrawal L.Conns Adenoma

A

Pituitary apoplexy Has bitemporal hemianopia which has been worsening..this should spark thoughts of a growing pituitary adenoma. But then she presents with an acute change. With a tumour a sudden change is usually vascular.. in this case the patient is suffering pituitary apoplexy whereby her enlarging pituitary tumour has undergone haemorrhage. This has basically wiped out pituitary function in a moment which explains her cardiovascular collapse and immensely painful headache. In fact apoplexy can present just like a sub-arachnoid haemorrhage in patients with known adenomas… Splitting headache - INCREDIBLY BAD - Xanthochromia (from extravasation of blood through the diaphrgam sellae into the sub aracahnoid space) - Vomiting - Loss of consciousness. The opthalmoplegia (diplopia) is a mass effect f the tumour being displaced by the bleed to impinge on the cavernous sinus and hence the IIIrd Cranial Nerve. Management is fluid resuscitation, and then hormone replacement of all lacking hromines! NB:In EMQS xanthochromia is usually pathognomonic of SAH.

166
Q

An overweight 60 year old lady is seen in clinic complaining of tiredness and general weakness, her eyesight is also suffering, she says she can’t see the room if she looks at a book and it’s getting worse. Physical examination reveals multiple scars on her abdomen, they are noted to be dark, in stark contrast to her purple striae. The notes show the patient was on metyrapone and aminoglutethamide before her surgery. Diagnosis? A.Adrenal adenoma B.Tuberculosis C.Diabetes Insipidus D.Addison’s disease E.Addisonian crisis F.pituitary apoplexy G.Nelson’s syndrome H.Cushing’s disease I.Ectopic ACTH producing tumour J.Sheehan’s syndrome K.Drug withdrawal L.Conns Adenoma

A

Nelson’s syndrome Worsening loss of peripheral vision infers pituitary adenoma. Hyperpigmented scars suggsts a state of increased ACTH production however this patient doesn’t seem to be in adrenal failure, she is overweight. She has had abdominal surgery and was on meyrapone ( a drug which inhibits cortisol production) and Aminoglutethamide (an adrenolytic) Basically this woman is overweight and has striae because she was cushingoid.. the cause was adrenal hyperplasia. She was on medication to control this but the decision was made to operate and a bilateral adrenalectomy was performed. The consequences f this is that she has very minimal cortisol production and her pituitary is striving to fuel this by pumping out lots of ACTH (hence the dark scars). The visual field problems are due to an undiagnosed pituitary adenoma that was non functional but since her surgery the pituitary tumour has grown substantially since there is no negative feedback coming from circulating cortisol. This is basically Nelson’s syndrome. NB : these days bilateral adrenalectomy is very rare and the pituitary is sometimes irradiated to prevent Nelson’s from occurring. This patient needs glucocorticoid replacement and transsphenoidal surgery.

167
Q

A woman developed pre-eclampsia during her first pregnancy, she also required syntocin and several units of blood during the birth. 4 months after the birth she still had not resumed menstruation and felt generally tired. She also noticed loss of pubic hair Diagnosis A.Adrenal adenoma B.Tuberculosis C.Diabetes Insipidus D.Addison’s disease E.Addisonian crisis F.pituitary apoplexy G.Nelson’s syndrome H.Cushing’s disease I.Ectopic ACTH producing tumour J.Sheehan’s syndrome K.Drug withdrawal L.Conns Adenoma

A

Sheehan syndrome Pre-eclampsia (proteinuria + HTN in pregnancy) is a major risk for post partum haemorrhage…In this ladies case she had to receive blood products and Syntocin (a synthetic oxytocin drug used to combat uterine atony and help stop haemorrhaging). Blood loss doesn’t normally cause changes in pituitary function however in pregnancy the pituitary is particularly vulnerable. Hyperplasia and Hypertrophy of the lactotrophe cells in pregnancy ( due to increased LH/FSH) leads to an increase in size of the adenphypophysis, however there is no such increase in the vascular supply to the anterior pituitary. When blood is lost in PPH the ant. pituitary thus infarcts and necroses. This is Sheehan’s syndrome…it can present insidiously or in a major way. It may be picked up if the mother fails to breast feed, but amennorhoea following birth and loss of pubic and axillary hair is also a common presentation. Only occasionally is full blown Sheehan’s encountered where the mother becomes hypothyroid and can develop Diabetes mellitus due to wildly deranged Ant. Pituitary function. DX: low oestradiol and pituitary hormone levels. NB: Diabetes Insipidus is a rare complication since the posterior pituitary has a rich arterial blood supply that is rarely compromised in PPH.

168
Q

Addisons can be caused by Cytomegalovirus (CMV)? True or False

A

True This is true although CMV would be a highly unlikely cause in an immunocompetent person. Patients who are HIV+ however are at risk from opportunisitic infections like this which can cause adrenal destruction.

169
Q

Phaechromocytomas are [blank-1] tumours originating usually from the adrenal [blank-2], they are usually [blank-3] though around 10% are [blank-4]. Following the rule of 10s, 10% of cases are also bilateral and 10% are extradrenal and 10% recur. [blank-5] of Phaechromocytomas are associated with familial gene mutations. In particular phaeos are seen in [blank-6] and [blank-7], it also features in [blank-8] though not [blank-9]. MEN 2B 25% benign medulla von hippel lindau - VHL malignant MEN 2A MEN 1 neuroendocrine

A

Phaechromocytomas are neuroendocrine tumours originating usually from the adrenal medulla, they are usually benign though around 10% are malignant. Following the rule of 10s, 10% of cases are also bilateral and 10% are extradrenal and 10% recur. 25% of Phaechromocytomas are associated with familial gene mutations. In particular phaeos are seen in MEN 2A and MEN 2B, it also features in von hippel lindau - VHL though not MEN 1. MEN1 = 3Ps: Pituitary, Parathyroid, Pancreas MEN2A = Medullary thyroid Ca, Phaeo + Pituitary MEN2B = Medullary thyroid Ca, Phaeo + neurofibromas + Marfanoid

170
Q

ADH is produced by the adenohypophysis? True or False

A

False ADH is produced along with oxytocin by the hypothalamus in the supraoptic and paraventricular nuclei. These hormones are projected via Parvocellular cells to the median eminence where they can influence the adenohypophysis (anterior pituitary)…remember ADH influences release of ACTH. Magnocellular pathways carry the ADH and oxytocin to the posterior pituitary to be released.

171
Q

A man complaining of vague abdominal pain + general lethargy has a short SynACTHen test performed. Pre test cortisol was 200mM 30 minutes after administration of tetracosactide the serum cortisol is 370mM Pick the most appropriate conclusion.. Primary adrenal insufficiency Normal response Secondary adrenal insufficiency

A

Primary adrenal insufficiency Tetracosactide is synthetic ACTH. When administered during a short synacthen test it can be used to determine whether a patient has adrenal or pituitary hypocortisolism. A Normal test would be : basal cortisol >170mM and 30 minutes cortisol should be at least double the basal measurement and be greater than 550mM. This man’s basal cortisol is normal however his 30 minutes cortisol has less than doubled and is less than 550mM which indicates that the adrenals are not functioning properly. In secondary adrenal insufficiency the basal cortisol would be very low but would exhibit large increases on administration of tetracosactide.

172
Q

Reduction of the HbA1c from 8% to 7% in diabetic reduces the risk of microvascular complications by approximately..? A) 20% B) 40% C) 5% D) 60%

A

40% So although HbA1c can not be used diagnostically in diabetes it represents the amount of endogenous glycation occuring in vivo . Glycation produces Advanced glycation endproducts which are responsible for damaging vasa nervorum etc causing microvascular complications of diabetes. Lens protein Glycation is also responsible for the increased occurrence of catract in Diabetics.

173
Q

A pregnant woman in her second trimester has a diffusely enlarged goitre with an audible bruit, she is currently thyrotoxic and carbimazole isn’t working as well as hoped. The doctors want to treat her with something in preparation for surgery. Appropriate treatment A.Radioiodine B.Stop treatment C.Potassium Iodide D.Carbimazole E.Surgical decompression F.Propylthiouracil G.Propranolol

A

Potassium iodide. This lady is suffering with Grave’s disease and requires removal of her goitre as she is likely to have high circulating levels of TSI (thyroid stimulating immunoglobulin) which can cross the placenta and stimulate toxicosis in the foetus..leading to failure to thrive etc. The fetal thyroid status can be measured via it’s heart rate. Carbimazole is standard management but if this doesn’t control maternal Grave’s surgery can be performed. As she has an audible bruit her goitre is likely extremely vascular and as she is pregnant the risks of excessive blood loss during thyroidectomy are multiple and serious. KI - potassium iodide can be given to saturate the thyroid and reduce vascularity pre surgery. Radioiodine is of course out of the question-risk of serious teratogenesis.

174
Q

A man is seen in the endocrinology clinic for his thyroid condition. He has the ‘thyroid stare’ and closer inspection of his eyes shows the optic disc is pale and swollen. Appropriate treatment A.Radioiodine B.Stop treatment C.Potassium Iodide D.Carbimazole E.Surgical decompression F.Propylthiouracil G.Propranolol

A

Surgical decompression There is significant retroorbital oedema and fibrosis around the optic nerve, decompressive surgery should help relieve this and prevent further atrophy.

175
Q

An 80 year old woman is brought into hospital following a house visit. She was initially confused, but in hospital she is found to be hypoventilating, her bloods show low BMs and hyponatraemia. Drs order hydrocortisone and liothyronine sodium Diagnosis A.Addisonian crisis B.Myxoedema coma C.Grave’s D.Hashimoto’s thyroiditis E.Thyroxine abuse F. External neck irradiation G. De Quervains thyroiditis H.Amiodarone

A

Myxoedema coma This patient is at the severe end of the hypothyroid spectrum. She has depressed level of consciousness (this can descend into frank coma) and is hypoventilating, other features that can present in this serious complication are bradycardia, and hyponatraemia. Further to this a massive proportion are hypothermic (core temp

176
Q

A 46-year-old man attends for outpatient assessment following referred by his GP. He complains of an episodic sudden onset of very a fast pulse rate, and a fluttering sensation within his chest. He takes his pulse at the time and states the rate can be greater than 200 beats per minute. Each episode lasts typically for few minutes and then reverts spontaneously. His GP had requested a 24 hour tape recording, but this was normal. Diagnosis a. Atrial fibrillation b. Aortic stenosis c. Aortic incompetence d. Mitral stenosis e. Supraventricular tachycardia f. Complete heart block g. Benign ventricular ectopics h. Anxiety

A

E.SupraventricularTachycardia(SVT). This is the classical history associated with SVT. It often presents in younger patients, although it can be seen in older patients. The arrhythmia typically arises from the atria and therefore the rate is often greater than 200 beats per minute. Episodes are often self-terminating, but if treatment is required adenosine is usually the first line drug. Beta- blockers are also useful. If haemodynamically unstable cardioversion should be performed as it is safe and effective.

177
Q

An 84 year old farmer is admitted to the acute medical receiving unit with dizzy spells. His pulse is very slow and irregular. His blood pressure is 120/80 mmHg. There are intermittent irregular visible pulsations noted on inspection of his venous pressure Diagnosis a. Atrial fibrillation b. Aortic stenosis c. Aortic incompetence d. Mitral stenosis e. Supraventricular tachycardia f. Complete heart block g. Benign ventricular ectopics h. Anxiety

A

A4: F. Complete Heart Block. Patients with complete AV block typically experience bradycardia (as low as 28 beats per minute during sleep), hypotension, and at times, hemodynamic instability. In some cases, exercising may be difficult, as the heart cannot react quickly enough to sudden changes in demand or sustain the higher heart rates required for sustained activity. They often present with dizzy spells. In this case treatment will be referral for a permanent pacemaker, however a temporary wire is not required as he is haemodynamically stable.

178
Q

Which of the following methods can be used to confirm the diagnosis of asthma? 1. PEF monitoring to detect diurnal variation 2. Assessing the response to a trial of inhaled / oral steroids for 2 weeks 3. Full history only 4. Positive exercise testing 5. Assessing the spirometry response to inhaled bronchodilators

A

1 + 2 + 4 + 5 Peak flow monitoring - demonstrate variability over time Response to steroids over time (>2 weeks) - 20% increase or 200ml ↑ in FEV1 Spirometry - Responsive to bronchodilators - 15% change or 200ml ↑ FEV1 Positive exercise test - Induced bronchoconstriction causing a reduction in FEV1 - >15% reduction is diagnostic

179
Q

Which of the following are recommended non-pharmacological methods of asthma management? 1. Smoking cessation 2. Weight reduction 3. Immunotherapy 4. Complimentary therapies 5. Allergen avoidance 6. Dietary manipulation

A

1 + 2 + 3 + 5 Allergen avoidance - can reduce disease severity Smoking cessation - can also reduce disease severity Complimentary therapies - no current evidence exists to prove that these reduce the severity of disease at all Dietary manipulation - no evidence exists to support this either Weight reduction - can result in improved control in patients who are obese Immunotherapy - in a small subgroup of patients, desensitisation via allergen specific immunotherapy may be beneficial

180
Q

Which of the following are signs of COPD? 1. Mucous gland hypoplasia 2. Chronic inflammation and fibrosis 3. Emphysema 4. Thinning pulmonary alveolar walls and wall remodelling

A

2 + 3 1) Mucous gland hyperplasia occurs, especially in large airways, with increased mucous secretion and therefore a chronic productive cough 2) Chronic inflammation and fibrosis occurs, characterised by CD8 lymphocyte, macrophage and neutrophil infiltration, and release of pro-inflammatory cytokines 3) Emphysema occurs due to alveolar wall destruction, causing irreversible enlargement of air spaces, and thus a loss of lung elasticity 4) Alveolar walls thicken and are remodelled due to hypoxia. This causes pulmonary vascular resistance, pulmonary hypertension and impaired gas exchange

181
Q

Which of the following are recommended non-pharmacological methods of treating COPD? 1. Smoking Cessation 2. Exercise Restriction 3. Pulmonary Rehabilitation 4. Diet 5. Education 6. Psychosocial Support

A

1 + 3 + 4 + 5 + 6 1) Smoking cessation is very important in treating COPD. It is the only intervention that is proven to decrease any decline in lung function that is smoking-related 2) Exercise restriction is NOT a method of treatment - in fact graded exercise is the mainstay of pulmonary rehabilitation 3) Pulmonary rehabilitation is very useful in improving exercise tolerance, quality of life and reducing hospital admissions. It involves graded exercise, breathing techniques and education 4) Diet is important for COPD control in the obese. Losing weight physically reduces the amount of weight the patient must lift their chest wall against with each breath, therefore making repiratory effort less. 5) Education has an important role in assisting patients to manage their illness and stop smoking too. 6) Psychosocial support also plays an important part, since practical support can be offered at home, as well as day centres, and logistics of illness can be considered, such as disability badges for cars. It is also an important way of assessing for signs of anxiety and depression

183
Q

A 36 year old baker presents to his GP complaining of shortness of breath and cough. He says he has noticed it becoming worse over the last 9 months, since he has started his new job, but it does not bother him so much at weekends. Diagnosis A.Cystic fibrosis B.EAA C.Bronchiectasis D.Sarcoidosis E.Tuberculosis F.Foreign body inhalation G.Asthma H.Community acquired pneumonia I.RSV infection J.Epiglottitis K.Aspergillosis L.Pertussis

A

Asthma This man’s symptoms directly correlate with the time that he has been working as a baker, including a variation between the weekdays when he is working, and the weekend when he is not. There are various occupations in which small dust particles produced by the occupation results in irritation of the lung lining – causing symptoms of asthma.

184
Q

A 43 year old businessman who has just returned from a conference in Cuba presents to his GP with a dry cough and dypnoea lasting 2 days. He reports having felt generally unwell for the last 3 days before his cough started. LFTs are deranged, and CXR shows bibasal consolidation. Diagnosis A.Allergic bronchopulmonary aspergillosis B.RSV infection C.ACE inhibitors D.Tuberculosis E.Bronchial asthma F.Cystic fibrosis G.Foreign body inhalation H.COPD I.Bronchial carcinoma J.Malaria K.Legionella pneumonia L.Beta-blockers

A

Legionella pneumophila Legionnaire’s disease is more common in those who have been staying in air-conditioned hotels (since Legionella pneumophila commonly colonises water tanks). This bacteria is found in aqueous environments such as lakes and almost all cases are from contaminated water systems, which relates to the risk factors of getting Legionella. This gentleman is experiencing all the classic symptoms, but people can also experience extra-pulmonary symptoms such as diarrhoea, vomiting, anorexia, hepatitis, renal failure and coma. It can also cause confusion as well as hyponatraemia, abdominal pain and bradycardia. Bi-basal consolidation on CXR is also characteristic. Legionella is a gram negative rod. Legionella infecting the lungs is legionnaires’ disease or Legionella pneumonia whereas non-lung infection is known as Pontiac fever. Smoking is also a risk factor. Legionella does not grow on routine culture media and diagnosis relies on urine antigen detection, serology or culture on special media.

185
Q

A 27 year old Eastern-European man presents to you following an episode of coughing up blood. He has been experiencing a persistent cough, producing copious amounts of purulent sputum. He says he suffered from measles last winter, but has otherwise always been a healthy person. On examination you can hear coarse inspiratory crepitations. The radiologist reports cystic shadows and thickened bronchial walls on CXR. Diagnosis A.Tuberculosis B.Pulmonary embolism C.Post-intubation D.Sarcoidosis E.Bronchiectasis F.Pancoast tumour G.Pulmonary oedema H.Cystic fibrosis I.Bronchial carcinoma J.Goodpasture’s syndrome K.Amyotrophic lateral sclerosis

A

Bronchiectasis Bronchiectasis is permanent bronchi dilatation due to bronchial wall damage and loss of elasticity. It is often as a consequence of recurrent/severe infections and most present with chronic productive mucopurulent cough. The most commonly identifiable cause is CF. Textbook signs are persistent cough, with copious purulent sputum together with intermittent haemoptysis. This can also be congenital, or post-infection (in this case our patient has recently recovered from measles, although other possible infections may be pertussis, pneumonia or TB). CXR report describing cystic shadows and thickened bronchial walls is also characteristic. Chest CT is the diagnostic test. Diagnosis is aided by sputum analysis.

185
Q

What are the 5 commonest sites of metastasis of lung cancer?

A

Skin Bone Adrenals Liver Brain

186
Q

A 29 year old patient is rushed to A+E with respiratory distress. His wife says he started coughing and gasping for breath, and that he was recently prescribed antibiotics for a throat infection. Diagnosis A.Cystic fibrosis B.EAA C.Bronchiectasis D.Sarcoidosis E.Tuberculosis F.Foreign body inhalation G.Asthma H.Community acquired pneumonia I.RSV infection J.Epiglottitis K.Aspergillosis L.Pertussis

A

Epiglottitis Epiglottitis is a cellulitis of the supraglottis that may lead to airway compromise and is an airway emergency, especially in children. It classically presents in children aged 2-6 years of age but may manifest in any age. It is possibly now more common in older children and adults due to the introduction and implementation of the Haemophilus influenzae type B (Hib) vaccine. The most common signs are a rapid onset of high fever, sore throat and the inability to control secretions. There may be classic tripod positioning, difficulty breathing and irritability. It is crucial not to take any action which could stimulate a child with this suspected diagnosis including examination of the oral cavity, blood draws or even separating them from their parent. The airway needs to be secured and antibiotics started. The conditon will often resolve rapidly. Most adults complain of sore throat and pain with swallowing. Note that vaccination does not exclude the possibility of this diagnosis.

187
Q

A young man involved in a RTA is brought into A&E with multiple injuries to his face. On examination, his eyes do not open to pain. He withdraws his left side to pain but his right side does not move at all. His right pupil is fixed, dilated and non-reactive. Diagnosis A.Hyperglycaemia B.Pancreatitis C.Hypoglycaemia D.Wernicke’s encephalopathy E.Anxiety attack F.Hypertension G.Subdural haematoma H.Pulmonary fibrosis I.Delirium tremens J.Acute gastritis K.Wolff-Parkinson-White syndrome L.Convulsions M.Atrial fibrillation N.Oesophageal varices

A

Subdural haematoma A subdural occurs due to blood collecting between the dura mater and the arachnoid mater surrounding the brain. It may be arterial or venous although is most often venous. The disease runs a varied course and the presentation occurs on a spectrum from asymptomatic to herniation syndromes. There is neurological deficit evident so surgery will be indicated here. The cause is trauma and this man will have suffered a head injury due to his RTA. It is important in the examination to look for signs of trauma such as scalp abrasions and bruises. Surgical options include twist-drill craniotomy with drainage (a bedside procedure where a hand drill is used to gain access to the subdural space and then a catheter is placed to act as a drain). Standard craniotomy is also an option, as is the creation of a burr hole. Remember that extradural haematomas classically have a ‘lucid interval’ and occur in younger patients, usually with an associated skull fracture, and CT of the haematoma does not cross suture lines.

188
Q

Mrs Loren, 26, gave birth to a healthy boy a week ago. She is attempting to breast feed but is having some difficulty. Over the past 24 hours, she felt like she had influenza coming on. She has a 5cm, very tender mass in her right breast and there is erythematous skin overlying this. Diagnosis A.Lipoma B.Basal cell carcinoma C.Radial scar D.Breast abscess E.Carcinoma of the breast F.Fibroadenosis G.Adenoma H.Sebaceous cyst I.Intraductal papilloma J.Breast bud K.Phylloides tumour L.Fat necrosis M.Fibroadenoma

A

Breast abscess Breast abscess presents with mastalgia and fever. Breast infection typically affects women who are lactating and the most commonly implicated pathogen is staphylococcus aureus. The painful red mass indicates the development of an abscess. Antibiotic therapy is indicated with surgical intervention such as aspiration and drainage with possible duct excision. Prompt management of mastitis when it presents will usually lead to a good timely resolution and prevent the development of complications such as an abscess. An USS can help to identify the underlying abscess which usually forms a hypoechoic lesion. Needle aspiration can be used both therapeutically and diagnostically and can be guided by ultrasound.

189
Q

An 8 year old boy is brought into A&E after falling from a tree while trying to retrieve his cat. The boy is crying and has periorbital bruising around his right eye and a small amount of blood in the right ear canal. His GCS is 15. A.Haemophilia B.HIV C.Clopidogrel D.Spontaneous E.Aplastic anaemia F.Wiskott-Aldrich syndrome G.Disulfiram H.Idiopathic thrombocytopenic purpura I.Skull fracture J.Diuretics K.Corticosteroids L.Hepatic cirrhosis

A

Skull fracture This is a basilar skull fracture and a CT scan (superior to MRI), in this case with 3D reconstructions, will be useful. This patient has had a fall and clearly hit his head. Basilar skull fractures have specific clinical features. Blood pooling from these fractures can cause periorbital bruising (raccoon eyes), brusing over the mastoid area (Battle’s sign) and bloody otorrhoea. There may also be CSF leak resulting in CSF otorrhoea or rhinorrhoea. A unilateral raccoon eye has an 85% positive predictive value for this diagnosis.

190
Q

A 34-year-old woman presents with irregular periods and weight gain. She had been on anti-depressants for six months and she had been complaining of tiredness. Pulse rate is 46/minute and regular. This is confirmed on ECG, which also shows low voltage. Diagnosis? A.Pregnancy B.Renal failure C.Amyloidosis D.Reduced activity E.Metabolic syndrome F.Addison’s disease G.Salt-wasting nephropathy H.Cushing’s syndrome I.Portal hypertension J.Comfort eating K.Heart failure L.Polycystic ovary syndrome M.Hypothyroidism

A

Hypothyroidism This patient has hypothyroidism. Worldwide, the most common cause is iodine deficiency. Other causes include Hashimoto’s or secondary and tertiary hypothyroidism. It can also result from viral de Quervain’s thyroiditis or postpartum thyroiditis. Symptoms include those mentioned (depression, fatigue, weight gain, bradycardia and menstrual problems) as well as others such as slow-relaxing reflexes on examination, constipation, cold intolerance, dry skin and muscle cramps. Diagnosis is based on measurement of TSH and thyroid hormones. Treatment is by replacement of T4 with or without T3 in combination. If the patient has normal T3 and T4 but mildly elevated TSH, this is described as subclinical hypothyroidism.

191
Q

A 45-year-old HCV positive Egyptian journalist presents with acute renal failure. He is complaining of increasing abdominal distension, pruritis, ankle oedema and weight gain. Serum albumin is low and there is hyponatraemia and thrombocytopenia. Diagnosis A.Pregnancy B.Renal failure C.Amyloidosis D.Reduced activity E.Metabolic syndrome F.Addison’s disease G.Salt-wasting nephropathy H.Cushing’s syndrome I.Portal hypertension J.Comfort eating K.Heart failure L.Polycystic ovary syndrome M.Hypothyroidism

A

Portal HTN HCV in this patient is causing hepatic cirrhosis which has decompensated resulting in ascites, secondary to portal hypertension. The hypoalbuminaemia is a sign of decreased hepatic synthetic function. Hyponatraemia is a common finding associated with ascites. It arises due to reduced protein synthesis and therefore a loss of colloid osmotic pressure and increased fluid loss from the intravascular compartment, stimulating ADH secretion. There is peripheral oedema here which is due to low albumin. The pruritis is due to reduced hepatic excretion of conjugated bilirubin and there may be accompanying jaundice too. The cause of his renal failure may well be hepatorenal syndrome in the context of his severe liver disease. His prognosis is poor.

192
Q

A middle-aged tramp presents in A&E intoxicated and confused. He has a half-finished bottle of vodka in his pocket. Additionally there is horizontal gaze palsy and severe ataxia with vertigo and headache. Diagnosis A.Fibromyalgia B.Delirium tremens C.Malnutrition D.Depression E.Peptic ulceration F.Wernicke’s encephalopathy G.Fatty liver H.Rhabdomyolysis I.Acute intoxication J.Cirrhosis K. Chronic subdural haematoma L.Macrocytosis

A

Wernicke’s encephalopathy Wernicke’s is due to acute thiamine deficiency, which is a problem in alcoholics. Others at risk include those with AIDS, cancer and treatment with chemotherapy, malnutrition and GIT surgery, especially bariatric procedures. It is a clinically under-diagnosed condition. The classic EMQ triad is of mental change, ophthalmoplegia and gait dysfunction, which is actually only seen in 10% of cases. In reality, the manifestations are varied and a high index of suspicion is needed. Despite there, the manifestiations typically include altered consciousness, gait disorders and eye movement abnormalities. This is an emergency and treatment is with parenteral replacement of thiamine. This avoids permanent neurological damage including later development of Korsakoff’s psychosis, which is irreversible. Note that thiamine should be given before dextrose! Magnesium deficiency also needs to be corrected as it is a co-factor in the functioning of thiamine dependent enzymes.

193
Q

83-year-old man with longstanding heart failure for which he takes Digoxin and diuretics. For the last 24 hours he has been vomiting and has passed very little urine. On examination he is pale and mildly dehydrated; examination of the abdomen is normal. Diagnosis A.Oesophageal cancer B.Pancreatitis C.Combined oral contraceptive pill D.Bowel obstruction E.Bulimia F.Pyloric stenosis G.Viral gastroenteritis H.Salmonella I.Peptic ulcer disease J.Uraemia K.Appendicitis L.Gastric cancer M.Intussusception

A

Uraemia This patient has developed acute renal failure, probably associated with the longstanding CCF. Advanced heart failure will lead to depressed renal perfusion and ARF. The decreased urine output is a symptom and the vomiting here is caused by uraemia or a general build up of waste products. An acute increase in creatinine will be seen, commonly with hyperkalaemia, hyperphosphataemia and a metabolic acidosis. There may also be respiratory compensation for this. Treatment is largely supportive, managing, in this case, the heart failure, and correcting abnormalities like volume status and the metabolic acidosis. Dialysis may be required.

194
Q

A 25-year-old traveling salesman is awoken in his hotel with crampy abdominal pain, feeling very ill and vomits three times over the next half an hour. He asks the receptionist to call a doctor. Diagnosis A.Oesophageal cancer B.Pancreatitis C.Combined oral contraceptive pill D.Bowel obstruction E.Bulimia F.Pyloric stenosis G.Viral gastroenteritis H.Salmonella I.Peptic ulcer disease J.Uraemia K.Appendicitis L.Gastric cancer M.Intussusception

A

Viral gastroenteritis Viral gastroenteritis often presents with mainly UGI symptoms like N&V more so than diarrhoea. It is on this basis that this is likely to be viral gastroenteritis caused by organisms such as rotavirus, norovirus and adenovirus. Staphylococcus aureus and Bacillus cereus also tend to cause mostly UGI symptoms and acts by preformed toxins, though these are not on the list of available options. The differentiation is made on studying the stool and identifying the organism.

195
Q

A 25-year-old man presents with a 6 month history of painless enlargement of the left hemiscrotum. The swelling is fluctuant, translucent, confined to the scrotum, and the testis cannot be felt separately. Diagnosis A.Heart failure B.Squamous cell carcinoma C.Varicocele D.Testicular torsion E.Epididymo-orchitis F.Hydrocoele G.Epididymal cyst H.Undescended testicle I.Hernia J.Testicular TB K.Testicular malignancy

A

Hydrocoele A hydrocele is a collection of serous fluid between the layers of the tunica vaginalis. Patients mainly present with a painless swollen scrotum (which can be on one or both sides) which feels like a water-filled balloon. Surgery is only performed if the hydrocele is problematic. As a result of the fluid, most hydroceles can be transilluminated. On examination, the hydrocele cannot be differentiated from the testicle. The mass can also increase in size with increased intra-abdominal pressure (such as coughing, crying or raising the arms) which causes peritoneal fluid to move into the scrotal sac. This causes the mass to vary in size during the day (smaller after lying down).

196
Q

A 70-year-old man present with mild dysuria, urinary hesitancy and terminal dribbling. He also has bilateral testicular pain, swelling and tenderness of both testes and epididymis. His temperature is 37.50ºC. Diagnosis A.Heart failure B.Squamous cell carcinoma C.Varicocele D.Testicular torsion E.Epididymo-orchitis F.Hydrocoele G.Epididymal cyst H.Undescended testicle I.Hernia J.Testicular TB K.Testicular malignancy

A

Epididymo-orchitis Generally speaking, younger males are more likely to have an STI whereas older men are more likely to have an infection with enteric organisms such as ESBL. He is mildly febrile with tenderness on-top of urinary symptoms which points towards epididymo-orchitis. A urethral swab should be sent for Gram stain and culture of secretions. A urine dipstick is also necessary combined with urine MC+S. Treatment involves bed rest, scrotal elevation, analgesia and antibiotics if indicated.

197
Q

An 80-year-old woman is admitted from a residential home with a two week history of purulent sputum and pyrexia (38ºC). Examination reveals a constant wheezing in inspiration and expiration localised over the right lung base. Diagnosis A.Pleural effusion B.Anaphylaxis C.Allergic alveolitis D.Bronchial adenoma E.Inhaled foreign body F.Cystic fibrosis G.COPD H.Pneumothorax I.Bronchial asthma J.Fibrosing alveolitis K.Influenza L.Left ventricular failure

A

Inhaled foreign body. This is aspiration pneumonia. There are symptoms of pneumoia with pyrexia and purulent sputum, along with risk factors for aspiration in this elderly person who may have difficulties swallowing or altered mental status from, for example, dementia. The location of the wheeze is also consistent with this diagnosis as the RLL is the most common site due to the anatomy of the bronchial tree. Complications include abscess and empyema. Treatment is predominantly with antibiotics and supportive care.

198
Q

A 35-year-old woman has a 10-year history of low retrosternal dysphagia and painless regurgitation of food in the mouth Diagnosis A.Cerebrovascular accident B.Pharyngeal pouch C.Myasthenia gravis D.Hiatus hernia E.Plummer-Vinson syndrome F.Carcinoma of oesophagus G.Thyroid goitre H.Achalasia I.Gastric volvulus J.Pneumonia K.Carcinoma of bronchus

A

Hiatus hernia A hiatus hernia is where intraabdominal contents protrude through the oesophageal hiatus of the diaphragm. Risk factors include obesity and high intra-abdominal pressure. The condition may be asymptomatic, or it may present with symptoms (which are non-specific) such as heartburn, dysphagia, pain on swallowing, wheezing, hoarseness and chest pain. A CXR is the first test done and may show an air bubble in the wrong place but barium studies are diagnostic and treatment depends on the symptoms and anatomy of the hernia. Hernias can be sliding or rolling (or mixed, or giant), uncomplicated or complicated by, for instance, obstruction and bleeding. Do you know the difference between a sliding and a rolling hiatal hernia?

199
Q

A 45-year-old lady presents with retrosternal dysphagia. She has spoon-shaped nails and is noted to be pale. Diagnosis A.Cerebrovascular accident B.Pharyngeal pouch C.Myasthenia gravis D.Hiatus hernia E.Plummer-Vinson syndrome F.Carcinoma of oesophagus G.Thyroid goitre H.Achalasia I.Gastric volvulus J.Pneumonia K.Carcinoma of bronchus

A

Plummer-Vinson syndrome Plummer-Vinson syndrome is the association of chronic IDA (shown here by the koilonychia and paleness on examination) with dysphagia due to a post cricoid web. Roughly 7% of those with IDA may complain of gradual onset dysphagia with the discomfort found in the area of the cricoid cartilage. Invasive procedures may be needed for management such as endoscopic dilation of the web but treatment is largely aimed at correcting the IDA.

200
Q

A 50 year old describes a 5 month history of heartburn and cramp-like chest pain relived by drinking cold water, both unrelated to food. There has also been intermittent dysphagia to both liquids and solids, regurgitation and weight loss of 2kg. Diagnosis A.Cerebrovascular accident B.Pharyngeal pouch C.Myasthenia gravis D.Hiatus hernia E.Plummer-Vinson syndrome F.Carcinoma of oesophagus G.Thyroid goitre H.Achalasia I.Gastric volvulus J.Pneumonia K.Carcinoma of bronchus

A

Achalasia This is achalasia which is a motility disorder with loss of peristalsis in the distal oesophagus and failure of the LOS to relax in response to swallowing. This presents commonly with dysphagia to both liquids and solids, regurgitation and retrosternal chest pain, which can be slowly progressive over time. In structural obstruction such as cancer, dysphagia to liquids is uncommon unless the disease is very advanced. Retrosternal pressure experienced can be precipitated by drinking liquids but is eased by continuing to drink, and the pain may be relieved by cold water. This may wake the individual from sleep. A UGI endoscopy is needed to exclude malignancy as a cause of dysphagia. The diagnosis is established on manometry or barium studies. Treatment is symptomatic.

201
Q

A 20-year-old woman is assaulted in a nightclub. She suffered a short episode of loss of consciousness, double vision and has vomited once. She now has a headache Diagnosis Subdural haematoma Concussion Extradural haematoma Base of skull fracture Diffuse axonal injury Cerebral contusion Depressed skull fracture Cerebral haematoma

A

Concussion Concussion is a closed head injury due a blow to the head. This woman has been assaulted, which is a common cause. Typical symptoms include headache (which tends to be cause the most problems in management), mental slowness, memory difficulties, N&V and LOC, though LOC is not necessary for the diagnosis. Symptoms typically go after a week to a month but can fluctuate. CT and MRI are typically normal. For uncomplicated cases, resting is sufficient and no intervention is needed.

202
Q

A 21-year-old man who has been involved in a road traffic accident develops a continuous blood-stained discharge from the nose and describes an altered sensation of smell. Diagnosis Subdural haematoma Concussion Extradural haematoma Base of skull fracture Diffuse axonal injury Cerebral contusion Depressed skull fracture Cerebral haematoma

A

Base of skull fracture This is a basilar skull fracture and a CT scan (superior to MRI), in this case with 3D reconstructions, will be useful. These patients have hit their head head. Basilar skull fractures have specific clinical features. Blood pooling from these fractures can cause periorbital bruising (raccoon eyes), brusing over the mastoid area (Battle’s sign) and bloody otorrhoea. There may also be CSF leak resulting in CSF otorrhoea or rhinorrhoea. A unilateral raccoon eye has an 85% positive predictive value for this diagnosis. Base of skull fractures can also affect cranial nerves leading to hearing impairment (due either to SN as a result of VII injury or conductive due to haemotympanum) and facial numbness or paralysis. These patients also need to have their GCS calculated. Treatment is primarily conservative although surgical intervention may be needed if there is associated CSF leak, intracranial pathology or CN deficit. RTAs are the second most common cause of skull fractures, the first being a fall from a height. There may also, like the first case, be a history of assault resulting in head trauma or even a gunshot to the head.

203
Q

The immediate management of an acute deep vein thrombosis in someone who is at a high risk of bleeding. Management A.Vitamin K B.Fresh frozen plasma C.Vitamin E D.Warfarin E.Platelet concentrates F.Low molecular weight heparin G.Fondaparinux (FXa inhibitor) H.Protein S concentrate I.Thrombin infusion J.Platelet concentrates plus fresh frozen plasma K.Fibrinogen L.Unfractionated heparin M.Aspirin

A

Unfractionated heparin The mainstay of treatment for acute DVT is anticoagulation. This can be either unfractionated heparin, a LMWH or a factor Xa inhitor like fondaparinux. Fondaparinux has a higher half life than LMWH and there is no effective way of reversing it. LMWH have a shorter half life and some of it can be removed with protamine. Heparin though can be reversed quickly with protamine. Hence, if the patient is at a high risk of bleeding, they should be treated with unfractionated heparin and you should avoid fondaparinux. If they start bleeding you can just chuck them protamine. This however requires monitoring of APTT and platelet counts. If the patient has heparin-induced thrombocytopenia, you can try using fondaparinux. LMWH is recommended in those with active cancer and preferred in pragnancy, and consideration needs to be given in those with renal impairment.

204
Q

Disseminated intravascular coagulation. Management A.Vitamin K B.Fresh frozen plasma C.Vitamin E D.Warfarin E.Platelet concentrates F.Low molecular weight heparin G.Fondaparinux (FXa inhibitor) H.Protein S concentrate I.Thrombin infusion J.Platelet concentrates plus fresh frozen plasma K.Fibrinogen L.Unfractionated heparin M.Aspirin

A

Plasma concentrates plus FFP DIC is a syndrome where coagulation pathways activate resulting in intravascular thrombosis, platelet and clotting factor depletion. The underlying disorder needs to be treated and FFP with platelet concentrate needs to be given. A platelet infusion should be considered and FFP is preferred for replacement of clotting factors and clotting inhibitors. Complications of DIC include life-threatening bleed, ARF and gangrene.

205
Q

A 40-year-old female has a routine blood test showing serum calcium 2.85 mmol/l (corrected). She takes no medications. PTH is also elevated and ALP is mildly raised. She is depressed. Diagnosis A.Myeloma B.Pseudohypercalcaemia C.Metastatic breast carcinoma D.Sarcoidosis E.Secondary hyperparathyroidism F.Medullary cell carcinoma of the thyroid G.Tuberculosis H.Milk-alkali syndrome I.Immobility J.Thiazide diuretics K.Hypervitaminosis A L.Vitamin D intoxication M.Primary hyperparathyroidism N.Paget’s disease

A

Primary hyperparathyroidism This is the most common cause of hypercalcaemia. 85% are due to a single adenoma in one of the glands while most of the rest are due to 4 gland disease. The range of disease is from very mild/asymptomatic through to severe disease with bone loss, fractures and osteitis fibrosa cystica. There is autonomous PTH production in primary HPT which causes deranged calcium metabolism. Biochemistry will show elevated serum calcium and inappropriate elevation of PTH. Depression, cognitive changes, change in sleep (possibly due to change in circadian rhythm) and myalgia are all common complaints. Osteoporosis may occur due to excess PTH causing bone resorption (osteoclasts are stimulated). Hypercalciuria may cause renal stones and nocturia. The only definitive cure is a parathyroidectomy although complications of this procedure include hypocalcaemia, injury to the recurrent laryngeal nerve, bleeding and a pneumothorax.

206
Q

A 55-year-old lady with a family history (mother) of death following hip fracture is obsessed with the risk of osteoporosis and has been purchasing large quantities of OTC medication. There is hypercalcaemia. Diagnosis A.Myeloma B.Pseudohypercalcaemia C.Metastatic breast carcinoma D.Sarcoidosis E.Secondary hyperparathyroidism F.Medullary cell carcinoma of the thyroid G.Tuberculosis H.Milk-alkali syndrome I.Immobility J.Thiazide diuretics K.Hypervitaminosis A L.Vitamin D intoxication M.Primary hyperparathyroidism N.Paget’s disease

A

Vitamin D intoxication This silly lady has self-over-dosed on vitamin D. This is a fat soluble vitamin which can be toxic when taken in excess over time. OD of alfacalciol or calcitriol can easily cause hypercalcaemia and this needs to be avoided. So why is this not milk-alkali syndrome? Milk-alkali syndrome is caused by excess milk, as the name suggests, or alkali, for example due to dyspepsia. Excess intake, or calcium supplementation OTC can lead to hypercalcaemia, so granted if she is buying lots and lots of chewable calcium carbonate or something this is possible, but it is more likely in post-menopausal women on supplementation - just think about what is more likely here. Also, the patient is not completely stupid and will not be OD on vitamin A, even though is a cause of hypercalcaemia.

207
Q

A 44-year-old man, BP 175/110 mmHg, plasma potassium 2.2 mmol/L. Cause of HTN? A.Renovascular disease B.Metabolic syndrome (Insulin resistance/syndrome X) C.’Essential’ hypertension D.Phaeochromocytoma E.Primary hyperaldosteronism (Conn’s syndrome) F.Isolated systolic hypertension G.Coarctation of the aorta H.Cushing’s syndrome

A

Primary hyperaldosteronism (Conn’s syndrome) The normal range for potassium 3.5-5mmol/l. You should really know the normal ranges for common values like sodium, potassium and urea by this stage. In Conn’s, potassium is normal or low. It is important when drawing blood to avoid haemolysing the sample, which will cause a falsely elevated potassium level (pseudohyperkalemia). It is important for screening to calculate the aldosterone/renin ratio, with >30 being suggestive of Conn’s. In Conn’s, aldosterone is raised and renin is low due to negative feedback. This is in contrast to renal artery stenosis where both aldosterone and renin will be raised. It is important to discontinue diuretics and other interfering medications for at least 6 weeks prior to measuring the ratio. The most reliable diagnostic test is a fludrocortisone suppression test. Treatment can be surgical with excision of the adenoma (if aldosterone production is lateralised to one side) or medical with spironolactone and amiloride. There are also familial forms of primary hyperaldosteronism which show an autosomal dominant mode of inheritance.

208
Q

A 58 year old man who is recently diagnosed with lung cancer has started chemotherapy and radiotherapy. He complains of a fever, weight loss and fatigue. There are swollen lymph nodes and tetany. Serum potassium is elevated and calcium is low. Diagnosis? A.Pseudohyperkalaemia B.Hyperglycaemia C.Acute kidney failure D.Chronic kidney disease E.Rhabdomyolysis F.Drug side effect G.Infection H.Addison’s disease I.DKA J.Congenital adrenal hyperplasia K.Tumour lysis syndrome L.Cushing’s syndrome

A

Tumour lysis syndrome There is recent diagnosis of malignancy here combined with the recent start of both chemotherapy and radiotherapy. This is tumour lysis syndrome which encompasses metabolic and electrolyte abnormalities, like hyperkalaemia, occuring after cytotoxic treatment in a patient with cancer. There is excessive cell lysis and the release of intracellular contents into the bloodstream leads to elevated levels of serum urate, potassium, phosphate and a reduction in calcium level. (Ca binds to the newly release PO4)

209
Q

A 72 year old woman is being treated for a diabetic foot ulcer and is afraid she may need an amputation. Serum potassium today is markedly elevated. Results over the past week have been normal. You find out the 2nd year medical student had some difficulty drawing the blood. Diagnosis A.Pseudohyperkalaemia B.Hyperglycaemia C.Acute kidney failure D.Chronic kidney disease E.Rhabdomyolysis F.Drug side effect G.Infection H.Addison’s disease I.DKA J.Congenital adrenal hyperplasia K.Tumour lysis syndrome L.Cushing’s syndrome

A

Pseudohyperkalaemia This is pseudohyperkalaemia caused by haemolysis of the sample. The medical student who has had some difficulty drawing the blood has haemolysed the sample. Potassium in serum will in this case exceed the plasma value by >0.5 mmol/L and the pink tinge when centrifuging the sample will also give this away.

210
Q

A 32-year-old man attends your surgery saying he brought up blood every morning for the last week. He is unsure whether he is vomiting or coughing it up. His haemoglobin (done yesterday) is 14 g/dL. Investigation? A.Clotting profile B.Liver biopsy C.Liver function test D.Barium swallow E.H.pylori breath test F.Chest x-ray G.Upper GI endoscopy H.Upper GI endoscopy I.Full blood count J.CT scan abdomen K.Colonoscopy

A

CXR The blood being brought up here occurs at a fixed time period every day and he is not anaemic. This makes you suspect a respiratory cause of his haemoptysis such as pneumonia – particularly TB, cancer, vasculitis like Wegener’s, bronchiectasis and bronchitis.

211
Q

A 28-year-old woman with a carcinoma of the cervix was admitted with plasma creatinine of 250μmol/l. BP was 130/80. Urinalysis was negative. Investigation to find underlying cause for renal failure? Renal ultrasound CVP measurement Renal biopsy Plasma electrophoretic strip Intravenous pyelogram Renal arteriogram HIV test Anti-neutrophil cytoplasmic antibody Anti-glomerular basement membrane antibody Captopril renogram

A

Renal USS A patient with cervical carcinoma is at risk of ureteric obstruction and then hydronephrosis. USS is the investigation of choice.

212
Q

An 82-year-old man admitted through A&E with confusion. BP 90/60. JVP not seen. His hands and feet were cool. Plasma creatinine 420 μmol/l, urea 55 mmol/l. After catheterisation, 200mls mucky urine was drained from his bladder. Investigation to find underlying cause for renal failure? Renal ultrasound CVP measurement Renal biopsy Plasma electrophoretic strip Intravenous pyelogram Renal arteriogram HIV test Anti-neutrophil cytoplasmic antibody Anti-glomerular basement membrane antibody Captopril renogram

A

CVP measurement A confused hypotensive shocked patient might be hypotensive because of cardiac disease (which will cause a raised JVP) or due to sepsis (which will cause a low JVP). If you can’t see the JVP, then you should put in a central line to measure the CVP accurately to tell you if the cause of the low blood pressure here is sepsis or cardiac failure.

213
Q

An 80 year old man presents with severely painful feet with mottled and purple toes with black areas. He tells you that he has also had constant severe back pain for a few days. Investigation A.Arteriogram B.Full blood count C.Anti-neutrophil cytoplasmic antibody D.Venous duplex scan E.CT scan F.Cold provocation test G.Blood cultures H.Blood sugar I.Lumbar puncture

A

CT scan This patient has a dissecting aortic aneurysm which can be diagnosed with a CT scan showing the presence of an intimal flap. The CT scan should include chest, abdomen and pelvis to visualise the extent of the aneurysm. Dissecting aneurysms are either type A, which involves the ascending aorta, or type B. Type A dissections require urgent surgery whereas type B can be managed medically if it is not complicated by end organ ischaemia. The aortic dissection has led to a cholesterol embolism. This can be diagnosed histopathologically with the finding of cholesterol crystals. The phenomenon where cholesterol is released from an atherosclerotic plaque is called ‘trash foot’. A highly technical medical term. This results in the mottled appearance of distal embolism associated with livedo reticularis. You can search the internet for some case reports of this phenomenon.

214
Q

A 55 year old woman presents with painful joints, a purpuric rash on her arms and legs. Systems review reveals heamoptysis and ear pain. On examination you find black patches on her toes. Investigation A.Arteriogram B.Full blood count C.Anti-neutrophil cytoplasmic antibody D.Venous duplex scan E.CT scan F.Cold provocation test G.Blood cultures H.Blood sugar I.Lumbar puncture

A

Anti-neutrophil cytoplasmic antibody This patient has Wegener’s granulomatosis, a systemic vasculitis affecting small and medium sized vessels. The classic triad includes upper and lower respiratory tract involvement and GN. Musculoskeletal manifestations such as arthralgia and signs of thromboembolism are commonly seen. A positive cANCA (antigen being proteinase 3) in the setting of the classic triad is sufficient to diagnose Wegener’s. Urinalysis and microscopy is also indicated to reveal renal involvement and a CT chest may reveal lung involvement, particularly in those who are asymptomatic for pulmonary involvement. This may show nodules or infiltrates.

215
Q

A 56 year old man on NSAIDs and amoxicillin for bronchitis develops a rash. He is mildy febrile despite the resolution of his bronchitis. The patient is confirmed to be in acute renal failure with elevated urea and creatinine and there is also pedal oedema. He is not oliguric. FBC shows eosinophilia. Diagnosis A.Renal artery stenosis B.Chemotherapy C.Shock D.Acute interstitial nephritis E.SLE F.Polycystic kidney disease G.Normal variant H.Essential hypertension I.Pre-eclampsia J.Diabetic nephropathy K.Obstructive uropathy

A

Acute interstitial nephritis Acute interstitial nephritis classically presents with acute renal failure associated with oliguria and the ‘hypersensitivity triad’ of rash, fever and eosinophilia triggered by a drug. This can commonly be antibiotics, especially beta-lactams, and NSAIDs, though the range of triggering medications is vast. Oliguria can be present in more severe cases. There is inflammation of the renal interstitium, as suggested by the name, and this is likely a hypersensitivity reaction. It can also occur in the setting of a chronic inflammatory disease instead of being drug triggered. It will usually resolve once you stop the offending drug and treatment is supportive, though corticosteroids can be given to dampen the reaction. Most patients recover but have some residual impairment.

216
Q

A 28 year old man presents with haematemesis and a 3 month history of abdominal pain. His BP is 82/41, HR 119 and afebrile. His peripheries feel cool to touch. He is catheterised and you note decreased urine output. Urine and creatinine is elevated and the consultant asks you why his kidneys are compromised. Diagnosis A.Renal artery stenosis B.Chemotherapy C.Shock D.Acute interstitial nephritis E.SLE F.Polycystic kidney disease G.Normal variant H.Essential hypertension I.Pre-eclampsia J.Diabetic nephropathy K.Obstructive uropathy

A

Shock In shock there is inadequate organ perfusion and when this includes the kidneys you get renal hypoperfusion (evidenced by this man’s oliguria) which can progress to acute renal failure. The reason here is hypovolaemia due to loss of intravascular volume from this man’s haematemesis. The cool peripheries (hands and feet) indicate poor peripheral perfusion. There is low BP and reflex tachycardia here too. Treatment here will be with volume replacement with IV saline and to treat the underlying cause, ideally in ICU. Do you know some causes of haematemesis?

217
Q

A 32 year old woman at 34 weeks gestation presents with acute RUQ pain and a frontal headache. There is oliguria and increasing oedema as well as proteinuria of 3+ on urinalysis. Creatinine is elevated. The doctor starts her on labetalol. Diagnosis A.Renal artery stenosis B.Chemotherapy C.Shock D.Acute interstitial nephritis E.SLE F.Polycystic kidney disease G.Normal variant H.Essential hypertension I.Pre-eclampsia J.Diabetic nephropathy K.Obstructive uropathy

A

Pre-eclampsia This is a syndrome characterised by new onset hypertension and proteinuria in pregnant women after 20 weeks gestation. The only definitive treament is to deliver the placenta and a risk-benefit to the mother and baby should be carried out to determine the best clinical step. The doctor starts her on an antihypertensive here so while the question does not directly tell you she is hypertensive, this can be inferred. If the patient doesn’t have either hypertension or proteinuria, then they do not have pre-eclampsia. The woman may be asymptomatic, or like this case, may present with symptoms indicating a more severe disease such as a headache which is usually frontal and upper abdominal pain usually in the RUQ (a clinical symptom of HELLP syndrome which is a subtype of severe disease characterised by Haemolysis, ELevated liver enzymes and Low Platelets). Oedema is very common but quite non-specific as a presentation. Oliguria may be present and again is a sign that the pre-eclampsia is severe. The raised creatinine indicates underlying renal impairment and renal failure can be a rare complication, often occuring as ATN with either sepsis or placental abruption.

218
Q

An 18-year-old develops fever, photophobia and neck stiffness. There is also confusion and vomiting as well as a petechial rash. A CT scan is needed but someone is already in the CT scanner. Management A.Carotid angiography B.Burr hole surgery C.Paracetamol D.Re-breathing E.Positron emission tomogram F.Lumbar puncture G.Thrombolysis H.Oral antibiotics I.CT head J.Visual field testing K.Aspirin L.Intravenous antibiotics

A

Intravenous antibiotics Obviously meningitis and likely meningococcal due to the petechial rash. You all got that bit. Right? Now CT head should be considered before LP if there is any evidence of raised ICP, which we can presume this person here has. An LP will confirm the diagnosis with bacterial meningitis showing a low CSF glucose, elevated CSF protein and positive CSF culture/gram stain or meningococcal antigen. BUT this is clearly a very severe case and an emergency and LP here is going to be delayed. You should start this person on IV broad spectrum antibiotics and consider getting blood culture results.

219
Q

A 17-year-old woman with a FH of headaches has now herself developed headaches which are throbbing in character and preceded by flashing lights. She wonders what her GP can do for her. Management A.Carotid angiography B.Burr hole surgery C.Paracetamol D.Re-breathing E.Positron emission tomogram F.Lumbar puncture G.Thrombolysis H.Oral antibiotics I.CT head J.Visual field testing K.Aspirin L.Intravenous antibiotics

A

Aspirin Migraine is a chronic condition, with genetic determinants, which usually presents in early to mid life. The typical migraine aura this patient describes (which can be visual, sensory or speech symptoms) which can occur during or before the headache, is pathognomic, but is not seen in the majority of patients. The aura can be positive phenomena (for example this patient seeing flashing lights) or negative phenomena (for example visual loss). Nausea, photophobia and disability (the headache gets in the way with the patient’s ability to function) accompanying a headache also suggest a migraine diagnosis. The headache of a migraine tends to be prolonged if untreated, and tends to be unilateral and pounding (but does not have to be). Tests aim to rule out other differentials, although if the history is compatible and neurological examination is unremarkable, further testing is not needed. Treatment of this chronic condition aims at treating acute attacks to restore function. Triptans can be used in specialist care for severe symptoms. These are 5HT1 agonists. Effective initial treatment in a primary care setting can involve NSAIDs, which are available OTC and include aspirin, as well as indometacin and naproxen. Paracetamol is less effective than NSAIDs but is still better than placebo – it is first line in those who are pregnant. Treatment should be taken as soon as a patient realises they are having an attack and may need to be repeated after the attack. A few patients who have frequent, severe or disabling headaches may require daily prophylaxis such as anticonvulsants, TCAs and beta blockers.

220
Q

A 60 year old man has suprapubic pain on standing and difficulty emptying his bladder with blood at the end of the stream. Diagnosis A.Localised prostate cancer B.Advanced prostate cancer C.Hydronephrosis D.Bladder calculus E.Iatrogenic F.Gram negative septicaemia G.Acute prostatitis H.Anal fissure I.Benign prostatic enlargement J.Bacterial cystitis

A

Bladder calculus There is pain on standing and difficulty urinating because the calculus falls and blocks the bladder outflow. Blood comes at the end of the stream from urine that has passed the damaged area of urothelium. Patients tend to present with suprapubic pain, haematuria and obstructive symptoms. Examination may reveal suprapubic tenderness. The first test to order is urinalysis. A non-contrast CT abdomen is also indicated to look for the stone.

221
Q

A 65-year-old man presents with a large painless bladder and overflow incontinence at night and a raised creatinine level. Diagnosis A.Localised prostate cancer B.Advanced prostate cancer C.Hydronephrosis D.Bladder calculus E.Iatrogenic F.Gram negative septicaemia G.Acute prostatitis H.Anal fissure I.Benign prostatic enlargement J.Bacterial cystitis

A

Hydronephrosis This patient has BPH which has caused hydronephrosis. This is an example of bilateral obstructive uropathy. Acute presentations are often painful whereas chronic presentations are more insidious in onset. Blockage of urinary flow by the enlarged prostate has led to urinary retention and overflow incontinence. Initial treatment aims to relieve the pressure on the kidneys. This involves catheterisation as the first line treatment. The patient should be started on alpha blockers at the time of catheterisation.

222
Q

Most appropriate monitoring investigation: Pravastatin A.ECG B.Thyroid function tests C.Echocardiogram D.White cell count E.INR F.Lung function tests G.Liver function tests H.Renal function tests I.Activated partial thromboplastin time J.GCS K.Serum drug level

A

Liver function tests Pravastatin is a statin. Other statins include the mighty atorvastatin, simvastatin and rosuvastatin. They inhibit HMG-CoA reductase, which is an enzyme involved in the cholesterol synthesis pathway. They hence lower LDL cholesterol levels very effectively to reduce the cardiovascular disease. Statins can cause altered LFTs and rarely even hepatitis and jaundice. There have also been rare case reports of liver failure. NICE have a guideline (which we must listen to…) which state that liver enzymes should be measured before treatment and repeated within 3 months and at 12 months of starting treatment (unless there are obvious signs and symptoms of hepatotoxicity).

223
Q

Most appropriate monitoring investigation: Cyclophosphamide A.ECG B.Thyroid function tests C.Echocardiogram D.White cell count E.INR F.Lung function tests G.Liver function tests H.Renal function tests I.Activated partial thromboplastin time J.GCS K.Serum drug level

A

White cell count This drug is used mainly with other drugs for treating malignancies including leukaemias, lymphomas and solid tumours. The rare and serious complication is haemorrhagic cystitis as a result of the urinary metabolite acrolein and mesna can be given as prophylaxis. It is an alkylating agent which damages DNA and interferes with cell replication. The only feasible option on this list is WCC where we have to assume this drug is being used for a leukaemia or lymphoma and we want to measure whether it is working.

224
Q

Most appropriate monitoring investigation: Phenytoin A.ECG B.Thyroid function tests C.Echocardiogram D.White cell count E.INR F.Lung function tests G.Liver function tests H.Renal function tests I.Activated partial thromboplastin time J.GCS K.Serum drug level

A

Serum drug level Phenytoin is useful for tonic-clonic and focal seizures but it has a narrow therapeutic index and a non-linear dose-plasma concentration relationship. Hence, small dose increases can lead to huge increases in plasma concentration and acute toxicity. Conversely if the patient skips a dose, there may also be marked change and the levels may be sub-therapeutic. You therefore need to monitor plasma drug concentration to improve the adjustment of dose. Parenterally, you can also consider fosphenytoin.

225
Q

A 20-year-old medical student is concerned about recent weight gain, lethargy and constipation. You notice bradycardia and a goitre. Her ankle reflexes are sluggish. Her periods have stopped. Diagnosis A.Graves disease B.Hodgkin’s disease C.Pancreatic carcinoma D.Euthyroid goitre E.Thyroid cancer F.TB abscess G.De Quervain’s thyroiditis H.Thyroglossal cyst I.Hashimoto’s thyroiditis J.Carotid artery aneurysm K.Superior vena cava syndrome L.Myxoedema

A

Myxoedema This medical student has hypothyroidism. Worldwide, the most common cause is iodine deficiency. Other causes include Hashimoto’s or secondary and tertiary hypothyroidism. It can also result from viral de Quervain’s thyroiditis or postpartum thyroiditis. Symptoms include those mentioned (weight gain, lethargy, sluggish reflexes, bradycardia and constipation) as well as depression, fatigue, constipation, cold intolerance, menstrual problems in females, dry skin and muscle cramps. Diagnosis is based on measurement of TSH and thyroid hormones. Treatment is by replacement of T4 with or without T3 in combination. If the patient has normal T3 and T4 but mildly elevated TSH, this is described as subclinical hypothyroidism.

226
Q

A 50-year-old man presents with acute ST elevation MI to London’s best hospital. He has already been given aspirin, oxygen, morphine and GTN and is haemodynamically stable. 20 minutes have passed since symptom onset. Management A.Add Atenolol B.Add Omega-3 oils C.Add low molecular weight Heparin D.Perform echocardiography E.Advise lifestyle measures F.Perform exercise ECG G.Add Aspirin H.Perform coronary thrombolysis I.Add clopidogrel J.Add an ACE inhibitor K.Perform CABG L.Perform coronary angioplasty M.Add Atorvastatin

A

Perform coronary angioplasty. For confirmed ST elevation MI the 1st line treatment which gives the best results is primary PCI with stenting and is indicated if the person presents to A&E within 90 minutes of first presentation. CABG should be strongly considered if the patient fails PCI and should be done within 12 hours of onset of symptoms, ideally within 6. This is London’s best hospital but if this were Orkney and they did not have PCI capacity or the ability to transfer to a PCI facility within 30 minutes or so, then you would thrombolyse if there are no contraindications. This must be done within 12 hours of symptom onset and ideally within 3 hours as the efficacy of fibrinolytics diminishes over time. You should be aware of the absolute contraindications to thrombolysis such as suspected aortic dissection and prior intracranial haemorrhage. This patient has already been given some treatment for suspected MI (aspirin, oxygen, morphine and GTN). Post-treatment this patient will need aspirin therapy and clopidogrel for at least a year – aspirin should be continued indefinitely. Patients should also be started on a beta blocker, ACE inhibitor and a statin, indefinitely.

227
Q

A 55-year-old woman with a 6-week history of intermittent angina is found to have a serum cholesterol of 7.9 mmol/l. Management A.Add Atenolol B.Add Omega-3 oils C.Add low molecular weight Heparin D.Perform echocardiography E.Advise lifestyle measures F.Perform exercise ECG G.Add Aspirin H.Perform coronary thrombolysis I.Add clopidogrel J.Add an ACE inhibitor K.Perform CABG L.Perform coronary angioplasty M.Add Atorvastatin

A

Exercise ECG This patient has presented with stable angina. Resting ECG is often normal however during exercise stress ECG (most often the Bruce Protocol) there will be ST segment depression during exercise indicative of ischaemia. Those unable to exercise to an adequate level may need stress myocardial perfusion imaging or stress echocardiography. 1st line treatment involves lifestyle changes and antiplatelet therapy with aspirin. Anti-anginal theray will also be given, first line being beta-blockade. Statin therapy, blood sugar control in diabetics and BP control with antihypertensives may also be necessary. Those with LMS disease, 3 vessel disease or a reduced EF may benefit from CABG. Single vessel disease may benefit from PCI.

228
Q

A 30-year-old teacher presents to her GP with a cough and shortness of breath, worsening over 48 hours following a ‘flu-like illness. On examination, she has a temperature of 39ºC. There is little to find on chest examination, but the chest x-ray shows bilateral shadowing. Management A.Cough syrup B.Steroid inhaler C.Salbutamol inhaler D.Diuretic E.Fluids and bed rest F.Nebulised salbutamol G.IV cefuroxime H.Oral clarithromycin I.Oral penicillin V J.Opiate

A

Oral clarithromycin This is a CAP which is confirmed by CXR shadowing, which would show airspace shadowing with air bronchograms. The history is also consistent with respiratory symptoms and pyrexia. The most sensitive test for CAP is a CXR. The treatment is initially empirical with antibiotics and management would be guided by this patient’s CURB-65 score. A macrolide is the first choice in adults with no contra-indications without a recent cours eof antibiotics or risk of drug resistance, but always check local prescribing policies. Azithromycin, clarithromycin or erythromycin are all valid choices here.

229
Q

A 40-year-old male smoker with COPD is in hospital with a 2 week cough productive of green sputum with SOB for 24 hours. Temperature is 39.3ºC, RR 35/min, and HR 120/min. There is dullness to percussion and reduced breath sounds at the left base. He is given nebulised salbutamol and systemic corticosteroids. Management A.Cough syrup B.Steroid inhaler C.Salbutamol inhaler D.Diuretic E.Fluids and bed rest F.Nebulised salbutamol G.IV cefuroxime H.Oral clarithromycin I.Oral penicillin V J.Opiate

A

IV cefuroxime This is an acute infective exacerbation of COPD and the patient has already received nebulised salbutamol and corticosteroids. The next important thing to do is to give IV antibiotics. Inpatient therapy for severe infections can be with IV ceftriaxone with azithromycin. High risk patients should receive piperacillin and tazobactam, or meropenem, to cover for pseudomonas.

230
Q

A 60-year-old man has been diagnosed as having bronchial carcinoma with secondaries. He is distressed by a chronic cough. Management A.Cough syrup B.Steroid inhaler C.Salbutamol inhaler D.Diuretic E.Fluids and bed rest F.Nebulised salbutamol G.IV cefuroxime H.Oral clarithromycin I.Oral penicillin V J.Opiate

A

Opiates Opiates are commonly prescribed for analgesia and can also be used as an anti-tussive in this case (suppresses ACh and NK release needed for cough activation and inteferes with serotonin receptors in the DRN). They are also abused for their euphoric effects mediated by their action on dopamine release at the nucleus accumbens. They have a host of side effects: GI effects (receptors present on the myenteric and submucosal plexus) of decreased gastric motility, emptying and increased gut water reabsorption… in short, constipation; Respiratory depression (desensitises central chemoreceptors to lessen the response to PaCO2); N&V by stimulating the CTZ; Pupillary constriction via the Edinger-Westphal nucleus; And causes symptoms like pruritis, urticaria, hypotension through histamine release via a direct effect on mast cells.

231
Q

A 64-year-old, previously obese woman complains of weight loss, despite increased appetite and says she has been treating herself for ‘repeated urine infections’. She also says that over the last few months she has been getting pins and needles in her legs. Investigation A.Bone marrow aspirate B.Thyroid function tests C.Full blood count and ESR D.Fasting blood glucose E.Blood test for auto-antibodies F.Brain scan G.Bronchoscopy H.History only I.Gastroscopy J.Chest x-ray K.Colonoscopy L.HIV antibody test M.Abdominal CT

A

Fasting blood glucose T2DM can present with yeast, skin and urinary tract infections on top of unintentional weight loss and fatigue. The pins and needles in her legs is a result of diabetic peripheral sensory neuropathy. This is a microvascular complication of DM and is characterised by peripheral nerve dysfunction. There tends to be loss of sensation typically occuring in a symmetrical ‘glove and stocking’ distribution. Patient’s may also describe a pain or paraesthesia. Examination should include peripheral pulses, reflexes and sensation to light touch, vibration (128Hz tuning fork), pinprick and proprioception. Any pain can be treated with medications like gabapentin. Symptomatic patients need a single random blood glucose of >11.1 or single fasting glucose of >7. Asymptomatic patients need two separate elevated readings for a diagnosis. Alternatively if there are borderline results, an OGTT can be conducted to see if plasma glucose is raised >11.1 two hours after an oral glucose load of 75g. A patient is said to have impaired fasting glucose if fasting glucose falls between 6.1-6.9. Impaired glucose tolerance is present if plasma glucose 2 hours after oral glucose load in OGTT falls between 7.8-11.0.

232
Q

A 32-year-old male nurse from Zimbabwe came into the UK as part of an NHS recruitment campaign. He presents with SOB, weight loss, general debility, cervical lymphadenopathy and purple tender lesions on his shin of 6 weeks duration. Investigation A.Bone marrow aspirate B.Thyroid function tests C.Full blood count and ESR D.Fasting blood glucose E.Blood test for auto-antibodies F.Brain scan G.Bronchoscopy H.History only I.Gastroscopy J.Chest x-ray K.Colonoscopy L.HIV antibody test M.Abdominal CT

A

CXR Sarcoidosis is a chronic multisystem disease with an unknown aetiology with the lungs most commonly affected (giving SOB in this case). The purple lesions are erythema nodosum and are tender erythematous nodules. Lupus pernio is another typical skin manifestation of sarcoidosis presenting with indurated plaques with discoloration on the face. CXR will typically show bilateral hilar lymphadenopathy and CXR findings are used in the staging of disease. Additionally, serum calcium and ACE levels may be raised. A transbronchial biopsy is essential for diagnosis in most cases and shows the presence of non-caseating granulomas. Black people have a higher lifetime risk of sarcoidosis, as do those of Scandinavian origin. The mainstay of treatment for severe disease involves systemic corticosteroids.

233
Q

A 78-year-old man presents with low back pain that doesn’t improve with rest. He has non-specific intermittent abdominal pain and night sweats. Investigations reveal a haemoglobin of 9.6 g/dL, a corrected calcium of 2.97 mmol/L, and an ESR > 100 mm/hr. Investigation A.Bone marrow aspirate B.Thyroid function tests C.Full blood count and ESR D.Fasting blood glucose E.Blood test for auto-antibodies F.Brain scan G.Bronchoscopy H.History only I.Gastroscopy J.Chest x-ray K.Colonoscopy L.HIV antibody test M.Abdominal CT

A

Bone marrow aspirate This patient has multiple myeloma. This is characteristed by clonal proliferation of plasma cells in BM and commonly presents with bony pain and symptoms of anaemia (which this patient’s Hb demonstrates). The elevated ESR is also suggestive and hypercalcaemia is present in 30%. The diagnostic test is serum or urine electrophoresis looking for a paraprotein spike of IgG or IgA and light chain urinary excretion (Bence Jones proteins). Bone marrow examination and skeletal survey will also need to be conducted. Bone marrow aspirate in this case and biopsy will show plasma cell infiltration in the bone marrow, and can help to differentiate multiple myeloma from MGUS and solitary plasmacytoma. Bone changes include osteopaenia, osteolytic lesions and fractures.

234
Q

A 64 year old lady is admitted with severe pneumonia with consolidation seen on her CXR. While on the wards she develops a warm, erythematous, tender and oedematous left leg. A few days later her breathing which was improving with antibiotic treatment, suddenly deteriorates. Which is the LEAST useful in the above? A) Pulse B) BP C) CTPA D) V/Q scan E) ABG

A

V/Q scan This patient has dual pathology, with resolving pneumonia and a PE secondary to a DVT. The least useful investigation in this pt. is a V/Q scan as the underlying pneumonia would make interpretation of the V/Q scan very difficult. Pulse, BP and ABG are essential for ensuring the patient is stable and remains so. CTPA would be the best method to image the lungs.

235
Q

A 78 year old woman develops left sided CP and tingling. This is the first episode of this type of pain and is not associated with SOB, dizziness, N or V. She is a lifelong smoker and takes simvastatin for hypercholesterolaemia. On placing the ECG electrodes on the chest you notice a band like area of erythema on the left side of her chest. The ECG shows LV hypertrophy and non-specific T wave changes. Which is the best management? A) Start anti-viral therapy B) Admit the pt to the CCU C) Give ibuprofen for the pain D) Measure troponin E) Perform a TTE.

A

Start anti-viral therapy The history in the scenario tries to trick you into thinking that there might be a cardiac cause, but the pain, tingling and the band-like erythema is more consistent with shingles caused by VZV. The best treatment is therefore anti-viral therapy.

236
Q

A 55 year old patient it referred by his GP to the General Surgery clinic at the local hospital because of a left varicoele and haematuria. O/E the patient has an abdominal mass on the left hand side. A CXR shows multiple coin lesions in both lung fields. Diagnosis? A) Transitional cell carcinoma B) Renal cell carcinoma C) Seminoma D) Prostate carcinoma E) Teratoma

A

Renal cell carcinoma The constellation of signs and symptoms of an abdominal mass, haematuria and left varicoele indicate an underlying renal call carcinoma. A left varicoele occurs in males due to compression of infiltration of the left testicular vein, which drains into the left renal vein. On the right it drains straight into the IVC. RCC has a well known association with paraneoplastic syndromes especially hypercalcemia via ectopic PTH and polycythemia via excessive EPO production. Cannon-ball pulmonary mets are a characteristic feature of RCC and testicular semimoma.

237
Q

A 45 year old patient presents to you with a few weeks’ history of odynophagia and dysphagia. O/E he is noted to have white plaques at the back of this throat although no ulcers can be seen. On endoscopy these white plaques extend into the oesophagus. He does not take an prescription drugs. There is nothing of note in his PMH.

Most useful test to diagnose underlying pathology?

A) Monospot test

B) OGTT

C) Blood tests

D) Viral markers

E) Bacterial MC+S

A

Viral markers

The patient is suffering from oral and oesphageal candidiasis, with the most likely cause in this patient being underlying immunosuppression. Oesophageal candidiasis is an AIDS-defining condition, which necessitates careful counselling, risk factor analysis and testing for HIV.

238
Q

A young child brought in to you by her worried mother with an erythematous rash on her cheeks but is otherwise well. O/E she has a slapped cheek appearance.

Most likely organism?

A) Human herpesvirus 6

B) Parvovirus B19

C) S. aureus

D) B-haemolytic Strep

E) Measles

A

Parvovirus B19

This presentation, also known as ‘fifth disease’ and ‘erythema infectiosum’ is caused by parvovirus B19. Constitutional symptoms are usually mild and patients usually present with arthralgia. In patients with sickle cell disease and hereditary spherocytosis, parvovirus B19 can lead to aplastic crises due to arrest of RBC production and if women are affected early in the course of pregnancy there is a small but definite risk of miscarriage.

239
Q

A 20 year old male athlete presents with increasing back pain and morning stiffness which lasts for a few hours. He also reports that he has been having considerable difficulty reaching his toes when warming up for his exercises.

Which is true?

A) The disease is associated with HLA-B21

B) The disease typically shows evidence of an IgM Ab directed against the Fc portion of Igs

C) The disease is associated with valvular heart disease

D) Patients have an higher risk of developing AL amyloidosis

E) Infliximab has been shown to be ineffective in the treatment of the disease.

A

The disease is associated with valvular heart disease

This history is consistent with ankylosing spondylitis, which is a seronegative arthropathy associated with HLA-B27. Males in ther late teens are most commonly affected, suffering from back pain, morning stiffness and spinal ankylosis, which eventually leads to the characteristic question mark posture. Patients with AS are at an increased risk of developing valvular heart defects and secondary amyloidosis (AA amyloid) due to chronic inflammation. They are 1.5-2x more likely to die compared to a control population. Infliximab is a promising new agent that can be used in the treatment of AS.

240
Q

An 84-year-old lady with moderate impairment of left ventricular ejection fraction. She has recently undergone hip replacement and has a positive 4 litre fluid balance in the last 2 days.

Which of the following JVP appearances is most likely to be seen in the clinical pictures described?

a. Raised JVP, normal waveform b. JVP rising with inspiration
c. Large ‘a’ wave
d. Absent ‘a’ wave

e. Large ‘v’ wave
f. Cannon ‘a’ wave
g. Elevated JVP but no waveform, or change with breathing h. Slow ‘y’ descent

A

Raised JVP, normal waveform.

This is most likely simple fluid overload and in the presence of reasonable renal function will respond to appropriate therapy with IV diuretics. The JVP will be elevated as the right side of the heart fails to clear the backload of fluid returning form the venous system.

241
Q

An 84 year old farmer is admitted to the acute medical receiving unit with collapse. His pulse is very slow and irregular. His blood pressure is 62 mmHg systolic and he is sweaty. There are intermittent irregular visible pulsations noted on inspection of his venous pressure.

Which of the following JVP appearances is most likely to be seen in the clinical pictures described?

a. Raised JVP, normal waveform

b. JVP rising with inspiration
c. Large ‘a’ wave
d. Absent ‘a’ wave

e. Large ‘v’ wave
f. Cannon ‘a’ wave
g. Elevated JVP but no waveform, or change with breathing

h. Slow ‘y’ descent

A

Cannon ‘a’ wave.

Patients with complete AV block typically experience bradycardia (as low as 28 beats per minute during sleep), hypotension, and as in this case, hemodynamic instability. In this case the cannon ’a’ wave is caused by the lack of synchronization between the atria and the ventricles. The ventricles occasionally contract against a closed AV valve, giving rise to the JVP appearance. A temporary wire is required as he is haemodynamically unstable.

242
Q

A 45-year-old lady is an inpatient in the oncology department with metastatic melanoma. She has received a recent course of chemotherapy, but has developed acute shortness of breath and is hypotensive. Bedside echocardiogram reveals a 2.5cm circumferential pericardial effusion.

Which of the following JVP appearances is most likely to be seen in the clinical pictures described?

a. Raised JVP, normal waveform

b. JVP rising with inspiration
c. Large ‘a’ wave
d. Absent ‘a’ wave

e. Large ‘v’ wave
f. Cannon ‘a’ wave
g. Elevated JVP but no waveform, or change with breathing

h. Slow ‘y’ descent

A

JVP rising with inspiration. This is called Kassmaul’s sign, or “paradoxical” rise in the JVP with inspiration. Normally the venous pressure (and the JVP) falls with a deep breath as intrathoracic venous return rises (this is why murmurs on the right side of the heart are louder with inspiration). If the JVP rises with inspiration it is ‘paradoxical’ and signifies that an extrinsic force, in this case, an effusion, compresses the ventricle. Treatment is peri-cardiocentesis. Effusions are very common in malignant disorders, but easily treatable.

243
Q

A 33-year-old lady with Tetalogy of Fallot (TOF) attends for annual review. She had corrective surgery as a young child, with closure of the VSD and placement of a pulmonary homograft. She keeps well. On inspection of her ECG there are tall R waves in leads V1-V4 and a deep s wave in lead I.

what is the most likely diagnosis from the ECG description provided?

a. Acute inferior myocardial infarction

b. Atrial fibrillation
c. Pericarditis
d. Anterior myocardial infarction

e. Complete AV block
f. Right Ventricular Hypertrophy

g. Ventricular Tachycardia
h. Supraventricular Tachycardia

A

F. Right Ventricular Hypertrophy.

Patients with TOF almost always have RVH (Tetralogy of Fallot corresponds to 1. VSD; 2. Over-riding aorta; 3. Pulmonary stenosis; and 4. RVH).

244
Q

A 64 year old lady with a history of rheumatic heart disease attends out-patients. She is thin, has a malar flush and has had a St Judes mitral valve replacement 3 years previously. She is now very short of breath when before this wasn’t a problem. ECG does not show any discernable p wave and the tracing has a ‘saw-tooth’ appearance. The QRS complex duration is 160ms. There is a negative deflection in lead V1.

what is the most likely diagnosis from the ECG description provided?

a. Acute inferior myocardial infarction

b. Atrial fibrillation
c. Pericarditis
d. Anterior myocardial infarction

e. Complete AV block
f. Right Ventricular Hypertrophy

g. Ventricular Tachycardia
h. Supraventricular Tachycardia

A

. Atrial fibrillation.

Almost all patients with rheumatic mitral valvular disease develop AF at some point. The saw tooth nature of this ECG would suggest flutter although it is hard to be definitive until you see the ECG. The negative deflection in lead V1 also suggests left bundle branch block, which is very common following surgery.

245
Q

A 64-year-old man has just been discharged following percutaneous coronary intervention. He was treated with drug-eluting stents. What combination of drugs must he remain on for at least a year?

what is the best treatment strategy?

a. Dual anti-platelet therapy
b. Dual anti-platelet therapy, low molecular heparin

c. Warfarin
d. Thrombolytic therapy
e. Long acting nitrate preparation
f. Non-steroidal anti-inflammatory preparations
g. Primary percutaneous coronary intervention
h. GTN spray

A

A. Aspirin and clopidogrel (dual anti-platelet therapy).

Following the use of drug-eluting stents there is now clear evidence from a number of trials that patients should remain on both aspirin and clopidogrel for one year, and then to switch to aspirin alone. This is because of the risk of sub acute stent thrombosis. Typically there is a 1% risk of acute stent thrombosis in the first 6 weeks. There is about a 10% risk of instent re-stenosis over the next one year

246
Q

A 24 year old lady has just been discharged following successful percutaneous closure of an ASD with an amplatzer device. She complains of palpitations and ECG confirms AF. Subsequent 24 hour tape confirms two very short-lived episodes of AF.

what is the best treatment strategy?

a. Dual anti-platelet therapy
b. Dual anti-platelet therapy, low molecular heparin

c. Warfarin
d. Thrombolytic therapy
e. Long acting nitrate preparation
f. Non-steroidal anti-inflammatory preparations
g. Primary percutaneous coronary intervention
h. GTN spray

A

A. Aspirin and clopidogrel.

The AF is short-lived and tends to happen after a device is placed in the atrial septum. In this case aspirin and plavix are used for 6 moths until the device has endothelialised and then stopped. If the AF was permanent then warfarin and electrical cardioversion should be considered.

247
Q

A 56-year-old gentleman has recently recovered from a large anterolateral infarct. He presents to A&E with heaviness in his chest and is clod and clammy. ECG demonstrates a wide-complex tachycardia. There is concordance across all leads. The rate is 170 bpm. Blood pressure is maintained at 120/80 mmHg.

For each of the arrythmias described what would be your first line treatment strategy?

a. Lignocaine
b. Cardioversion
c. Adenosine
d. Amiodarone
e. Digoxin
f. Metoprolol
g. Warfarin
h. Magnesium sulphate

A

A. Lignocaine.

This is Ventricular Tachycardia (VT). Because the blood pressure is maintained lignocaine is a good first line agent. It is a membrane stabiliser and is used effectively for VT. If the patient were to become haemodynamically unstable he should be cardioverted.

248
Q

A 72-year-old lady has just undergone a laparoscopy for acute cholecystitis. She is pyrexic, although white cell count and CRP is normal. She has developed atrial fibrillation.

For each of the arrythmias described what would be your first line treatment strategy?

a. Lignocaine
b. Cardioversion
c. Adenosine
d. Amiodarone
e. Digoxin
f. Metoprolol
g. Warfarin
h. Magnesium sulphate

A

D. Amiodarone.

This is post-operative AF and chemical cardioversion with amiodarone is a reasonable option. She almost certainly is on IV antibiotics, as infection may be the precipitating factor causing the AF, as opposed to any underlying cardiac abnormality.

249
Q

A 21-year-old girl with trisomy 21 and arterial saturations of 81% at rest. Echocardiogram shows bidirectional flow at the upper end of the septum involving the mitral valve, which is also leaking. What is the diagnosis?

For each of the clinical scenarios and corresponding congenital cardiac anatomical defects described below, what is the correct term?

a. Tetralogy of fallot
b. Congenital correction of transposition of the great arteries

c. Atrioventricular septal defect
d. Ebsteins anomaly of the tricuspid valve
e. Pulmonary stenosis
f. Ventircular septal defect
g. Atrial Septal defect
h. Patent Foramen Ovale

A

C-Atrioventricular septal defect.

The anatomy described is AVSD, which is commonly seen in T21. This girl unfortunately has not had this defect repaired and thus has developed Eisenmenger complex. This arises when there is sustained left to right flow for many years which leads to chronically high right sided pressures.

250
Q

A 41-year-old man with O2 saturations of 78% on room air and is increasingly SOB. There is a loud pan-systolic murmur and a pulmonary ejection systolic murmur. Echocardiogram demonstrates the left ventricle is anterior and supplies the pulmonary circulation. There is also a VSD. What is the diagnosis?

For each of the clinical scenarios and corresponding congenital cardiac anatomical defects described below, what is the correct term?

a. Tetralogy of fallot
b. Congenital correction of transposition of the great arteries

c. Atrioventricular septal defect
d. Ebsteins anomaly of the tricuspid valve
e. Pulmonary stenosis
f. Ventircular septal defect
g. Atrial Septal defect
h. Patent Foramen Ovale

A

B-Congenital correction of transposition of the great arteries (CCTGA).

Transposition of the great vessels means the aorta arises from the RV and the pulmonary arteries arise from the LV. It is not compatible with life unless there is a co-existent septal defect to allow blood to mix. In CCTGA the ventricles are anatomically switched. There is an anterior LV which gives rise to the pulmonary circulation and the RV is the systemic ventricle. It is often associated with pulmonary stenosis and VSD.

251
Q

A 60-year-old lady presents with acute shortness of breath. Her oxygen level (PaO2) is low at 6.1. CXR reveals unilateral pulmonary oedema and there is left bundle branch block on the ECG. Coronary angiography had been performed 2 months previously and was known to be normal. BP is 210/110 mmHg. What is the most likely diagnosis?

For each clinical scenario described what is the most likely cause of the hypertension.

a. Cushings syndrome

b. Acromegaly
c. Renal artery stenosis

d. Phaeochromcytoma
e. Coarctation of aorta
f. Type 1 diabetes
g. Essential hypertension
h. Sub-dural haematoma

A

C-Renal artery stenosis.

This is a rare cause of hypertension and can rarely give rise to unilateral pulmonary oedema. The exact mechanism for this is not known. Diagnosis is by ultrasound or magnetic resonance imaging of the renal vessels.

252
Q

A 26 year old girl presents 18 weeks pregnant. She has been increasingly SOB. It emerges she was told she had a heart murmur as a child. You examine her and note a systolic murmur radiating through to the back and the femoral pulses are very difficult to feel. What is the diagnosis?

a. Cushings syndrome

b. Acromegaly
c. Renal artery stenosis

d. Phaeochromcytoma
e. Coarctation of aorta
f. Type 1 diabetes
g. Essential hypertension
h. Sub-dural haematoma

A

. Coarctation of aorta-

mild coarctation may often be asymptomatic. However, the weak distal pulses suggest this as a diagnosis. She will need an echocardiogram and will require surgical correction following delivery.

253
Q

A 72 year-old man becomes hypotensive 2 hours after apparently successful thrombolysis for an inferior myocardial infarction. A right praecordial lead ECG shows normal ST segments in V4R to V6R. On examination there is a new systolic murmur and the LV apex is thrusting. There is florid pulmonary oedema.

a. Pulmonary embolism
b. Inferior myocardial infarction

c. Cardiac tamponade
d. Left ventricular failure
e. Aortic dissection
f. Severe mitral regurgitation
g. Severe aortic stenosis
h. Right ventricular infarction
i. Hypertrophic cardiomyopathy

j. Coarctation of the aorta

A

Severe mitral regurgitation

Mechanical complications of myocardial infarction include papillary muscle infarction (resulting in severe mitral regurgitation) and VSD which can both cause hypotension. The findings of a normal RV lead ECG, a systolic murmur and volume overloaded apex make acute MR the most likely diagnosis although an echocardiogram would need to be performed to exclude the formation of a VSD which tends to occur with anterior myocardial infarctions. Acute MR results in severe pulmonary oedema.

254
Q

A patient presents with heart failure following a number of myocardial infarcts. Their symptoms are difficult to control and you prescribe carvedilol, furosemide, perindopril, spironolactne and metolozone. They are also on warfarin therapy as the ejection fraction is less than 20% on echo. Following discharge they present with profound hypokalaemia.

Which drug would you withdraw first?

A. carvedilol
B. furosemide
C. perindopril
D. spironolactone

E. metolozone

A

e. metolozone.
Metolozone is a potent thiazide diuretic and thus can cause profound hypokalaemia. Furosemide can also cause low potassium but is not as strong. Carvedilol, perindopril and spironolactone all have a tendency to raise serum potassium and should be continued.

255
Q

A 47-year-old lady with a background history of scleroderma presents with progressive shortness of breath. Her lungs are clear but there is a pan-systolic murmur at the lower left sternal border. CXR and CT chest are normal. Echocardiogram reveals severe tricuspid incompetence and an estimated pulmonary artery pressure of 89mmHg.

Which therapeutic modality would be the first line agent of choice in this case?

  • sildenafil
  • carvedilol
  • salbutamol
  • bosentan
  • inhaled prostacyclin
A

Sildenafil.
This is primary pulmonary hypertension, a progressive and serious condition which often requires treatment. It is related to autoimmune rheumatological disorders. In this case first line treament would be sildenafil which promotes pulmonary vasodilatation and thus reduces PA pressure. If this fails then bosentan (an endothelin receptor antagonist) and prostacyclin may be of benefit.

256
Q

A 62-year-old gentleman with a history of surgical correction of a aortic coarctation in childhood, presents at the clinic progressively SOB. There is an ejection systolic murmur heard best over the aortic area. You diagnose aortic stenosis.

What is the most likely aetiology of the aortic stenosis?

  • Aortic sclerosis
  • Age degeneration
  • Post surgery to correct coarctation
  • Bicuspid aortic valve
  • Post infective endocarditis
A

Bicuspid aortic valve
Bicuspid aortic valve is often associated with coarctation and often leads to aortic stenosis requiring surgery later in life.

257
Q

A patient becomes hypotensive following thrombolysis for an inferior myocardial infarction. Right ventricular infarction is suspected. What is the most appropriate initial treatment?

a) IV nitrates
b) IV dobutamine

c) IV noradrenaline
d) IV fluids
e) Insertion of a temporary pacemaker wire

A

IV fluids

The aim of treatment in patients with haemodynamically significant right ventricular infarction is to increase the right-sided stroke volume. If there is no evidence of associated left ventricular dysfunction, patients should be given several litres of IV normal saline rapidly. Drugs that reduce the pre-load, such as nitrates, opiates and diuretics, should be avoided. If volume loading does not improve the blood pressure, then inotropes can be considered.

In patients with left ventricular dysfunction, reducing the afterload will help forward flow from the right ventricle, therefore, vasoconstrictors such as noradrenaline should be avoided where possible. Dobutamine can augment forward flow and the cardiac output through its inotropic and chronotropic effects upon the left ventricle.

An infarcted, poorly functioning right ventricle can have its stroke volume significantly compromised by concurrent bradyarrhythmias in which case temporary pacing is indicated.

258
Q

MCQ

Features of severe aortic stenosis include:

  • Collapsing pulse
  • S4
  • LBBB
  • Plateau pulse
  • Displaced apex beat
A

S4 + LBBB + Plateau pulse

Severe aortic stenosis is associated with a plateau, slow rising pulse. The stiff ventricle contracting may cause S4 to be heard. LBBB can arise due to conduction disease. The apex is usually heaving and undisplaced.

259
Q

MCQ

Features of aortic coarctation include:

  • diastolic murmur
  • diastolic tail on echocardiogram of the descending aorta
  • radio-femoral delay
  • rib-notching
  • kyphoscoliosis
A

Diastolic tail on echocardiogram of the descending aorta + radio-femoral delay + rib-notching

It is typically a systolic murmur that is heard best at the back in the inter-scapular space. Rib- notching is because of the formation of collateral vessels. Kyphoscoliosis is more likely associated with Marfan’s syndrome.

260
Q

Which one of the following is associated with hypoparathyroidism?

  • Coeliac disease
  • Cystic fibrosis
  • DiGeorge syndrome
  • CREST syndrome
  • Crigler Najjar syndrome
A

C: DiGeorge syndrome is a familial condition, characterised by hypoparathyroidism associated with intellectual impairment, cataracts and calcified basal ganglia.

261
Q

Which of the following combination of biochemical disturbances is the classical finding in a patient with untreated Addison’s disease?

  • Hypernatraemia, hypokalaemia, hypoglycaemia
  • Hyponatraemia, hyperkalaemia, hypoglycaemia
  • Hyponatraemia, hypokalaemia, hypoglycaemia
  • Hypernatraemia, hypokalaemia, hyperglycaemia
  • Hypernatraema, hyperkalaemia, hyperglycaemia
A

B. Hypoadrenalism classically causes hyponatraemia, hyperkalaemia and hypoglycaemia.

Mineralocorticoid deficiency has its predominant effect on the extracellular balance of sodium and potassium in the distal tubule of the kidney. Glucocorticoid deficiency has a major effect on carbohydrate metabolism, with associated hypoglycaemia.

262
Q

A 23yr old female presents with a 2month history of hirsutism, acne and deepening of her voice. She has a markedly elevated testosterone.

What is the likely aetiology?

  • Polycystic ovarian syndrome
  • Adrenal tumour
  • Prolactinoma
  • Carcinoid syndrome
  • Wilson’s syndrome
A

B. Adrenal tumour.
The short duration of her symptoms, and the substantially raised testosterone would make one suspicious of a virilising tumour. Serum testosterone may be elevated in PCOS, but not markedly so, and the symptoms would usually be of longer duration.

263
Q

MCQ

A 22yr old girl presents with hirsutism and oligomenorrhoea. She has a history of acne. Her BMI is 30. You think she likely has polycystic ovarian syndrome (PCOS).
Which of the following may be used in the management of PCOS?

  • weight loss
  • topical eflornithine
  • sulphonylurea
  • loop diuretic
  • dianette
A

A, B, E-

Topical eflornithine is used to treat facial hirsutism. The anti-androgen, cyproterone acetate competitively inhibits androgen production at peripheral receptors and may also reduce androgen synthesis. This is found in combined preparation in the oral contraceptive Dianette.

264
Q

A 32-year-old woman presents with intermittent headaches lasting around 24 hours, associated with nausea and sensitivity to movement, light and sound. The frequency of the headaches is four or five per month. What is the best prophylactic treatment to give her to try and prevent the headaches?

A. Sumatriptan

B. Paracetamol

C. Verapamil

D. Ibuprofen

E. Propranolol

A

E. Propranolol
Management of migraine involves treatment of the acute episode and prophylactic treatment in patients with regular headaches (usually greater than 3 per month). Acute treatment is with simple analgesia (and antiemetic if necessary) initially, but some patients may benefit from using a triptan. A number of different drugs can be used for prophylactic treatment including propranolol, amitriptyline, sodium valproate and topiramate.

265
Q

MCQ

A 32 year-old man is known to have complex partial seizures consisting of brief episodes of altered consciousness associated with olfactory hallucinations, episodes of déjà vu and a rising epigastric sensation. Which of the following drugs may be helpful in controlling his seizures?

  • Carbamazepine
  • Sodium valproate
  • Propranolol
  • Amitriptyline
  • Lamotrigine
A

A, B and E are commonly used antiepileptic medications which may be used in complex partial seizures. Carbmazepine is commonly used as a first-line medication in patients with partial seizures but other drugs including sodium valproate and lamotrigine can be used as well. C and D are not antiepileptic medications. Propranolol and amitriptyline are both commonly used for migraine prophylaxis.

266
Q

MCQ

Which of the following disorders may cause an autonomic neuropathy?

  • Guillain-Barre syndrome
  • Diabetes
  • Charcot-Marie-Tooth disease
  • Amyloidosis
  • Sarcoidosis
A

A, B and D may cause an autonomic neuropathy. These are less common than other neuropathies but can occur in a number of different conditions, particularly diabetes, amyloidosis and Guillain- Barre syndrome. Patients with autonomic neuropathies may have a wide variety of symptoms including cardiovascular problems e.g. postural hypotension, gastrointestinal problems e.g. diarrhoea or constipation, erectile dysfunction, abnormal sweating and urinary dysfunction.

267
Q

A 67-year-old woman had a 3 month history of weakness of his left leg and a 1 month history of weakness in both arms. He felt things were getting progressively worse. On examination he had wasting and fasciculations in all four arms. He was weak in all four limbs with brisk reflexes throughout with upgoing plantars. He had a normal sensory examination. What is the most likely diagnosis?

  • Polymyositis
  • Parkinson’s disease
  • Multiple sclerosis
  • Motor neurone disease
  • Myasthenia gravis
A

D. Motor neurone disease

He has progressive weakness with a mix of upper and lower motor neurone signs on examination in the presence of a normal sensory exam. This is consistent with motor neurone disease. Parkinson’s disease does not cause weakness. Multiple sclerosis is a central nervous system disorder and therefore associated with upper motor neurone signs. Myasthenia gravis is a neuromuscular junction disorder and polymyositis is a muscle disorder, neither of which would be associated with pathologically brisk reflexes (an upper motor neurone sign).

268
Q

MCQ

A 67-year-old man has motor neurone disease. Which of the following features would be seen in a pseudobulbar palsy?

  • Tongue wasting and fasciculations
  • Dysphagia
  • Brisk jaw jerk
  • Spastic tongue
  • Absent gag reflex
A

B, C and D are seen in a pseudobulbar palsy, an upper motor neurone disorder whilst A and E (as well as B) may be seen in a bulbar palsy, a lower motor neurone disorder. In a pseudobulbar palsy there is a spastic dysarthria, dysphagia, brisk jaw jerk and a spastic tongue. In a bulbar palsy, there is a nasal dysarthria, dysphagia, absent gag reflex and a wasted tongue with fasciculations.

269
Q

A 25-year-old man returns from holiday to Spain and develops sudden weakness of the legs followed a few days later by weakness in the arms. On examination he has weakness in all four limbs and his reflexes are absent.

  • Brown-Sequard syndrome
  • Peripheral neuropathy
  • Myopathy
  • Brainstem lesion
  • Spinal cord compression
  • Anterior spinal cord lesion
  • Neuromuscular junction disorder
  • Plexopathy
  • Posterior spinal cord lesion
  • Motor cortex lesion
A

Peripheral neuropathy

This man most likely has Guillain-Barre syndrome (GBS) i.e. a peripheral neuropathy. His reflexes are absent - a lower motor neurone lesion. Acute ascending weakness in a lower motor neurone pattern is most likely caused by GBS.

270
Q

A 71-year-old man with known prostate cancer presents with increasing difficulty walking over the last couple of weeks. He has weakness of the flexor muscles in the legs and brisk reflexes. All sensory modalities are impaired in the legs with light touch and pinprick sensation impaired onto the abdomen up to the umbilicus.

  • Brown-Sequard syndrome
  • Peripheral neuropathy
  • Myopathy
  • Brainstem lesion
  • Spinal cord compression
  • Anterior spinal cord lesion
  • Neuromuscular junction disorder
  • Plexopathy
  • Posterior spinal cord lesion
  • Motor cortex lesion
    *
A

Spinal cord compression.

This gentleman has a spinal cord lesion affecting the whole cord (corticospinal tracts, dorsal columns and spinothalamic tracts). He has a sensory level suggesting a lesion affecting the spinal cord about T10.

271
Q

A 61-year-old man has difficulty in producing speech. His speech contains incorrect words that are similar in sound to the correct word. His speech is also missing function words such as ‘and’, ‘the’ and ‘of’. Comprehension of speech is normal.

  • Broca’s aphasia
  • Pseudobulbar dysarthria
  • Cerebellar dysarthria
  • Wernicke’s aphasia
  • Conduction aphasia
  • Cerebellar dysarthria
  • Bulbar dysarthria
  • Hypokinetic (Parkinsonian) dysarthria
  • Transcortical sensory aphasia
A

Broca’s aphasia

This man has a nonfluent or expressive aphasia with speech errors and agrammatism characteristic of Broca’s aphasia in contrast to the fluent or receptive aphasia seen in Wernicke’s aphasia.

272
Q

A 75-year-old man has difficulty with repeating words but relatively normal speech production apart from the occasional speech error. Comprehension of speech is normal.

  • Broca’s aphasia
  • Pseudobulbar dysarthria
  • Cerebellar dysarthria
  • Wernicke’s aphasia
  • Conduction aphasia
  • Cerebellar dysarthria
  • Bulbar dysarthria
  • Hypokinetic (Parkinsonian) dysarthria
  • Transcortical sensory aphasia
A

Conduction aphasia

This man has relatively intact speech production and comprehension but impaired repetition – this is consistent with conduction aphasia (and distinct to either Broca’s aphasia where there is speech production impairment and Wernicke’s aphasia where there is speech comprehension impairment).

273
Q

A 65-year-old man has had rapidly progressive memory problems over the last few months and his arms have started intermittently jerking.

  • Idiopathic Parkinson’s Disease
  • Essential tremor
  • Huntington’s disease
  • Hemiballismus
  • Drug-induced parkinsonism
  • Progressive supranuclear palsy
  • Wilson’s disease
  • Thyrotoxicosis
  • Multiple system atrophy
  • Sydenham’s chorea
  • Creutzfeldt-Jakob disease
  • Physiological tremor
A

CJD

This man has myoclonus of the arms. This occurs in a number of conditions but in association with a rapidly progressive dementia is likely to be caused by CJD.

274
Q

A 26-year-old woman presents with a headache and confusion and on examination was found to have papilloedema. She has a tonic-clonic seizure whilst still in A+E.

  • Juvenile myoclonic epilepsy
  • Hyponatraemia
  • Hypoxia
  • Venous sinus thrombosis
  • Encephalitis
  • Hypoglycaemia
  • Hypocalcaemia
  • Temporal lobe epilepsy
  • Alcohol withdrawal seizures
  • Glioma
  • Head injury
  • Idiopathic generalized epilepsy
A

Venous Sinus Thrombosis

The presence of headache, seizures and raised intracranial pressure (the patient has papilloedema) is seen in venous sinus thrombosis. This is more likely than encephalitis which may also cause headache and seizures.

275
Q

A 30-year-old woman has a normal pupillary size on observation but on shining the light into each eye in turn, the left pupil dilates rather than constricts.

  • Horner’s syndrome
  • Third nerve palsy
  • Adie’s pupil
  • Opiate overdose
  • Argyll Robertson pupil
  • Marcus Gunn pupil
  • Pontine haemorrhage
  • Tricyclic antidepressant overdose
A

Marcus Gunn pupil

This woman has a relative afferent pupillary defect or a Marcus Gunn pupil. Unlike most other pupillary abnormalities the pupils are normal on observation. However, the ‘swinging flashlight’ test reveals a paradoxical dilation of the pupil on shining the light into the eye with the abnormal optic nerve.

276
Q

A 45-year-old woman has a dilated left pupil. Her neurological exam is otherwise normal apart from decreased reflexes throughout.

  • Horner’s syndrome
  • Third nerve palsy
  • Adie’s pupil
  • Opiate overdose
  • Argyll Robertson pupil
  • Marcus Gunn pupil
  • Pontine haemorrhage
  • Tricyclic antidepressant overdose
A

Adie pupil

A unilateral dilated pupil without a ptosis or eye movement disorder is most commonly due to an Adie’s pupil. Loss of deep tendon reflexes is often seen in this condition – Holmes-Adie syndrome.

277
Q

A 72-year-old woman is found to have a right posterior cerebral artery infarct.

  • Central scotoma
  • Left homonymous hemianopia
  • Right inferior homonymous quadrantanopia
  • Left inferior homonymous quadrantanopia
  • Bitemporal hemianopia
  • Right superior homonymous quadrantanopia
  • Left superior homonymous quadrantanopia
  • Right homonymous hemianopia
  • Tunnel vision
A

Left homonymous hemianopia

Posterior cerebral artery infarcts are usually associated with homonymous hemianopias. Beyond the optic chiasm the visual hemifield is represented on the opposite side of the brain i.e. left hemifield in the right hemisphere.

278
Q

A 25-year-old woman develops a painful visual disturbance in her left eye. She had previously had an episode of ataxia a year earlier lasting a few months.

  • Central scotoma
  • Left homonymous hemianopia
  • Right inferior homonymous quadrantanopia
  • Left inferior homonymous quadrantanopia
  • Bitemporal hemianopia
  • Right superior homonymous quadrantanopia
  • Left superior homonymous quadrantanopia
  • Right homonymous hemianopia
  • Tunnel vision
A

Central scotoma

This woman is likely to have multiple sclerosis (two episodes of focal neurological deficits separated in time and space). The latest deficit is likely to be an optic neuritis which may cause a central scotoma.

279
Q

A 45-year-old presents with a feeling of the world spinning for the last few days. She has also vomited a few times.

  • Benign paroxysmal positional vertigo
  • Vasovagal syncope
  • Vestibular migraine
  • Vertebrobasilar ischaemia
  • Labyrinthitis
  • Meniere’s disease
  • Postural hypotension
  • Cardiac arrhythmia
A

Labyrinthitis

A ‘labyrinthitis’ or vestibular neuronitis causes vertigo lasting days, compared to BBPV which causes vertigo lasting seconds, and vertebrobasilar ischaemia lasting minutes and Meniere’s disease lasting hours.

280
Q

A 15 year old boy presents with a rash on his buttocks, arms and legs, a few days after a sore throat. The mother is extremely worried because on trying to the tumbler test, the rash did not go away. He also complains of pain in his joints and abdomen and urine dipstick shows the presence of blood and protein. What is the most likely diagnosis?

A) Disseminated bacterial meningitis

B) HUS

C) DIC

D) Goodpasture’s disease

E) Henoch-Schonlein purpura

A

Henoch-Schonlein purpura

Commonly said to be a systemic variant of IgA GN, the presentation is typical for HSP.

281
Q

A 45 year old clerical assistant comes to see you in GP with a sore throat that has been getting worse over the last week and which is really affecting her work. She also reports a non-productive cough which she has had for the last 2 weeks and that she is feeling hot and bothered. Her PMH is significant for treated hyperthyroidism and vitiligo, for which she has been seen by a dermatologist.

Which is the most appropriate?

A) Urgent CXR

B) Urgent TFTs

C) Urgent FBC

D) Monospot test

E) Start on steroids and amoxicillin.

A

Urgent FBC

This patient is a treated hyperthyroidism patient and is therefore likely to be on carbimazole or PTU and so you have to be aware of the rare possibility of agranulocytosis and exclude this with a FBC. Should the blood test show a low PMN count, urgent admission to hospital and treatment with broad spec antibiotics is required, with cessation of carbimazole or PTU.

282
Q

A 70 year old man has come to see you with fullness in the abdo and tiredness. His health has generally been very good and he reports no problems apart from a few lumps that appear occassionally but slowly subside. His routine blood tests show anaemia, with a decrease in the haptoglobin levels and a marked lymphocytosis. What is the most likely diagnosis?

A) CLL

B) ALL

C) PML

D) Hodgkin’s disease

E) Multiple myeloma

A

CLL

The fullness in his abdomen indicates splenomegaly, which occurs in a quarter of patients with CLL. This leukaemia is seen most commonly in the elderly and has a variable course, often affecting the patient for a number of years but not causing significant distress. CLL is associated with AI haemolysis caused by warm IgG Ab, which causes RBC breakdown releasing free Hb, this is scavenged by haptoglobin readily thus accounting for the fall in haptoglobin. Other complications include aplastic anaemia and overwhelming sepsis.

283
Q

Which is NOT typically inherited in an AutoR manner?

A) Hereditary haemochromatosis

B) Hereditary spherocytosis

C) Wilson’s disease

D) Infantile PCKD

E) CF

A

Hereditary spherocytosis.

This inherited in AutoD manner.

Adult PCKD is AutoD but infant is AutoR.

284
Q

A 92 year old gentleman is brought into the ED with dehydration and continual vomiting. He is known to have a large caecal adenocarcinoma but has been deemed too high an operative risk by the surgeons, who are unwilling to operate. You start IV fluids and insert a large NG tube, which relieves somes of his symptoms. After 5 days he continues to deteriorate, with worsening abdo tenderness and large volumes of NG aspirate. After discussion with the family and taking into account the patient’s wishes it is decided to keep the patient comfortable and start him on the Liverpool Care Pathway for the dying patient. Which of the following is true with regard to end of life care?

A) LPC should only be used when cancer is the underlying diagnosis

B) Fluids are routinely continued to prevent dehydration in the dying patient.

C) Octreotide has a useful role in the above patient

D) Metoclopramide would be a useful SC antiemetic in the above patient.

E) Medication should be given on a PRN basis as much as possible to avoid distress caused by continuous syringe driver infusion.

A

Octreotide has a useful role in the above patient.

The care of the dying patient requires a strong MDT approach, including the patient’s own team of doctors and nurses along with palliative care services and hospices. Palliative care has a role not only in patients with cancer but alsi in any disease requiring support at the end of life, such as end stage heart failure and COPD.

The LCP is a quality improvement framework which aims to improve the care of the dying patient in the hospital setting irrespective of the cause of the terminal condition.

Fluids are not routinely continued and medication is best given by a syringe driver to provide continuous analgesia with or without antiemetic cover, rather than the peak and trough therapy achieved with PRN, which should be used for breakthrough symptoms.

In patients with bowel obstruction, as in this patient, prokinetic antiemetics are CI and cyclizine and ondasetron would be better suited agents. Octreotide is useful in limiting gastric secretions.

285
Q

Elevated JVP with absent pulsation

A tricuspid stenosis
B ventricular fibrillation
C tricuspid regurgitation
D constrictive pericarditis
E aortic regurgitation
F atrial fibrillation
G complete heart block
H left heart failure
I mitral stenosis
J aortic stenosis
K superior vena cava obstruction
L normal JVP

A

SVC obstruction

Bronchial carcinoma is a well-recognized cause of this medical emergency.
Symptoms include early morning headache (feeling of fullness in
the head) and signs include facial congestion and oedema involving the
upper limb.
Either radiotherapy or chemotherapy may be useful depending on the
sensitivity of the tumour type.

286
Q

Large ‘a’ waves and slow ‘y’ descent in JVP. Patient has ascites.

A tricuspid stenosis
B ventricular fibrillation
C tricuspid regurgitation
D constrictive pericarditis
E aortic regurgitation
F atrial fibrillation
G complete heart block
H left heart failure
I mitral stenosis
J aortic stenosis
K superior vena cava obstruction
L normal JVP

A

Tricuspid stenosis

Rheumatic fever is the most common cause of tricuspid stenosis. There is
usually involvement of other valves, e.g. coexisting mitral stenosis. The
prominent symptom is fatigue. The presence of shortness of breath suggests
concomitant mitral valve disease.
Surgical intervention by tricuspid valve replacement is usually carried
out only when there are other defective valves also being operated on.

287
Q

Cannon ‘a’ waves.

A tricuspid stenosis
B ventricular fibrillation
C tricuspid regurgitation
D constrictive pericarditis
E aortic regurgitation
F atrial fibrillation
G complete heart block
H left heart failure
I mitral stenosis
J aortic stenosis
K superior vena cava obstruction
L normal JVP

A

Complete heart block

Cannon waves occur when there is atrioventricular (AV) dissociation.
The classic example is complete heart block but it may also be seen in
ventricular tachycardia, and in patients with a single chamber
pacemaker and continuing atrial contractions. This is rare nowadays
because such patients are invariably given dual-chamber pacemakers.
The cannon wave is generated by the atrium contracting in the presence
of a closed tricuspid valve resulting from simultaneous ventricular
systole.

288
Q

Dominant R in V1, inverted T waves in V1–V3, deep wide
S waves in V6.

A subendocardial infarction
B pulmonary embolus
C right bundle-branch block
D Mobitz type II second-degree
heart block
E hyperkalaemia
F inferior myocardial infarction
G anterolateral myocardial
infarction
H left bundle-branch block
I hypokalaemia
J pericarditis
K atrial fibrillation
L mitral regurgitation
M mitral stenosis

A

RBBB

The ‘MARROW’ pattern, e.g. ‘RSR’ pattern in V1 (M) with deep wide S
wave in V6 (W). Causes include atrial septal defect (ASD) and pulmonary
embolus.

289
Q

Sinus rhythm, bifid ‘p’ waves best seen in II, V3 and V4.

A subendocardial infarction
B pulmonary embolus
C right bundle-branch block
D Mobitz type II second-degree
heart block
E hyperkalaemia
F inferior myocardial infarction
G anterolateral myocardial
infarction
H left bundle-branch block
I hypokalaemia
J pericarditis
K atrial fibrillation
L mitral regurgitation
M mitral stenosis

A

Mitral stenosis

Known as P mitrale, this bifid P wave suggests left atrial hypertrophy (as a resultt of mitral stenosis).

A peaked P wave is called P pulmonale and suggests right atrial hypertrophy.

290
Q

Hypertension in a 75 year old who is a heavy smoker with widespread peripheral vascular disease.

A Cushing’s syndrome
B systemic sclerosis
C coarctation of the aorta
D Conn’s syndrome
E pregnancy
F polycystic kidneys
G malignant hypertension
H hyperparathyroidism
I renal artery stenosis
J portal hypertension
K phaeochromocytoma

A

Renal Artery Stenosis

Renal disease is the most common cause of secondary hypertension.
Intrinsic renal disease, e.g. glomerulonephritis, makes up most of these
cases. Renal artery stenosis is responsible for around 25 per cent of all
cases of renal hypertension Treatment is directed at reducing blood pressure
and preserving renal function. Revascularization may be performed
but there is no consensus as to which patients may benefit from this
therapy compared with from medical treatment.

291
Q

A 65-year-old man with heart failure requires rate control to treat coexisting
atrial fibrillation.

A) nifedipine

B) metolazone

C) 100% O2, IV diamorphine, IV furosemide, sublingual GTN

D) IV ISMN

E) lidnocaine

F) digoxin

G) 100% O2, oral diamorphine, oral furosemide, IV GTN

H) IV adenosine

I) Spironolactone

J) Start CPR

K) Oral furosemide.

A

Digoxin

Digoxin has a narrow therapeutic window and toxicity with normal
doses can be precipitated by hypokalaemia, hypomagnesaemia, renal impairment and hypercalcaemia. Signs of digoxin toxicity include
confusion, nausea, arrhythmias and visual disturbance.

292
Q

A 70-year-old woman with a history of chronic heart failure presents
with severe pulmonary oedema.

A) nifedipine

B) metolazone

C) 100% O2, IV diamorphine, IV furosemide, sublingual GTN

D) IV ISMN

E) lidnocaine

F) digoxin

G) 100% O2, oral diamorphine, oral furosemide, IV GTN

H) IV adenosine

I) Spironolactone

J) Start CPR

K) Oral furosemide.

A

100% O2, IV diamorphine, IV furosemide, sublingual GTN

Acute pulmonary oedema is a life-threatening medical emergency and
the diuretic should be given parenterally. Elderly patients may suffer
from COPD/asthma or have a coexisting chest infection and should
therefore be given antibiotics and nebulized bronchodilators as required.

293
Q

Treatment of mild symptoms of shortness of breath and ankle
oedema in a 65-year-old man with left ventricular dysfunction
caused by ischaemic heart disease. He is already taking an
ACE inhibitor.

A) nifedipine

B) metolazone

C) 100% O2, IV diamorphine, IV furosemide, sublingual GTN

D) IV ISMN

E) lidnocaine

F) digoxin

G) 100% O2, oral diamorphine, oral furosemide, IV GTN

H) IV adenosine

I) Spironolactone

J) Start CPR

K) Oral furosemide.

A

Oral furosemide

Patients with mild left ventricular dysfunction may be satisfactorily controlled
on an ACE inhibitor. If, however, shortness of breath and ankle
oedema are not sufficiently controlled, oral diuretics are added. The drug
of choice is a loop diuretic.

294
Q

A 55-year-old man admitted with an acute myocardial infarction
develops a short run of VT. He requires treatment for prophylaxis
against recurrent VT.

A intravenous adenosine
B oral warfarin
C oral lidocaine (lignocaine)
D digoxin warfarin for a month
E intravenous magnesium
ventricular pacing
F intravenous amiodarone
G low-molecular-weight (LMW)
heparin
H oral amiodarone warfarin
I direct current (DC) shock
heparin
J oral sotalol warfarin
K none of the above

A

IV amiodarone

Amiodarone has class I, II, III and IV actions but is used clinically for its
class III actions. Class III drugs prolong the plateau phase of the cardiac
action potential and increase the absolute refractory period. As a
consequence they also prolong the Q–T interval.
Amiodarone is the drug of choice to treat VT. When it is used chronically
it has a number of adverse effects but these are not an issue in the acute
scenario. These adverse effects include bradycardia, pulmonary fibrosis,
hepatic fibrosis, corneal microdeposits (regress if drug is stopped),
photosensitive rash and thyroid dysfunction

295
Q

A 45 year old man with chronic glomerulonephritis presents with a severe headached. O/E he has papilloedema and bilateral retinal haemorrhages. His BP is 240/132 mmHg.

A oral dobutamine
B DC shock and adrenaline
C 100 per cent O2, subcutaneous LMW heparin, intravenous fluids
D nifedipine
E atropine
F aspirin, heparin
G DC shock and atropine
H labetalol
I emergency renal dialysis
J aspirin, GTN
K aspirin, streptokinase

A

Labetalol

This is the presentation of malignant hypertension. The therapeutic aim
should be rapid, but gradual and controllable reduction in blood pressure
is the ideal. Both oral and parenteral therapy may be used depending on
the clinician’s preference. Sublingual nifedipine is contraindicated
because it may produce a profound uncontrollable reduction in the
patient which may compromise cerebral perfusion.

296
Q

A 45 year old oil rig worker is admitted with fever and shortness of breath following a return from a secondment overseas. Four days into his admission he develops worsening symptoms and a chest radiograph reveals a pleural collection. A pleural tap is performed due to concerns this may represent an empyema. Which of the following results is most fitting with an empyema?

a. Protein 2.4 g/dl; LDH 378 mmol/l; pH 7.15
b. Protein 3.8 d/dl; LDH 370 mmol/l; pH 7.15
c. Protein 2.8 g/dl; LDH 277 mmol/l; pH 7.45
d. Protein 3.8 g/dl; LDH 294 mmol/l; pH 7.45
e. Protein 3.4 g/dl; LDH 213 mmol/l; pH 7.38

A

Protein 3.8 g/dl LDH 370 mmol/l pH 7.15

A pleural effusion is a collection of fluid in the pleural space, between the parietal and serosal layers of the pleura. Pleural aspiration is usually necessary to determine the cause and nature of the effusion, whereby a sample is sent for biochemical analysis, bacteriological culture and microscopic examination. The effusion can then be classified into a transudate or exudate depending on its biochemical characteristics. pH is measured when there is concern regarding an empyema as it typically has a pH of less than 7.2.

Transudates have a protein content of <3g/dl and a lactate dehydrogenase (LDH) concentration of <200mmol/l. Causes of transudates include: cardiac failure, hypoalbuminaemia (‘nutritional failure’), renal failuire, liver failure and constrictive pericarditis.

Exudates have a protein content of >3g/dl and a LDH concentration of >200mmol/l. Causes of exudates include: lobar pneumonia, lung cancer, mesothelioma, pulmonary embolus, TB and empyema.
Empyemas are infected pleural collections which require prompt drainage. This is attempted in the first instance with insertion of a pleural drain., If drainage fails, surgical intervention is required. Empyemas have a low pH (< 7.2), are exudates (protein > 3g/dl) and have a high LDH content (> 200 mmol/l).

297
Q

A 68 year year old retired British Coal employee with a long history of shortness of breath attends a respiratory outpatient clinic. This is his yearly clinic review at which he has a chest x-ray. His symptoms are unchanged from 12 months previously. The report of his CXR reads ‘multiple cavitating lesions in both upper lobes, which are unchanged from the previous film’.

a. Squamous cell carcinoma metastasis
b. Squamous cell primary lung malignancy

c. Wegener’s granulomatosis
d. Tuberculosis
e. Pulmonary infarct
f. Staphylococcus pneumonia
g. Progressive massive fibrosis
h. Aspergilloma

A

Progressive Massive Fibrosis (G)
Progressive massive fibrosis (PMF) is due to progression of Coal worker’s pneumoconiosis. With coal worker’s pneumoconiosis, the chest x-ray shows small pulmonary nodules less than 1.5mm. With progressive massive fibrosis, the chest x-ray reveals large fibrotic masses (1-10 cm), predominantly in the upper zones. These masses may cavitate over time.

298
Q

A 62 year old man with a 10 year history of emphysema admits to increasing shortness of breath over the past 3 weeks. Over the last few days he has had several episodes of haemoptysis. He is known to have several upper lobe bullae. His chest x-ray report reads, ‘Air crescent sign within a cavitating right upper lobe lesion, correlating with one of the established bullae’.

a. Squamous cell carcinoma metastasis
b. Squamous cell primary lung malignancy

c. Wegener’s granulomatosis
d. Tuberculosis
e. Pulmonary infarct
f. Staphylococcus pneumonia
g. Progressive massive fibrosis
h. Aspergilloma

A

Aspergilloma (H)
Aspergilli are typically inhaled as small spores which do not affect people without underlying lung or immune system disease. However, people with pre-existing lung problems, especially cavitating lung diseases, are at risk. This is because the fungus can settle in the cavities within the lungs, and is able to grow free from interference as the immune system is unable to penetrate the cavity. The chest x- ray findings show lesions with an air “halo/crescent” around them.People can often be unaware for some time before incidental diagnosis. However, a small percentage of aspergilliomas invade into the cavity wall and can result in bleeding and hence haemoptysis, which may require resection of the damaged lung area containing the aspergilloma.

299
Q

A 35 year old man with a 3 month history of sinus disease for which he is attending an ENT clinic is admitted with increasing shortness of breath. His chest x-ray reveals multiple cavitating lungs lesions in both lungs. His weight is stable and his white blood cell count is normal.

a. Squamous cell carcinoma metastasis
b. Squamous cell primary lung malignancy

c. Wegener’s granulomatosis
d. Tuberculosis
e. Pulmonary infarct
f. Staphylococcus pneumonia
g. Progressive massive fibrosis
h. Aspergilloma

A

Wegener’s granulomatosis (C)

The patient’s clinical presentation is most in keeping with Wegener’s granulomatosis. This is a multisystem disorder due to a granulomatous reaction within small/medium sized arteries and veins. It typically starts with rhinorrhoea, with subsequent nasal mucosal ulceration, cough, dyspnoea, haemoptysis, epistaxis and/or pleuritic pain. A “saddle nose” deformity is also a feature sometimes seen in this condition. The chest x-ray shows nodular masses or pneumonic infiltrates with cavitation. This disease responds well to immunosuppression, eg. with cyclophosphamide.

300
Q

A 17 year old 1st year university student attends her GP with a short history of shortness of breath and a productive cough. She has been ignoring her symptoms with all the excitement of university and life in halls of residence, but it is now restricting her activities of daily living. A chest x-ray reveals a 3 cm cavitating left upper lobe lesion, with an enlarged left hilum.

a. Squamous cell carcinoma metastasis
b. Squamous cell primary lung malignancy c. Wegener’s granulomatosis
d. Tuberculosis
e. Pulmonary infarct
f. Staphylococcus pneumonia
g. Progressive massive fibrosis
h. Aspergilloma

A

Tuberculosis (D)

Tuberculosis transmission is through air and direct contact, whereby the initial infection is with Mycobacterium tuberculosis. Primary TB is usually asymptomatic; however, occasionally there may be erythema nodosum, a small pleural effusion or pulmonary collapse caused by compression of a lobar bronchus by enlarged nodes.

The chest x-ray typically shows consolidation, air space or nodular changes in the upper zones, with loss of volume of the lung, and an enlarged hilum due to the enlarged lymph nodes.

301
Q

A 65 year old lady is admitted with episodes of syncope Her fasting blood sugar is 2.6 mmol/l. Respiratory examination is normal. The chest x-ray is abnormal but there is no mediastinal abnormality.

A) Bronchoalveolar cell carcinoma

B) Squamous cell carcinoma
C)Small cell carcinoma
D) Pleural fibroma

E) Mesothelioma

F)Thymoma

G) Teratoma
H) Large cell carcinoma

A

Pleural fibroma
Pleural fibroma is a rare and benign primary tumor, which is mainly asymptomatic. It is not associated with asbestos exposure. The solitary mass can grow to be very large due to its slow growth from the visceral pleural. Pleural fibromas most commonly affect women with the mean age of presentation being around 50 years of age. These tumours generally behave in a benign fashion, although malignant tumours do exist. These tumours are known to be able to produce insulin-like growth factor II (IGF-II), causing the increased utilization of glucose, inducing the hypoglycaemia seen here.

302
Q

A 34-year-old woman originally complaining of nasal obstruction
develops cough, haemoptysis and pleuritic chest pain. Her chest
radiograph shows multiple nodular masses.

A tuberculosis (TB)
B haemothorax
C bronchogenic carcinoma
D polyarteritis nodosa
E Wegener’s granulomatosis
F bronchiectasis
G Goodpasture’s syndrome
H haemophilia
I pulmonary oedema
J pneumonia
K pulmonary embolus
L Churg–Strauss syndrome

A

Wegener’s granulomatosis

Wegener’s granulomatosis is a small artery vasculitis (PR3 ANCA positive),
which is characterized by lesions involving the upper respiratory tract,
lungs and kidneys. Look out for eye signs that are present in up to 50 per
cent, e.g. scleritis, uveitis, retinitis. However, the vasculitis and granuloma
deposition can affect any organ and so less common associated symptoms
and signs are legion. Treatment options include the use of immunosuppressive
medications, e.g. high-dose corticosteroids with cyclophosphamide.

303
Q

Multiple bilateral nodules between 0.5 and 5 cm in a former miner
with rheumatoid arthritis.

A mitral stenosis
B bronchiectasis
C right upper lobe collapse
D Kartagener’s syndrome
E aortic stenosis
F previous varicella pneumonitis
G asbestosis
H right lower lobe collapse
I Wegener’s granulomatosis
J Caplan’s syndrome
K post-fracture fat embolus
L right ventricular failure
M left ventricular failure

A

Caplan syndrome

Caplan’s syndrome is a pulmonary manifestation of rheumatoid arthritis
(RA) which is characterized by the presence of pulmonary nodules. It
typically occurs in patients with RA who are exposed to coal dust,
although the granulomas can also appear in workers exposed to other
dusts, e.g. silicosis and asbestos. Symptoms include cough, shortness of
breath and haemoptysis.
RA has several other respiratory manifestations/associations including
fibrosing alveolitis, pleural effusions and, very rarely, obliterative
bronchiolitis. RA can also affect the cricoarytenoid joints leading to
upper respiratory tract obstruction.

304
Q

Trachea deviated to right, horizontal fissure and right hilum displaced
upwards.

A mitral stenosis
B bronchiectasis
C right upper lobe collapse
D Kartagener’s syndrome
E aortic stenosis
F previous varicella pneumonitis
G asbestosis
H right lower lobe collapse
I Wegener’s granulomatosis
J Caplan’s syndrome
K post-fracture fat embolus
L right ventricular failure
M left ventricular failure

A

Right upper lobe collapse

These chest radiograph findings are characteristic of right upper lobe
collapse. Try to find the horizontal fissure because its position is a good
clue to the presence of volume loss. The horizontal fissure on the right
lung should run from the middle of the right hilum and can be traced to the level of the sixth rib in the axillary line. In right upper lobe collapse,
the horizontal fissure will be elevated.

Chest radiograph findings of a left upper lobe collapse are somewhat different.
There is no left middle lobe and hence no horizontal fissure. The
upper lobe is anterior to a greater proportion of the lower lobe. Hence,
left upper lobe collapse can give rise to a hazy white appearance over a
large part of the left lung field. This should not be confused with a
pleural effusion because in collapse there will be tracheal deviation to
the side of the lesion, elevation of the hilum and preservation of the
costophrenic angle.

305
Q

Numerous calcified nodules sized less than 5 mm located predominantly
in the lower zones of the lungs.

A mitral stenosis
B bronchiectasis
C right upper lobe collapse
D Kartagener’s syndrome
E aortic stenosis
F previous varicella pneumonitis
G asbestosis
H right lower lobe collapse
I Wegener’s granulomatosis
J Caplan’s syndrome
K post-fracture fat embolus
L right ventricular failure
M left ventricular failure

A

Previous varicella pneumonitis

Multiple, small, calcified nodules may occur after varicella pneumonitis.
Other causes of numerous calcified nodules include TB, histoplasmosis
and chronic renal failure.

306
Q

Double shadow right heart border, prominent left atrial appendage,
left main bronchus elevation.

A mitral stenosis
B bronchiectasis
C right upper lobe collapse
D Kartagener’s syndrome
E aortic stenosis
F previous varicella pneumonitis
G asbestosis
H right lower lobe collapse
I Wegener’s granulomatosis
J Caplan’s syndrome
K post-fracture fat embolus
L right ventricular failure
M left ventricular failure

A

Mitral stenosis

Advanced mitral stenosis is associated with characteristic findings caused
by left atrial enlargement. These include elevation of the left main
bronchus, widening of the carina, double right heart border and a prominent
left atrial appendage. Calcification of the mitral valve may also be
seen, as may pulmonary oedema. Left ventricular enlargement is not a
feature despite the presence of pulmonary oedema.

307
Q

The chest radiograph of a 13-year-old boy with cystic fibrosis has
tramline and ring shadows.

A pneumonia
B silicosis
C bronchiectasis
D aspergillosis
E mesothelioma
F cryptogenic fibrosing alveolitis
G chronic obstructive pulmonary disease (COPD)
H carcinoma of bronchus
I tuberculosis
J berylliosis
K sarcoidosis
M extrinsic allergic alveolitis

A

Bronchiectasis

Ring shadows and tramlining are a characteristic radiological finding in
bronchiectasis, which is a common early complication of cystic fibrosis.
Patients present with a cough productive of large amounts of purulent
sputum and there can be haemoptysis. On examination the patient may be clubbed with coarse inspiratory crackles that can be heard over the
infected areas of lung.
Other causes of bronchiectasis include Kartagener’s syndrome, pertussis
and bronchial obstruction.

308
Q

A 7-year-old girl with slight wheeze and shortness of breath despite
inhaled salbutamol.

A low-dose oral aminophylline
B oral prednisolone
C inhaled beclomethasone
D inhaled ipratropium bromide
E nebulized ipratropium bromide
F inhaled sodium cromoglycate
G nebulized salbutamol
H inhaled salmeterol
I home 100 per cent oxygen
J inhaled aminophylline
K inhaled salbutamol
M oral sodium cromoglycate
N inhaled salbutamol with spacer

A

Inhaled salbutamol with spacer

This patient’s symptoms may respond to inhaled salbutamol if it can be
delivered effectively to the lungs. Use of a spacer improves delivery in
children and patients with poor inhaler technique. In young patients, it
is particularly important to optimize bronchodilator therapy as early
initiation of corticosteroids may lead to growth retardation.

309
Q

A 65-year-old man with long-standing COPD presents with severe
shortness of breath. He has been treated with oxygen and nebulized
bronchodilators. An hour later: PaO2 6.0 kPa (on max. O2), PaCO2
16.0 kPa, pH 7.2.

A intubation
B left-sided decompression
C chest radiograph
D nasal intermittent positive pressure ventilation
E 28 per cent O2, nebulized salbutamol, oral prednisolone
F emergency tracheostomy
G 28 per cent O2, nebulized salbutamol, intramuscular adrenaline
H right-sided decompression
I 28 per cent O2, nebulized salbutamol, intravenous hydrocortisone
J chest drain
K 100 per cent O2, intramuscular adrenaline, nebulized salbutamol
L intravenous hydrocortisone
M 100 per cent O2, nebulized salbutamol, intravenous hydrocortisone

A

Nasal intermittent positive pressure ventilation

This patient has a severe exacerbation of COPD and consequent type II
respiratory failure that has responded poorly to medical therapy.
Conventional management would involve formal intubation, ventilation
and transfer to an intensive care unit (ICU). The use of non-invasive intermittent positive pressure ventilation (NIPPV) in such patients has
been associated with a reduction in the number of patients requiring formal
intubation. This is advantageous because anaesthesia and intubation
can be very difficult in a patient with respiratory failure. NIPPV should
be tried unless the patient is in extremis. Successful NIPPV requires a
conscious and cooperative patient.

310
Q

A 17-year-old woman presents with wheeze and marked perioral
swelling: PaO2 7.0 kPa (on 28 per cent O2), PaCO2 4.1 kPa.

A intubation
B left-sided decompression
C chest radiograph
D nasal intermittent positive
pressure ventilation
E 28 per cent O2, nebulized salbutamol, oral prednisolone
F emergency tracheostomy
G 28 per cent O2, nebulized salbutamol, intramuscular adrenaline
H right-sided decompression
I 28 per cent O2, nebulized salbutamol, intravenous hydrocortisone
J chest drain
K 100 per cent O2, intramuscular adrenaline, nebulized salbutamol
L intravenous hydrocortisone
M 100 per cent O2, nebulized salbutamol, intravenous hydrocortisone

A

100% O2, IM adrenaline, nebs salbutamol

This is a classic presentation of acute anaphylaxis, e.g. a type I IgEmediated
hypersensitivity reaction. The symptoms and signs include
rash, oedema, tachycardia, hypotension and wheeze. Laryngeal oedema
giving rise to upper airway obstruction is particularly worrying because
it may impede endotracheal intubation.
Initial treatment of choice is 0.5 ml epinephrine (adrenaline) 1:1000 solution
(500g) delivered intramuscularly which can be repeated in the
absence of clinical improvement or if deterioration occurs. Intravenous
epinephrine is dangerous and should be given slowly only in a dilution
of at least 1 in 10 000 in an immediately life-threatening situation, e.g.
frank cardiac arrest.

311
Q

A 14 year old with asthma presents with an acute severe asthma
attack: PaO2 10.0 kPa (on 28 per cent O2), PaCO2 8.0 kPa

A intubation
B left-sided decompression
C chest radiograph
D nasal intermittent positive
pressure ventilation
E 28 per cent O2, nebulized salbutamol, oral prednisolone
F emergency tracheostomy
G 28 per cent O2, nebulized salbutamol, intramuscular adrenaline
H right-sided decompression
I 28 per cent O2, nebulized salbutamol, intravenous hydrocortisone
J chest drain
K 100 per cent O2, intramuscular adrenaline, nebulized salbutamol
L intravenous hydrocortisone
M 100 per cent O2, nebulized salbutamol, intravenous hydrocortisone

A

IV hydrocortisone

The British Thoracic Society suggests that features of a severe asthma
attack include peak expiratory flow rate (PEFR) 50 per cent
predicted/best, respiration rate >25 breaths/min, pulse >110 beats/min
and inability to complete a sentence with one breath.
Markers of a life-threatening attack include a PEFR <33 per cent of predicted/
best, silent chest, cyanosis, poor respiratory effort, bradycardia,
arrhythmia, hypotension, exhaustion, confusion, PaO2 <8 kPa, acidosis
with pH <7.35, high PaCO2. Intubation and transfer to the ICU must be
considered if the patient is not responding to drug therapy.
This patient is both hypoxic and retaining CO2. This is a poor prognostic
sign because people with acute asthma usually have a low CO2. The presence of a high CO2 is associated with imminent respiratory collapse. Despite elevated CO2, 100 per cent O2 should be given because, in this patient, there is no risk of respiratory depression resulting from a hypoxic ventilatory drive.
Between attacks the patient’s CO2 should be within the normal range. It is a
common mistake to restrict oxygen to patients with asthma and a high CO2.

312
Q

A young man presents with an acute onset shortness of breath.
Examination reveals decreased expansion on the right: SaO2 95
per cent.

A intubation
B left-sided decompression
C chest radiograph
D nasal intermittent positive
pressure ventilation
E 28 per cent O2, nebulized salbutamol,
oral prednisolone
F emergency tracheostomy
G 28 per cent O2, nebulized salbutamol,
intramuscular adrenaline
H right-sided decompression
I 28 per cent O2, nebulized
salbutamol, intravenous
hydrocortisone
J chest drain
K 100 per cent O2, intramuscular
adrenaline, nebulized salbutamol
L intravenous hydrocortisone
M 100 per cent O2, nebulized
salbutamol, intravenous
hydrocortisone

A

CXR

A chest radiograph is indicated here to confirm the diagnosis of pneumothorax
and to assess the degree of collapse.
In healthy patients a small pneumothorax will often heal without further
intervention. The patient should be observed for 6 hours and, if there is
no increase in the size of the pneumothorax, may be discharged with
early follow-up and repeated chest radiograph. Spontaneous pneumothoraces
are relatively common in young adults (especially tall thin men)
and older patients with emphysema. In patients with thoracic
disease/large pneumothoraces, simple aspiration is recommended as firstline
treatment. If this is unsuccessful, a chest drain will be required.

313
Q

A 65-year-old woman with longstanding COPD presents with shortness
of breath and cough productive of coloured sputum.

A inhaled salbutamol
B 60 per cent O2 nebulized salbutamol, oral prednisolone, oral amoxicillin
C intravenous salbutamol
D long-term O2 therapy
E lung transplantation
F 28 per cent O2, nebulized salbutamol & ipratropium, oral prednisolone, oral amoxicillin
G 28 per cent O2, inhaled bronchodilators, intravenous amoxicillin
H oral aminophylline
I ICU admission and intubation
J 100 per cent O2, amoxicillin
K intravenous salmeterol
L NIPPV

A

28% O2, Nebs salbutamol + ipratropium, oral prednisolone, oral amoxicillin

This is infective exacerbation of COPD.

314
Q

A 65-year-old patient with advanced COPD treated with bronchodilators
and steroids still feels breathless. His baseline PaO2 is around
6.5 kPa.

A inhaled salbutamol
B 60 per cent O2 nebulized salbutamol, oral prednisolone, oral amoxicillin
C intravenous salbutamol
D long-term O2 therapy
E lung transplantation
F 28 per cent O2, nebulized salbutamol & ipratropium, oral prednisolone, oral amoxicillin
G 28 per cent O2, inhaled bronchodilators, intravenous amoxicillin
H oral aminophylline
I ICU admission and intubation
J 100 per cent O2, amoxicillin
K intravenous salmeterol
L NIPPV

A

Long term O2 therapy

Long-term oxygen therapy should be considered in clinically stable nonsmokers
with PaO2 7.3 kPa. There is evidence to suggest that keeping
the PaO2 8.0 kPa for more than 15 h every day increases survival.

315
Q

A 35-year-old patient on the ward admitted to hospital 10 days ago presents
with severe pneumonia.

A intravenous benzylpenicillin
B oral flucloxacillin
C oral tetracycline
D oral ciprofloxacin
E intravenous ceftazidime
F oral isoniazid & rifampicin
G intravenous flucloxacillin
H oral amoxicillin
I high-dose AZT (zidovudine) & pyrazinamide
J intravenous amoxicillin
K intravenous co-trimoxazole
L intravenous teicoplanin
M intravenous cefuroxime & erythromycin

A

IV ceftazidime

This infection is hospital acquired and therefore the range of pathogens
is likely to be different from those causing community-acquired
pneumonia. Such pathogens include Gram-negative aerobes, e.g.
Pseudomonas spp., and are often multiply antibiotic resistant. Third generation
cephalosporins, e.g. ceftazidime, have some anti-pseudomonal
activity and can prove effective in these circumstances.

316
Q

A 40-year-old builder presents with a severe community-acquired pneumonia.
Atypical pathogens are suspected

A intravenous benzylpenicillin
B oral flucloxacillin
C oral tetracycline
D oral ciprofloxacin
E intravenous ceftazidime
F oral isoniazid & rifampicin
G intravenous flucloxacillin
H oral amoxicillin
I high-dose AZT (zidovudine) & pyrazinamide
J intravenous amoxicillin
K intravenous co-trimoxazole
L intravenous teicoplanin
M intravenous cefuroxime & erythromycin

A

IV cefuroxime + erythromycin

This is standard therapy for a severe community-acquired pneumonia.
Rifampicin can be added empirically if there is a high clinical suspicion
of legionella infection.

317
Q

A 22-year-old human immunodeficiency virus (HIV)-positive individual
on anti-retroviral therapy presents with Pneumocystis carinii pneumonia.

A intravenous benzylpenicillin
B oral flucloxacillin
C oral tetracycline
D oral ciprofloxacin
E intravenous ceftazidime
F oral isoniazid & rifampicin
G intravenous flucloxacillin
H oral amoxicillin
I high-dose AZT(zidovudine) & pyrazinamide
J intravenous amoxicillin
K intravenous co-trimoxazole
L intravenous teicoplanin
M intravenous cefuroxime & erythromycin

A

IV co-trimoxazole.

Treatment of choice is high-dose co-trimoxazole delivered intravenously
for 2–3 weeks. Intravenous pentamidine may be used if co-trimoxazole is
contraindicated or not tolerated. Corticosteroids are often used as an
adjunct to treatment if there is hypoxaemia.

318
Q

Poisoning: Key features + Antidote

Digoxin

A

Key features

  • AV node block - bradycardia
  • Enhanced automaticity - arrhythmias + tachycardia
  • Yellow Halo (Xanthopsia)
  • Nausea + anorexia

Antidote: Digibind

319
Q

Poisoning: Key features + Management

Paracetamol

A

Key features:

  1. First few hours
    * N+V
  2. 24-48 hours - hepatotoxicity
  • Hepatic tenderness
  • Jaundice
  • Asterixis
  • Fetor hepaticus
  • Haemorrhage

Management

  • <75 mg/kg - no hepatotoxicity
  • 75-150 mg/kg - grey zone
  • >150 mg/kg - hepatotoxicity
  • Charcoal if under <1hr
  • N-acetyl cysteine
    • most effective in first 8 hours
320
Q

Poisoning: Key features + Antidote

Benzodiazepines

A

Key features

  • Drowsiness
  • Respiratory Depression
  • Worsened by alcohol

Antidote: Flumezanil

321
Q

Poisoning: Key features + Antidote

Salicylates

A

Key features

  • Tinnitus + vertigo
  • Hyperventilation
  • Dehydration + vomiting
  • Mixed respiratory alkalosis + metabolic acidosis

Management

  • <500mg/L
    • supportive
  • >500mg/L
    • First 2 hours - activated charcoal
    • Alkaline diuresis with IV sodium bicarbonate
  • >700mg/L
    • Haemodialysis if renal or cardiac failure or seizures occur.
322
Q

Poisoning: Key features + Antidote

Organophosphorous insecticides

A

Key features

  • Farmer
  • inactivate AChE
  • SLUD
    • Salivation
    • Lacrimation
    • Urination
    • Diarrhoea

Antidote :

  • atropine
  • severe cases - pralidoxime mesilate
323
Q

Poisoning: Key features + Antidote

Heparin

A

Key features

  • Bleeding

Antidote : Protamine

324
Q

Poisoning: Key features + Antidote

B blocker

A

Key features

  • severe bradycardia
  • hypotension

Antidote

  • atropine + glucagon
325
Q

Poisoning: Key features + Antidote

Carbon monoxide

A

Key features

  • Lows sats but normal ABG

Antidote

  • 100% O2
  • Hyperbaric chamber
326
Q

Poisoning: Key features + Antidote

TCA antidepressants

A

Key features

  • anticholinergic
  • Hypotension
  • arrhythmias
  • Hallucinations + seizures

Antidote

  • Lorazepam
327
Q

Poisoning: Key features + Antidote

Cyanide

A

Key features

  • Malaise
  • Collapse + confusion + coma
  • SOB
  • Abdo + chest pain

Antidote: Dicobalt edetate

328
Q

Poisoning: Key features + Antidote

Arsenic

A

Key features

  • Initially
    • gastroenteritis, garlic odour + hypersalivation
  • Delayed
    • multi-organ failure

Antidote : Dimercaprol

329
Q

Poisoning: Key features + Antidote

Lithium

A

Key features

  • Tremors
  • Ataxia
  • Nystagmus
  • Dysarthria
  • Muscle twitching + seizures

Management

  • Dialysis
330
Q

A 44 year old man with increasing shortness of breath and lethargy has a chest radiograph which demonstrates that the lungs are clear. However, there is right paratracheal lymphadenopathy. Serum ACE is elevated. A presumptive diagnosis of sarcoidosis is made for which treatment commencement is being considered.

a. Chest x-ray
b. CT pulmonary angiogram
c. Pulmonary function tests
d. Arterial blood gas sampling
e. High resolution CT scan of chest
f. Bronchoscopy
g. Transbronchial biopsy
h. Positron emission tomography (PET) scan (including ches

A

Pulmonary function tests
Sarcoidosis may have various clinical features, including bilateral hilar lymphadenopathy, erythema nodosum, pulmonary fibrosis, elevated serum ACE levels and more. The diagnosis of sarcoidosis can often be made on clinical grounds, particularily when a young adult presents with classical features such as the bilateral hilar lymphadenopathy and erythema nodosum.
Although a tissue biopsy would be diagnostic, lung function tests are often conducted first as they are less invasive. The lung function tests may be normal or show reducing lung volumes, impaired gas transfer and a restrictive ventilatory defect due to progression of the disease causing fibrosis of the lungs. Pulmonary function tests may then be performed and reviewed following steroid therapy to access response to treatment.

331
Q

A 45 year old patient with rheumatoid disease has complained of increasing shortness of breath over the past 9 months. 1 year previously her disease modifying anti-rhematoid drug was changed to methotrexate.

a. Chest x-ray
b. CT pulmonary angiogram
c. Pulmonary function tests
d. Arterial blood gas sampling
e. High resolution CT scan of chest
f. Bronchoscopy
g. Transbronchial biopsy
h. Positron emission tomography (PET) scan (including chest)

A

High resolution CT scan of chest
It is known that methotrexate may induce lung disease such as pulmonary fibrosis. With interstitial lung diseases such as those caused by methotrexate, high resolution CT (HRCT) imaging is the ideal imaging modality. As HRCT’s aim is to assess a generalized lung disease, the test is conventionally performed by taking thin sections 10-40 mm apart. The result is a few images which should be representative of the lungs in general, but which cover only approximately one tenth of the lungs. Because HRCT does not image the whole lungs (by using widely spaced thin sections), it is unsuitable for the assessment of lung cancer or other localised lung diseases.

332
Q

A 44 year old man has a screening chest x-ray for work placement in Australia. He has a 10 pack- year history of of smoking. He is asymptomatic. Chest x-ray shows a 2cm right upper lobe nodule with calcification within.

a. Bronchial carcinoma
b. Round pneumonia
c. Lung abscess
d. Pulmonary infarct
e. Arteriovenous malformation

f. Bronchogenic cyst
g. Pulmonary harmatoma
h. Aspergilloma

A

Pulmonary harmatoma

This is the most common benign tumour of the lung and is usually seen on chest x-ray as a very well defined round lesion 1-2 cm in diameter in the periphery of the lung. They occur as solitary round nodules with no predilection for any lobe. Growth is extremely slow, but the tumour can eventually reach several centimeters in diameter. Harmatomas are more common in men and in smokers. Peripheral nodules are asymptomatic, although endobronchial lesions are frequently associated with symptoms or signs of obstruction. As such, they must be distinguished from malignant neoplasms or recognized as the cause of a pneumonia or atelectasis. They usually contain fat and ‘popcorn’ calcification.

333
Q

A 27 year old asymptomatic lady has a screening chest x-ray for her new employment. There is a single right middle lobe lesion on chest x-ray. Her only medical history is of admission to hospital with a brain abscess 6 months ago.

a. Bronchial carcinoma
b. Round pneumonia
c. Lung abscess
d. Pulmonary infarct
e. Arteriovenous malformation

f. Bronchogenic cyst
g. Pulmonary harmatoma
h. Aspergilloma

A

Arteriovenous malformation
Despite the presumed congenital origin of AVMs, the clinical presentation most commonly occurs in young adults. Pulmonary arteriovenous malformations are more common in females and may present with primarily neurologic manifestations when pulmonary symptoms are absent or unrecognized. Brain abscess, embolic stroke, and hemorrhage from concomitant brain AVMs are well-recognized complications. Whenever a pulmonary AVM is suspected, the presence of a right-to- left shunt should be confirmed by the performance of a 100% oxygen study, contrast-enhanced echocardiography, or radionuclide perfusion lung scanning. Hereditary haemorrhagic telangectasia is stongly associated with AVMs.

334
Q

A 45-year-old man is found to have deranged liver function tests by his GP. As part of the investigations, the GP requested viral hepatitis serology:

Hepatitis A antibody - Negative

Hepatitis B core antibody - Positive

Hepatitis B surface antigen - Negative

Hepatitis C antibody - Positive

The following conclusion that can be made about the patient is:

A) He is a carrier for hepatitis A
B) He is a carrier for hepatitis B
C) He is a carrier for hepatitis C
D) He has been vaccinated against hepatitis B

E) He does not have viral hepatitis

A

He is a carrier for hepatitis C

Hepatitis A (HAV) and hepatitis E (HEV) are hepatitis viruses that do not have a chronic carrier state – you either survive the infection or will die from it. If someone is HAV antibody positive then it means that they have had the infection in the past but have survived it but are not currently infected with it.
Hepatitis B (HBV), hepatitis C (HCV) and hepatitis D (HDV) can cause chronic infection (although HDV can only occur in the presence of ongoing HBV infection). HBV infection is diagnosed by looking for the presence of surface antigen (HBsAg). If HBsAg is positive then the patient has HBV infection – if this is present for more than 6 months the patient is said to have chronic HBV infection. HBV core antibody (HBcAb) gives an indication of previous infection or natural immunity. In this case, the patient has had HBV infection in the past but has managed to clear it. Individuals who are given HBV vaccination are given a recombinant version of HBsAg and NOT core antibody or core antigen (HBcAb or HBcAg). If you are HBcAb positive (like this patient), then there is no benefit from giving the patient additional HBV vaccination. Patients who are immune from HBV, either through active or passive immunity, will be HBV surface antibody (HBsAb) positive.

Unlike HBV, the serological diagnostic test for HCV is the presence of antibody in the blood; hence if you are HCV antibody positive then you are infected with HCV as is the case here. The next test to be done would now be HCV PCR RNA to characterize the HCV viral load (i.e. how much virus the patient has) and also the genotype, because this will affect treatment duration and prognosis.

335
Q

A 56-year-old woman was referred to the gastroenterology clinic because of watery diarrhoea for the past 4 months. She opens her bowels up to 15 times per day but with no blood per rectum. This is associated crampy abdominal pain but no nausea or vomiting. Her weight has dropped by 7 kilograms in that time. Recently she has developed episodes of facial and neck flushing which could last several minutes, but resolved spontaneously. There is no other previous medical or drug history and she has not travelled abroad recently. On examination, she was apyrexial, but had some generalised abdominal tenderness and 2 cm hepatomegaly.The most likely diagnosis is:

A) Carcinoid syndrome

B) Colorectal carcinoma

C) Cushing’s syndrome

D) Giardiasis

E) Whipple’s disease

A

Carcinoid syndrome
This patient has abdominal pain, diarrhoea and flushing which are the classical features of carcinoid syndrome. Bronchospasm can also occur, but not very often if the tumours occur in the mid-gut which is the commonest location. Carcinoid tumours produce a variety of vasoactive amines (such as 5-HT, noradrenaline and dopamine), peptides (such as bradykinin, vasoactive intestinal peptide and substance P) and prostaglandins which account for the symptoms. These vasoactive products are inactivated by the liver which is why gastrointestinal carcinoid syndrome only occurs when hepatic metastases arise (the likely cause of her hepatomegaly). Whipple’s disease is an extremely rare disease characterized by abdominal pain, diarrhoea, weight loss and arthralgia (typically affecting large joints). It is caused by a bacterial infection. Giardia is unlikely in a patient who has not travelled abroad.

336
Q

A 24 year old male with tiredness. His full blood count reveals an anaemia (Hb 9.5 g/dL) and severe thrombocytopenia (platelet count 5x 10^9/L). He reports fevers and headaches in the last 2 days and passing dark brown urine. His creatinine is raised at 150 umol/L. While waiting in the ED, he has a seizure.

A. Howell-Jolly bodies

B. Auer rods
C. Spherocytes
D. Smear cells

E. Heinz bodies
F. Sickle cells
G. Red cell agglutination
H. Atypical lymphocytes
I. Hypersegmented neutrophils

J. Red cell fragments

Choose the most relevant and likely blood film abnormality.

A

Red cell fragments

This patient has features that suggest TTP (thrombotic thrombocytopenic purpura), which is a haematological emergency associated with a high mortality due to cardiac and CNS micro-vessel ischaemia resulting from fibrin-rich clots.

It is characterized by a pentad

  1. MAHA (micro-angiopathic haemolytic anaemia, characterized by red cell fragmentation and polychromasia)
  2. Thrombocytopenia
  3. renal impairment
  4. Fever
  5. Neurological symptoms.

Note that the pentad is not always present and is not necessary for the diagnosis. The differential includes other causes of MAHA, including malignancy, sepsis (meningococcal sepsis needs to be excluded here and the patient empirically treated after blood cultures are taken as an LP would be contraindicated with his low platelet count), malignant hypertension and the haemolytic uraemic syndrome (HUS). Treatment of TTP is with immunosuppression and plasma exchange. Platelets should not be administered.

337
Q

A man who had an emergency splenectomy after an RTA.

A. Howell-Jolly bodies

B. Auer rods
C. Spherocytes
D. Smear cells

E. Heinz bodies
F. Sickle cells
G. Red cell agglutination
H. Atypical lymphocytes
I. Hypersegmented neutrophils

J. Red cell fragments

A

Howell-Jolly bodies

Howell-Jolly bodies are small blue round red cell RNA inclusions. These are usually removed by the spleen as the cell matures, such that a hypofunctioning or absent spleen leads to persistence of these nuclear remnants. Other features of hyposplenism include a raised platelet count, red cell shape abnormalities and a lymphocytosis. The patient should receive penicillin V prophylaxis and be immunized against encapsulated organisms (e.g Neisseria, Pneumococcus and Haemophilus).

338
Q

A 12 year old Caucasian boy with chronic anaemia and a history of neonatal jaundice and who has recently developed episodes of RUQ pain. His sister required a splenectomy at the age of 10.

A. Howell-Jolly bodies

B. Auer rods
C. Spherocytes
D. Smear cells

E. Heinz bodies
F. Sickle cells
G. Red cell agglutination
H. Atypical lymphocytes
I. Hypersegmented neutrophils

J. Red cell fragments

A

Spherocytes

The history suggests an inherited anaemia given the long history, neonatal jaundice and the fact that his sister required a splenectomy. The common causes that present with neonatal jaundice include HS and G6PD deficiency. The latter, although associated with Heinz bodies, does not usually require splenectomy and is rarer in the Caucasian population, being more common in the Mediterranean and African population. The diagnosis is therefore likely to be HS, in which numerous spherocytes are seen, which appear as hyperdense round red cells without a clear central pale area. He has probably developed pigment gallstones from the increased red cell turnover causing his abdominal pain. One would expect a raised unconjugated bilirubin.

339
Q

A 70 year old presents with lymphadenopathy and splenomegaly and investigations reveal an IgM paraprotein of 40g/L. Hb 10g/L. Creatinine and calcium normal. Skeletal survey shows no bone lesions.

A. Myeloma
B. Acute promyelocytic leukaemia
C. Burkitt lymphoma
D. Follicular lymphoma
E. Chronic lymphocytic leukaemia
F. Chronic myeloid leukaemia
G. Acute lymphoblastic leukaemia
H. Waldenstrom’s macroglobulinaemia
I. Monoclonal gammopathy of unknown significance

J. Acute myeloid leukaemia
H. Myelodysplasia

A

Waldenstrom’s macroglobulinaemia

A paraprotein is most often seen in myeloma, although a few other lymphoproliferative disorders can be associated with this. True IgM myeloma is very rare (frequency: IgG>IgA>IgD>IgM). More likely is that this patient has a type of lymphoma (lymphoplasmacytic lymphoma) producing excess IgM. Collectively the syndrome is called Waldenstrom’s macroglobulinaemia, which usually also presents with bone marrow infiltration, splenomegaly and sometimes enlarged lymph nodes, but not lytic bone lesions or hypercalcaemia. The IgM can cause hyperviscosity due to its size.

340
Q

A 10 year old child with sickle cell disease presents with right sided hemiparesis. A CT head shows an evolving infarct. Hb 9.0 g/dl.

A. Iron chelation
B. Top-up transfusion
C. Exchange transfusion
D. Hydroxyurea
E. Hydration, oxygen and analgesia
F. Admit and IV cephalosporin
G. Anticoagulation
H. Bone marrow transplantation
I. HDU/ITU management with trial of non-invasive ventilation

J. Iron replacement therapy
K. No treatment required

A

Exchange transfusion

Strokes are a major cause of morbidity in young children with sickle cell disease. One should reduce the % Hb S as much as possible (ideally to <20%) with an exchange transfusion, which usually replaces most of the patient’s own sickle Hb containing red cells with Hb A containing adult red cells through a process of serial dilution with donor blood, by taking out 1 unit and replacing with another etc, whilst maintaining euvolaemia. If top-up transfusion is used, one would make the patient hyperviscous and risk further strokes. Patients who have had strokes are usually treated with chronic regular transfusions every month or so to suppress their marrow from making the abnormal HbS containing red cells. Iron overload and red cell alloimmunisation are problems that need to be managed in these patients in the longer term.

341
Q

Four hour acute painful bony crisis in a 23 year old with frequent sickle crises.

A. Iron chelation
B. Top-up transfusion
C. Exchange transfusion
D. Hydroxyurea
E. Hydration, oxygen and analgesia
F. Admit and IV cephalosporin
G. Anticoagulation
H. Bone marrow transplantation
I. HDU/ITU management with trial of non-invasive ventilation

J. Iron replacement therapy
K. No treatment required

A

Hydration, oxygenation and analgesia

In the immediate setting one would manage this patient with adequate oral hydration unless the patient cannot drink (IV in that case), analgesia (patient should have a protocol, but morphine s/c or orally is usually needed; pethidine should not be used), oxygen if saturations are <95%, and searching for and treatment of infective causes if present or if patient has a fever. Incentive spirometry is useful and encourages the small airways to be kept patent by getting the patient to blow against resistance. The patient should be monitored closely for adequate pain relief and any respiratory deterioration. In the longer term, this patient may benefit from hydroxyurea.

342
Q

A 19 year old student presents with abdominal pains. She has just returned from her gap year in which she went backpacking across India with some of her friends. She also complains of being constipated & is suffering from a ‘dry cough’. On examination she has an enlarged spleen.

A Uteric colic

B Typhoid

C Aortic aneurysm

D Acute pancreatitis

E Gall stones

F Peptic ulcer

G Ruptured ectopic pregnancy

H Appendicitis

I Sickle cell crisis

J MI

A

Typhoid

Typhoid is caused by Salmonella enterica and its various serovars.

Initially presents with fluctuating fever and bradycardia as well as malaise, headache, dry cough and epistaxis. There can be leukopenia and lymphocytosis.

Later you can develop delirium, Rose spots, COBH and hepatosplenomegaly.

343
Q

A 50 year old man presents with a 24 hour history of sudden onset of severe epigastric pain, which has now become generalised. On examination, he has pyrexia of 37.80C, a rigid abdomen & absent bowel sounds. White blood cell count is 18.09/l. Serum amylase is raised at 450IU/L. An x-ray of his abdomen shows opacities in the right hypochondrium & gas under the diaphragm.

A Diverticulitis

B Perforated peptic ulcer

C Small bowel obstruction

D Aortic dissection

E Ruptured aortic aneurysm

F Ruptured ectopic pregnancy

G Acute cholecystitis

H Acute pancreatitis

I Mesenteric infarction

J Meckel’s diverticulum

A

Perforated peptic ulcer.

The serum amylase is only slightly elevated. In acute pancreatitis it is normally elevated 4 fold and can be up into the 1000s. Perforation will result in the air under the diaphragm. If the perforation is posteriorly it can involve the gastroduodenal artery.

344
Q

A 30 year old unconscious man with diarrhoea & 30% burns.

A Paget’s disease

B Hypercalcaemia

C Hyperkalaemia

D Osteoporosis

E Hypocalcaemia

F Hypernatraemia

G Low serum zinc

H Hypokalaemia

I Osteomalacia

J Hyponatraemia

A

Hypernatremia

This patient has suffered extensive fluid losses due to the diarrhoea and burns. This has results in an osmotic imbalance resulting in hypernatremia. This can present as lethargy, weakness and irritability culminating in seizures and coma.

345
Q

A 20 year old lady who took excessive amounts of laxatives presents with cramps & muscle weakness.

A Paget’s disease

B Hypercalcaemia

C Hyperkalaemia

D Osteoporosis

E Hypocalcaemia

F Hypernatraemia

G Low serum zinc

H Hypokalaemia

I Osteomalacia

J Hyponatraemia

A

Hypokalemia.

This lady has suffered extensive fluid losses due to the laxatives and has deveoped hypokalemia. This presents with muscle weakness, cramping and arrhythmias.Other causes include excessive salbutamol or insulin.

ECG changes: long PR, ST depression, flat/TWI, long QT and U wave

346
Q

A 60 year old man with anorexia, tiredness, abdominal pain & weight loss. He also has a low potassium & raised serum alkaline phosphatase.

A Paget’s disease

B Hypercalcaemia

C Hyperkalaemia

D Osteoporosis

E Hypocalcaemia

F Hypernatraemia

G Low serum zinc

H Hypokalaemia

I Osteomalacia

J Hyponatraemia

A

Hypercalcemia.

347
Q

17 school girl, PC: pain, changed from dull ache to severe continuous throbbing pain. Breast feels hot on palpation, & she CO hot flushes. OE whole breast tender & engorged & nipple is tender.

  • Ca breast
  • Nodularity
  • Mastalgia
  • Fibroadenoma
  • Breast abscess
  • Ductal papilloma
  • Pregnancy
  • Brodie’s disease
A

Breast abscess.

Brodie’s tumour = Phyllodes tumour: large, fast growing fibroepithelial mass from periductal stromal cells. Accounts for 1% of breast cancers. Treatment is WLE.

348
Q

23 city female PC: tense heavy uncomfortable breast, prickling sensation deep inside breast. She has noticed enlargement & breast feels lumpy & subcut. veins are dilated. Skin of breast is warm.

  • Ca breast
  • Nodularity
  • Mastalgia
  • Fibroadenoma
  • Breast abscess
  • Ductal papilloma
  • Pregnancy
  • Brodie’s disease
A

Pregnant.

349
Q

V. high Ca; Low PO4; High PTH; Low ALP

A Primary hyperparathyroidism

B Hypercalcaemia of malignancy

C Osteomalacia

D Osteoporosis

E Hypoparathyroidism

F End stage renal failure

G Paget’s disease

H Immobility

I Sarcoidosis

J Myeloma

A

Primary hyperparathyroidism

350
Q

High Ca; High PO4; Low PTH; High ALP

A Primary hyperparathyroidism

B Hypercalcaemia of malignancy

C Osteomalacia

D Osteoporosis

E Hypoparathyroidism

F End stage renal failure

G Paget’s disease

H Immobility

I Sarcoidosis

J Myeloma

A

Hypoparathyroidism

351
Q

A 65 year old woman has multiple tense blisters with underlying erythema, which started on her arms & legs & now involves her trunk. Skin biopsy shows a spilt at the level of the basement membrane.

A Trauma

B Impetigo

C Herpes zoster

D Erythema multiforme

E Insect bites

F Fixed drug eruption

G Dermatitis herpetiformis

H Porphyria cutznea tarda

I Epidermolysis bullosa

J Pemphigoid

K Pemphigus

L Steven-Johnson syndrome

A

Pemphigoid

Pemphigoid is an AI bullous skin disease. There is no acantholysis (loss of intercellular connections between keratinocytes) as seen in pemphigus. It is mainly caused by IgG although IgA autoAb can occur. It is associated more with women over 60.

352
Q

A 68 year old Asian woman has multiple blisters on her trunk & a few on her limbs. The blisters are fragile & most have ruptured, leaving erythematous scaly patches. Pressure on an intact blister seems to cause it to spread. Her mouth is very sore & eating is difficult.

A Trauma

B Impetigo

C Herpes zoster

D Erythema multiforme

E Insect bites

F Fixed drug eruption

G Dermatitis herpetiformis

H Porphyria cutznea tarda

I Epidermolysis bullosa

J Pemphigoid

K Pemphigus

L Steven-Johnson syndrome

A

Pemphigus

Pemphigus is an AI bullous disease affecting the skin and mucous membranes. AutoAb to desmoglein cause destruction of the desmosomes that act as intercellular connections between keratinocytes. This results in acantholysis.

There are 2 main types: Pemphigus vulgaris and pemphigus foliaceus

353
Q

A 60 year old woman treated with total thyroidectomy for thyroid carcinoma now presents with visual changes. On examination, she has bitemporal hemianopia. CT of the head shows a cystic lesion compressing the optic tracts.

A Arteriovenous malformation

B Berry aneurysm

C Brain abscess

D Extradural haematoma

E Subdural haematoma

F Carotid artery occlusion

G Pituitary adenoma

H Medulloblastona

I Craniopharyngioma

J Meningioma

K Arteriovenous malformation

L Metastatic carcinoma

A

Craniopharyngioma

A slow growing tumour that arises from the pituitary stalk. Can be cystic in nature.

354
Q

A 30 year old man has been picked up in the street by police. He was initially drowsy but is now agitated & aggressive. His trousers are wet with urine.

A Dementia

B Delerium tremens

C Post-ictal state

D Hypoxia

E Cerebrovascular accident

F Hypoglycaemia

G Hypothermia

H Encephalitis

I Urinary tract infection

J Intoxication

K Acute psychosis

L Hypothyroidism

A

Intoxication.

355
Q

A 30 year old man had an appendicectomy 2 days ago & is now agitated & confused. He is sweaty & has a marked tremor of his hands. He claims that his sleep was disturbed by insects in his bed.

A Dementia

B Delerium tremens

C Post-ictal state

D Hypoxia

E Cerebrovascular accident

F Hypoglycaemia

G Hypothermia

H Encephalitis

I Urinary tract infection

J Intoxication

K Acute psychosis

L Hypothyroidism

A

UTI

356
Q

A 75 year old woman was found on the floor at home having collapsed. She is drowsy & confused & has been incontinent of urine. She is shaking. Her pulse is 50 beats per min & her ECG shows J waves.

A Dementia

B Delerium tremens

C Post-ictal state

D Hypoxia

E Cerebrovascular accident

F Hypoglycaemia

G Hypothermia

H Encephalitis

I Urinary tract infection

J Intoxication

K Acute psychosis

L Hypothyroidism

A

Hypothermia

J waves are characteristic of hypothermia. The size of the J wave roughly correlates with the severity of the hypothermia. It is also known as an Osborn wave.

357
Q

A 55 year old man who takes bendrofluazide for hypertension, presents with a 2 month history of watery diarrhoea with occasional blood & mucus mixed in the stool. He has serum potassium of 2.3mmol/l.

A Food poisoning

B Ulcerative colitis

C Crohn’s disease

D Diverticular disease

E Irritable bowel syndrome

F Drug-induced diarrhoea

G Salmonella infection

H Villous adenoma of the rectum

I Cryptosporidium infection

J E Coli infection

K Coeliac disease

L Chronic pancreatitis

A

Villous adenoma of the rectum.

Adenomas are divided into 3 subtypes based on histologic criteria, as follows: (1) tubular, (2) tubulovillous, and (3) villous. According to World Health Organization (WHO) criteria, villous adenomas are composed of greater than 80% villous architecture. Tubular adenomas are encountered most frequently (80-86%). Tubulovillous adenomas are encountered less frequently (8-16%), and villous adenomas are encountered least frequently (5%)

Villous adenomas are associated more often with larger adenomas and more severe degrees of dysplasia. These adenomas occur more frequently in the rectum and rectosigmoid, although they may occur anywhere in the colon. They generally are sessile structures that appear as velvety or cauliflowerlike projections.

Note that the vast majority of patients are asymptomatic and have unremarkable laboratory findings. Approximately two thirds of colorectal polyps are asymptomatic.

The most common presenting symptom is occult/overt bleeding (hematochezia) with an anemia, which may be microcytic. Polyps may bleed only intermittently into the stromal component, thus accounting for inconsistent findings.

Nonspecific symptoms include diarrhea, constipation, and flatulence.

358
Q

A 33 year old airline steward presents with a 1 week history of fever, dry cough, & shortness of breath. On examination, he is tachypnoeic. His lungs are clear to auscultation.

A Anaemia

B Valvular disease

C Bronchial asthma

D Atelectasis

E Atypical pneumonia

F Bronchial carcinoma

G Acute pulmonary oedema

H Pulmonary embolus

I Exacerbation of chronic bronchitis

J Metastatic carcinoma

A

Atypical pneumonia.

Atypical pneumonia often presents with atypical generalized symptoms such as fever, myalgia, and headache. Often there are no signs of lobar consolidation as the infection is restricted to small areas.

359
Q

An 18 year old girl attends family planning clinic, having recently started contraception. On examination she has a systolic murmur & weak foot pulses.

A Essential hypertension

B Renal artery stenosis

C Polycystic kidney disease

D Phaemochromocytoma

E Cushing’s syndrome

F Conn’s syndrome

G Acromegaly

H Coarctation of the aorta

I Hydronephrosis

J “White-coat” hypertension

K Drug induced

A

Coarctation of the aorta.

Congenital narrowing of the aorta where the ductus arteriosus joins. Presents in adults with HTN, SOB(OE), headache, muscle weakness, cold peripheries and weak peripheral pulses. Can present with a systolic murmur.

360
Q

A 55-year-old obese smoker presents with pain in his legs on walking 800 metres, which is immediately relieved by rest. His ankle-brachial pressure index is 0.9.

Managment

A.
Conservative management

B.
Femoro-popliteal bypass

C.
Embolectomy

D.
Anticoagulation

E.
Amputation

F.
Percutaneous transluminal angioplasty

G.
Fasciotomy

H.
Femoral-femoral crossover graft

I.
Antiplatelet drug

J.
Thrombolysis

K.
Endarterectomy

L.
Aorto-bifemoral bypass

A

Conservative management

This is peripheral vascular disease with classic symptoms of claudication (in reality, these classic symptoms only occur in a small minority of patients). ABPI should be performed in symptomatic patients and a result less than or equal to 0.9 is diagnostic for the presence of peripheral vascular disease. You need to however bear in mind that this test may not be accurate if the patient has non-compressible arteries (mainly in diabetic patients). This patient has only presented with claudication which is not severely lifestyle limiting. It depends on how much needing to rest every 800 metres or so bothers him. If he does not feel that this is really a functional disability then no additional treatment is required, but follow-up appointments with a doctor should be made to monitor the development of ischaemic symptoms or coronary and cerebrovascular complaints.

If the symptoms are lifestyle limiting then the patient should undergo a supervised exercise programme (only some rather limited quality cohort studies at the moment show an improvement in walking time and symptoms) and medication for symptomatic relief for a period of 3 months. Medication can include cilostazol, pentoxifylline (widely used but no more effective than placebo in RCTs) or naftidrofuryl. Risk factors should also continually be targetted – BP control, statins to lower LDL, beta blockers to target cardiovascular risk and antiplatelet therapy, for instance. If no improvement is made with this regime then patients should be referred to a vascular specialist to have their anatomy defined and assessed for possible revascularisation.

361
Q

A 65-year-old female presents with sudden-onset pain in her left calf. Although her patient notes are unavailable, she tells you that she is taking digoxin and verapamil for her ‘funny’ heart beat. On examination, the left leg is pale, cold and painful.

Management

A.
Conservative management

B.
Femoro-popliteal bypass

C.
Embolectomy

D.
Anticoagulation

E.
Amputation

F.
Percutaneous transluminal angioplasty

G.
Fasciotomy

H.
Femoral-femoral crossover graft

I.
Antiplatelet drug

J.
Thrombolysis

K.
Endarterectomy

L.
Aorto-bifemoral bypass

A

Embolectomy

Have a think about the differential diagnosis of sudden onset limb pain. Do you remember the 6 Ps of critical limb ischaemia? This patient’s arrhythmia has caused an embolic event, leading to acute limb ischaemia. There is as a result a sudden decrease in limb perfusion with threatened tissue viability. An emergency vascular assessment needs to be done with duplex ultrasound. Treatment depends on whether the patient already has a history of significant atherosclerosis. If so, there will already be a built up collateral supply so there is potentially a longer time window to act and so anticoagulation and thrombolysis are options. Otherwise an embolectomy will be indicated with a Fogarty catheter if there is not a long enough time window. This is typically done by inserting a Fogarty catheter with an inflatable balloon attached to its tip into the offending atery and passing the tip beyond the clot. The balloon is then inflated and then the catheter is withdrawn to remove the clot.

362
Q

A 62-year-old lorry driver presents with sudden-onset weakness of the right side of his body as well as ipsilateral loss of vision on the left, which he describes as like a ‘curtain’ descending over his field of vision. His symptoms resolve completely a few minutes later

Investigation?

  • Duplex doppler ultrasound
  • CT scan
  • Ankle-brachial pressure index
  • No investigation necessary
  • Venography
  • Contrast angiography
  • Magnetic resonance venography
  • ESR
  • Coagulation studies
  • Brain MRI
  • Blood glucose level
  • EMG walking test
  • Serum CK
A

Duplex dopper USS

This man is presenting with classic features of a TIA. A TIA is colloquially called a ‘mini stroke’ with symptoms typically lasting under an hour (and resolve within 24 hours). An antiplatelet drug such as aspirin is effective secondary prevention if the patient is not already anticoagulated. The patient will be anticoagulated if they have a likely or known cardioembolic source such as AF. Clopidogrel is an alternative in those who do not tolerate aspirin.
The description of transient visual disturbance like a curtain descending over the eye is characteristic of amaurosis fugax. Amaurosis fugax is a transient and painless loss of vision in one eye due to the passage of an embolus into the central retinal artery. The cause could be embolic from the internal carotid artery to cause an occlusion of the ipsilateral retinal artery. Patients presenting with TIAs should be investigated for carotid artery stenosis with a carotid Doppler ultrasound as there is a high risk of having a subsequent full blown stroke. Furthermore if there is a stenosis of >70%, the patient may be a candidate for carotid endarterectomy. Presence of ipsilateral carotid stenosis suggests artery-to-artery embolic event as the cause and this should be the target for surgical or interventional treatment. Follow up tests could be CT angiography or MRA to expand on the abnormal Doppler results. They are not appropriate first line investigations to do here. Head CT is usually normal in TIA. ECG should also be done to investigate for AF which is a common risk factor for embolic cerebral ischaemia.

363
Q

A 65-year-old man is brought into A&E after his son witnessed him collapse in his home. He reveals how his father complained of an excruciating pain in his lower back. On examination, the patient is pale and cold with shut-down perpheries. There is a palpable epigastric mass.

Investigation?

  • Duplex doppler ultrasound
  • CT scan
  • Ankle-brachial pressure index
  • No investigation necessary
  • Venography
  • Contrast angiography
  • Magnetic resonance venography
  • ESR
  • Coagulation studies
  • Brain MRI
  • Blood glucose level
  • EMG walking test
  • Serum CK
A

No investigation

This is a history of a ruptured AAA. There is back pain here and shut down peripheries and pallor due to blood loss. This patient is in haemorrhagic shock. As this AAA has ruptured, this man will need urgent surgical repair, with of course standard resuscitation measures. Investigations would just waste time. The airway will needed to be managed with supplemental oxygen and ET intubation, a central venous catheter will need to be inserted, an arterial catheter and urinary catheter will also be needed for monitoring, and the target systolic BP is 50-70. Infusing too many fluids may increase the risk of death. The most effective form of surgical repair is an EVAR (endovascular AAA repair), anatomy permitting, otherwise traditional open repair is performed. Open repair has a mortality of 48%. Antibiotics will also be needed to cover bacteria to prevent graft infection. This will be prescribed in line with local protocols.

364
Q

A 39-year-old multiparous woman presents to the clinic with varciosities in both legs. Although asymptomatic, she wishes to undergo surgery to remove them as they are causing her great embarassment. In order to determine the best treatment plan, the surgeon would like to map out all the incompetent venous pathways.

Investigation?

  • Duplex doppler ultrasound
  • CT scan
  • Ankle-brachial pressure index
  • No investigation necessary
  • Venography
  • Contrast angiography
  • Magnetic resonance venography
  • ESR
  • Coagulation studies
  • Brain MRI
  • Blood glucose level
  • EMG walking test
  • Serum CK
A

Duplex doppler USS

Ablative procedures include stripping and ligation, the aim of which is to permanently remove the varicose vein. Radiofrequency ablation (RFA) can also be done, as well as endovenous laser therapy and foamed sclerotherapy. Phlebectomy or sclerotherapy can also be performed. This is generally reserved for symptomatic cases, although this woman has a cosmetic issue with the appearance of her legs which is causing her distress. There are complications of ablation which the patient will need to be made aware of though, such as bleeding, infection, saphenous nerve injury and neovascularisation.
A duplex ultrasound is the investigation which is required here. It can assess reversed flow and valve closure time. This should be done with the patient standing and with the leg in external rotation for best sensitivity. Specific segments which are affected by reflux can be delineated as the superficial and deep truncal veins, perforators and tributaries can all be visualised. Reflex in the great saphenous or common femoral can be detected with Valsalva while more distal reflux can be elicitied by compressing the leg above the Doppler probe to see if any blood is being forced back towards the feet.

365
Q

You see a 75-year-old gentleman in clinic who has been followed up for his foot ulcers for over 10 years. He tells you during the consultation that he has recently had problems sleeping at night with leg pain waking him up. He notices that it helps if he hangs his foot over the edge of the bed.

A.
Rhabdomyolysis

B.
Intermittent claudication

C.
Acute limb ischaemia

D.
Spinal stenosis

E.
Ankylosing spondylitis

F.
Critical limb ischaemia

G.
Deep vein thrombosis

H.
Viable limb

I.
Dead limb

J.
Baker’s cyst

K.
Leriche syndrome

L.
Compartment syndrome

M.
Muscle tear

A

Critical limb ischaemia

Is this patient’s ulcer an arterial ulcer with diabetic neuropathy, or a diabetic ulcer? Difficult to tell. However, this patient does have critical limb ischaemia. Critical limb ischaemia is chronic severe limb ischaemia. This patient has rest pain which is relieved by hanging the leg dependent – this is classical. These patients have chronic ischaemic symptoms of the leg such as ischaemic rest pain, gangrene and non-healing wounds. Ischaemic cause of these symptoms needs to be established urgently, or if the patient already has a documented history of PVD, then they can be referred to a vascular surgeon for revascularisation. Their arterial anatomy needs to be defined and assessed. Risk factors also need to continue to be aggressively targetted. If they are not a candidate for revascularisation then the patient should be assessed for amputation where necessary and be on appropriate risk factor reduction medication.

366
Q

A 65-year-old man is referred to you for an emergency appointment after attending his GP earlier that morning. You are called by the nurse to the waiting room, where you see this patient looking pale, sweaty and unwell. You establish that he has pain in his chest radiating to his back. You call a crash team. Later on, you find out that his Troponin I was 0.032 and that he was taken to theatre.

Diagnosis

A.
Myocardial infarction

B.
Perforated duodenal ulcer

C.
Pyelonephritis

D.
Renal colic

E.
Mesenteric infarction

F.
Pancreatitis

G.
Metastatic disease

H.
Addison’s disease

I.
Hepatitis

J.
Ruptured abdominal aortic aneurysm

K.
Volvulus

L.
Spinal stenosis

M.
Dissecting aortic aneurysm

N.
Pericarditis

A

Ruptured AAA

This is a history of a ruptured AAA. There is abdominal (which the patient states as chest) pain radiating around to the back here and pallor due to blood loss suggesting this diagnosis. As this AAA has ruptured, this man will need urgent surgical repair, with of course standard resuscitation measures. Investigations would just waste time although it seems he has had a troponin, and likely ECG too. Misdiagnosing this condition is pretty poor of the doctors with the history here of pain which radiates to the back. The airway will needed to be managed with supplemental oxygen and ET intubation, a central venous catheter will need to be inserted, an arterial catheter and urinary catheter will also be needed for monitoring, and the target systolic BP is 50-70. Infusing too many fluids may increase the risk of death. The most effective form of surgical repair is an EVAR (endovascular AAA repair), anatomy permitting, otherwise traditional open repair is performed. Open repair has a mortality of 48%. Antibiotics will also be needed to cover bacteria to prevent graft infection. This will be prescribed in line with local protocols.

367
Q

An ambulance crew is dispatched to attend a 999 call made by passers-by. A 71-year-old diabetic lady at a bus stop collapsed, could not get up and was complaining of back pain. After basic investigations, she was taken to hospital. Subsequent CT showed no abnormalities.

Diagnosis

A.
Myocardial infarction

B.
Perforated duodenal ulcer

C.
Pyelonephritis

D.
Renal colic

E.
Mesenteric infarction

F.
Pancreatitis

G.
Metastatic disease

H.
Addison’s disease

I.
Hepatitis

J.
Ruptured abdominal aortic aneurysm

K.
Volvulus

L.
Spinal stenosis

M.
Dissecting aortic aneurysm

N.
Pericarditis

A

MI

Given the list of options here, this is likely to be an atypical MI which is more common in diabetics and the elderly, likely to be due to autonomic neuropathy. They are known as ‘silent’ MI and should be excluded in all causes of collapse. An ECG is indicated here after the clear CT scan (although may have already been done on admission by alert staff). STEMI, new LBBB or confirmed posterior MI is an indication for PCI/thrombolysis. It is worth noting that RV infarction is present in 40% of inferior infarcts so in this case, right sided ECG leads should also be obtained.

368
Q

A 52 year old otherwise fit and healthy man is found to have a 6.3cm AAA. He is very surprised and requests that it is treated so he does not die suddenly in the future.

Intervention

A.
Alpha blocker

B.
Methyldopa

C.
Ultrasound

D.
Open repair of aneurysm

E.
Endovascular aneurysm repair

F.
Femoral-distal bypass

G.
Angioplasty

H.
Embolectomy

I.
Angiography

J.
Endarterectomy

K.
Aortobifemoral bypass

A

Open repaur of aneurysm

Incidental finding of a large AAA requires elective surgical repair (exceeding 5.5cm in men, 5cm in women – repair of aneuryms greater or equal to 5.5cm offers a survival advantage). Additionally, rapid increase in size is also an indication for elective repair. Young and healthy patients, particularly women, may benefit from early repair of smaller AAAs. (>5cm). Data suggests EVAR is equivalent to open repair in terms of overall survival but there is a higher rate of secondary interventions with EVAR. Therefore younger and healthier patients may benefit more from open repair. Elderly patients and those with co-morbidities: an EVAR is the best way forward here. However note also that EVAR could entail a complication of endovascular repair leak, which would require corrective treatment. Endoleak is persistent blood flow outside the graft and within the aneurysm sac. There is 24% risk after EVAR. However, this is not a complication of open repair, which is probably preferred in most cases in those who are fit and healthy enough to have it such as this patient. Management of this complication would depend on the type of endoleak.

369
Q

A 62-year-old male undergoes a CT abdominal scan for investigation of renal calculi. You notice a 4.9cm infra-renal AAA.

Management of AAA

A.
Ultrasound in 6 months

B.
Semi-urgent surgical repair

C.
Ultrasound in 1 or more years

D.
Urgent surgical repai

r

E.
Elective EVAR (endovascular AAA repair) 

F.
Aggressive fluid resuscitation

G.
Abdominal XR

H.
Palliative care

I.
Elective open repair

A

USS in 6 months.

Incidental finding of a large AAA requires elective surgical repair (exceeding 5.5cm in men, 5cm in women – repair of aneuryms greater or equal to 5.5cm offers a survival advantage). Additionally, rapid increase in size is also an indication for elective repair. Young and healthy patients, particularly women, may benefit from early repair of smaller AAAs. (>5cm). Data suggests EVAR is equivalent to open repair in terms of overall survival but there is a higher rate of secondary interventions with EVAR. Therefore younger and healthier patients may benefit more from open repair.

  • For asymptomatic small AAA, surveillance is indicated. Infra and juxtarenal AAAs between 4.0-5.4cm in diameter (bear in mind that young and healthy patients with >5cm may benefit from repair!) should be monitored by USS or CT every 6-12 months. There is good quality evidence that the risk of rupture is 20% for aneurysms larger than 5.0-6.0cm in diameter.
  • AAAs <4.0 cm require USS every 2-3 years
370
Q

Sildenafil is a PDE-4 inhibitor which acts on arterial wall receptors

True or False

A

False

Sildenafil inhibits PDE-5

371
Q

30% of strokes result can be attributed to carotid artery disease

True or False

A

True

372
Q

Amaurosis fugax results from an embolus in the retinal vein

True or False

A

False

Retinal artery NOT vein

RTFQ

373
Q

Shunts are often used in carotid endarterectomy surgery

True or False

A

False

Shunting during CEA has been a subject of controversy with some advocating its use, whilst others arguing it has no noticeable benefit and may even put the patient at risk of complications. In the procedure itself, the carotids are clamped and the lumen of the internal carotid artery is opened longitudinally. To maintain cerebral perfusion, some surgeons use a temporary shunt. The stenotic plaque is then removed from the carotid artery, which is then sutured directly or patched.

374
Q

If whilst on aspirin a patient has a stroke, adding dipyridamole is indicated

True or False

A

True

Dipyridamole is a PDE inhibitor. It affects NO and inflammatory cytokines; overall, it prevents thrombus formation. Bleeding risk.

375
Q

5 most common cancers to metastasis to bone?

A

Kidney, Thyroid, Lung, Prostate, Breast

Kids Trying Lunges Perform Badly.

376
Q

For each of the tumours below, select the most likely causative carcinogen. Each option may be used once, more than once or not at all.

Prostate carcinoma

A.
Asbestos

B.
Oestrogen

C.
Cadmium

D.
Aniline dyes

E.
Azo dyes

F.
Aflatoxin B1

G.
Epstein-Barr virus (EBV)

A

Cadmium

Studies have shown that men in certain occupations, with higher levels of exposure to cadmium, have a higher risk of prostate cancer. It is thought that cadmium interacts with certain hormone receptors and mimics the effects of oestrogen and testosterone, thus abnormally stimulating the growth of the prostate. Furthermore cadmium is thought to interfere with the body’s ability to absorb zinc (which is believed to have a protective effect against prostate cancer.)

377
Q

A 2-year-old boy presents with abdominal distension, haematuria and vague pain. Examination reveals a large nodular mass on the left kidney.

Eponymous neoplasm:

A.
Pancoast tumour

B.
Burkitt’s lymphoma

C.
Grawitz’s tumour

D.
Kaposi’s sarcoma

E.
Hodgkin’s lymphoma

F.
Brodie’s tumour

G.
Ewing’s sarcoma

H.
Wilm’s tumour

A

Wilm’s tumour

Wilms’ tumour (a.k.a. nephroblastoma) is the commonest intra-abdominal tumour in children, rarely occuring in adults. It is a malignant tumour derived from embryonic mesodermal tissues. Patients present with a loin mass, weight loss, anorexia and fever. Although the tumour is rapidly growing an behaves aggressively, it has an 80% survival rate at 5 years.

Can you identify the others?

378
Q

A 35-year-old woman comes to see you in the clinic. She is noticeably distressed and complains of a bulky mass in her left breast which has grown rapidly over the past month. On examination you notice that the contour of the breast has been distorted and the overlying skin is red and tender. Core biopsy reveals mixed connective tissue and epithelial elements.

Eponymous neoplasm:

A.
Pancoast tumour

B.
Burkitt’s lymphoma

C.
Grawitz’s tumour

D.
Kaposi’s sarcoma

E.
Hodgkin’s lymphoma

F.
Brodie’s tumour

G.
Ewing’s sarcoma

H.
Wilm’s tumour

A

Brodie’s tumour

Brodie’s tumour (a.k.a. phyllodes tumour) are rare tumours of the fibroepithelial stroma of the breast. The history of a rapidly growing mass that distorts the shape of the breast points towards this disease. Most of these tumours are benign and the prognosis after surgery is excellent.

Can you identify the others?

379
Q

A 45-year-old woman presents with an irregular 4cm firm lump in the left breast. O/E the lump is fixed to the overlying skin and their is bloody discharge from the left nipple.

Tumour marker:

A.
CA 19-9

B.
BRCA-1

C.
α-Fetoprotein

D.
Carcinoembryonic antigen

E.
Prostatic acid phosphatase

F.
Tyrosinase

G.
CA 15-3

H.
Prostate-specific antigen

A

Ca 15-3

High Ca 15-3 levels suggest metastatic breast disease. However, it has poor sensitivity and is therefore not used as a screening procedure to detect breast carcinoma. This tumour marker has recently been superseded by Ca 27-29, which is claimed to be more sensitive and specific. However, CA 27-29 lacks predictive value in the earliest stages of breast cancer and thus has no role in screening for or diagnosing the malignancy.

380
Q

A 65 year old male undergoing chemotherapy for acute myeloid leukaemia starts to deteriorate on the ward. He complains of a tingling sensation in his fingers and around his lips. He also complains of muscle weakness and appears confused. His biochemistry reveals that he is hyperkalaemic, hyperphosphataemic, hyperuricaemic and hypocalcaemic. His creatinine is also elevated to 300 umol/L.

Extra-pulmonary findings of lung cancer:

A.
SIADH

B.
Meningitis

C.
Spinal cord compression

D.
Superior vena caval obstruction

E.
Anaphylaxis

F.
Ectopic PTH secretion

G.
Tumour lysis syndrome

H.
Raised intracranial pressure

I.
Chemotherapy complication

A

Tumour lysis syndrome

Tumour lysis syndrome usually occurs in patients undergoing chemotherapy for lymphoproliferative malignancies, who have just initiated treatment. Lysis of tumour cells leads to the release of large amounts of potassium, phosphate and uric acid into the circulation. The excess phosphate binds to calcium, leading to hypocalcaemia and its clinical features. Patients are also at risk of developing acute renal failure due to the deposition of uric acid and calcium phosphate crystals in the renal tubules.

381
Q

A 70-year-old man complains of weight loss, headache, blurry vision and haematuria. O/E you notice cervical lymphadenopathy and splenomegaly. Bone marrow biopsy shows a lymphoplasmacytoid cell infiltrate with few plasma cells.

Diagnosis

A.
Acute lymphoblastic leukaemia

B.
Waldenstrom’s macroglobulinaemia

C.
Chronic lymphocytic leukaemia

D.
Myeloma

E.
Acute myeloid leukaemia

F.
Chronic myeloid leukaemia

G.
Hodgkin’s lymphoma

H.
Monoclonal gammopathy of unknown significance

I.
Myelofibrosis

J.
Non-Hodgkin’s lymphoma

K.
Burkitt’s lymphoma

A

Waldenstrom’s macroglobulinaemia

Waldenstrom’s macroglobulinaemia is a lymphoproliferative disorder of B cells, which take on a lymphoplasmacytoid appearance. It is characterised by the production of immunoglobulin M (IgM), which gives rise to the clinical features of hypervisosity (nosebleeds, blurred vision, retinal haemorrhage etc.)

382
Q

A 7-year-old boy is brought to the GP by his mother. She claims he has been very lethargic for the past few months and occasionally has a fever. FBC reveals Hb 9.3 g/dL, WCC 82 x 109/L and platelets 30 x 109/L

Diagnosis

A.
Acute lymphoblastic leukaemia

B.
Waldenstrom’s macroglobulinaemia

C.
Chronic lymphocytic leukaemia

D.
Myeloma

E.
Acute myeloid leukaemia

F.
Chronic myeloid leukaemia

G.
Hodgkin’s lymphoma

H.
Monoclonal gammopathy of unknown significance

I.
Myelofibrosis

J.
Non-Hodgkin’s lymphoma

K.
Burkitt’s lymphoma

A

ALL

Acute lymphoblastic leukaemia (ALL) is the most common leukaemia in children. The immature blast cells infiltrate the marrow and lymphoid tissue causing anaemia, bleeding and a vulnerability to infection. Although the FBC reveals that the patient is anaemic, the WCC is raised due to circulating blast cells. The investigation of choice is bone marrow aspiration (which shows a hypercellular marrow with >20% blasts) and treatment is with combination chemotherapy.

383
Q

EMQ features:

CLL -

CML -

ALL -

AML -

A

CLL: enlarged rubbery lymph nodes, non tender, also smear cells

CML: Philadelphia chromosome, sweats, more likely splenomegaly, increased WCC

ALL: bruising, pale, usually children btw 2-4 years old

AML: Auer rods, SOB, bone pain.

384
Q

Glucagonoma

For each of the malignancies listed below, please select the recognised presentation from the list of options.

A.
Necrolytic migratory erythema

B.
Erythrocytosis

C.
Acanthosis nigricans

D.
Autoimmune haemolytic anaemia

E.
Troisier’s sign

F.
Hypoglycaemia

G.
Eaton-Lambert syndrome

H.
Tetany

I.
Erythema ab igne

A

Necrolytic migratory erythema

This rash is characteristic of glucagonoma and is also known as ‘glucagonoma syndrome’. It is characterised by the spread of erythematous blisters across the lower abdomen, buttocks, perineum and groin.

385
Q

An 82-year-old woman presents with constipation, lower abdominal pain and a feeling of incomplete emptying. She looks emaciated and has lost 5 kgs over the past month.

Tumour marker:

A.CA 19-9
B.BRCA-1

C.α-Fetoprotein

D.Carcinoembryonic antigen

E.Prostatic acid phosphatase

F. Tyrosinase

G.CA 15-3

H.Prostate-specific antigen

A

CEA

Although carcinoembryonic antigen (CEA) lacks the sensitivity and specificity to be a diagnostic test for colorectal cancer, it has found a valuable application in the detection of recurrence of malignant disease following treatment.

386
Q

A pleomorphic adenoma typically arises in the parotid gland.

True or False

A

True

387
Q

A 24-year-old Cypriot man presents with a worsening anaemia and
jaundice. A blood film taken several weeks after the incident shows
the presence of Heinz bodies.

A autoimmune haemolytic anaemia
B spherocytosis
C pyruvate kinase deficiency
D aplastic anaemia
E thalassaemia minor
F glucose-6-phosphate dehydrogenase
deficiency
G thalassaemia
H iron deficiency anaemia
I sickle-cell anaemia
J folate deficiency
K sideroblastic anaemia
L pernicious anaemia

A

G6PDH deficiency

Glucose-6-phosphate dehydrogenase (G6PDH) deficiency is the most
common red blood cell enzyme defect and is more common in patients
of Mediterranean origin. Individuals with this condition are susceptible
to oxidative crises precipitated by fava beans and drugs such as
ciprofloxacin and sulphonamides. G6PDH plays an important role in the
hexose monophosphate shunt which provides NADPH (reduced nicotinamide
adenine dinucleotide phosphate – the reducing agent). NADPH
is used to regenerate glutathione. In the absence of glutathione, red blood
cells are exposed to oxidative stress. Heinz bodies represent oxidized
haemoglobin.

388
Q

A 50-year-old man presents with tiredness, dyspnoea and paraesthesia.
On examination he showed extensor plantars, brisk knee jerks and
absent ankle jerks. His blood film shows macrocytic anaemia.

A autoimmune haemolytic
anaemia
B spherocytosis
C pyruvate kinase deficiency
D aplastic anaemia
E thalassaemia minor
F glucose-6-phosphate dehydrogenase
deficiency
G thalassaemia
H iron deficiency anaemia
I sickle-cell anaemia
J folate deficiency
K sideroblastic anaemia
L pernicious anaemia

A

Pernicious anaemia

Pernicious anaemia is the most common cause of vitamin B12 deficiency.
This is an autoimmune condition characterized by atrophy of the gastric
mucosa with subsequent failure of intrinsic factor production. Intrinsic
factor is required for vitamin B12 absorption. Other causes of vitamin B12
deficiency are terminal ileum disease, gastrectomy and low dietary intake
(e.g. vegans).

389
Q

Presence of Burr cells in a patient being treated in the ICU with
multiple organ failure.

A vitamin B12 deficiency anaemia
B thalassaemia
C hypothyroidism
D haemophilia B
E uraemia
F sickle-cell anaemia
G hyposplenism
H hypersplenism
I bone marrow failure
J iron deficiency anaemia
K anaemia of chronic disease
L none of the above

A

Uraemia

Burr cells indicate uraemia.

390
Q

Howell–Jolly bodies in a patient with coeliac disease.

A vitamin B12 deficiency anaemia
B thalassaemia
C hypothyroidism
D haemophilia B
E uraemia
F sickle-cell anaemia
G hyposplenism
H hypersplenism
I bone marrow failure
J iron deficiency anaemia
K anaemia of chronic disease
L none of the above

A

Hyposplenism

Howell–Jolly bodies are found in hyposplenism. They are basophilic nuclear remnants found in RBCs not mopped by the spleen.

391
Q

A 27-year-old man presents with haemolytic anaemia after surgery.
He reports a history of recurrent abdominal pains. The Ham’s test is
positive.

A myeloma
B Hodgkin’s lymphoma
C polycythaemia rubra vera
D amyloidosis
E chronic lymphocytic leukaemia
F Waldenström’s macroglobulinaemia
G acute lymphoblastic leukaemia
H Burkitt’s lymphoma
I paroxysmal nocturnal haemoglobinuria
J essential thrombocythaemia
K chronic myeloid leukaemia
L antiphospholipid syndrome

A

Paroxysmal nocturnal haemoglobinuria

Paroxysmal nocturnal haemoglobinuria (PNH) is an acquired defect
where red cells are unusually sensitive to destruction by activated complement.
Patients may present non-specifically with recurrent abdominal
pains or with haemolytic anaemia. Venous thrombotic episodes are very
common in patients with PNH. The condition acquired its name from the
characteristic finding of dark-coloured urine when voiding at night or
early in the morning.

In Ham’s test, the cells from a PNH patient will lyse more readily in acidified
serum than normal red blood cells. Serum heated to around 56°C
will inactivate complement and thus haemolysis will not occur.
As a result of the high risk of venous thrombosis, long-term coagulation
may be necessary. Deficient haematopoeisis may also occur and PNH
may progress to aplastic anaemia in around 10–20 per cent individuals.

392
Q

A 60-year-old man presents with headaches, blurred vision and itching
over the whole body (the last after a hot bath). Positive findings
on examination include plethoric facies and moderate splenomegaly.
Haematocrit: 65 per cent.

A myeloma
B Hodgkin’s lymphoma
C polycythaemia rubra vera
D amyloidosis
E chronic lymphocytic leukaemia
F Waldenström’s macroglobulinaemia
G acute lymphoblastic leukaemia
H Burkitt’s lymphoma
I paroxysmal nocturnal haemoglobinuria
J essential thrombocythaemia
K chronic myeloid leukaemia
L antiphospholipid syndrome

A

Polycythemia Rubra vera

Polycythaemia rubra vera (PRV) is a primary polycythaemia where there
is excessive proliferation of red blood cells caused by a clonal disorder of
pluripotent stem cells. The condition usually presents in late middle age –
elderly patients with signs of hyperviscosity, e.g. blurry vision (retinal
haemorrhages), headaches, bleeding, etc. Severe pruritus after a hot bath
is a commonly reported finding. The patient typically has plethoric facies
and may be cyanotic as a result of stagnation and deoxygenation of the
blood in peripheral vessels. Splenomegaly is very common in PRV, unlike
secondary polycythaemia. Blood tests show an elevated packed cell
volume. The white cell count is usually raised. A raised red cell mass can
be shown with 51Cr studies.
The aim of treatment is to normalize the full blood count and prevent the
complications of thrombosis and haemorrhage. Venesection is the treatment
of choice. Hydroxyurea chemotherapy can also be used to control
thrombocytosis.

393
Q

A 65-year-old lung cancer patient with brain metastases is suffering
from severe headache. He has papilloedema in both eyes.

A hyoscine butylbromide
B hyoscine, midazolam
C paracetamol
D haloperidol, midazolam
E ondansetron
F dexamethasone
G metoclopramide, phenytoin,
hyoscine
H metoclopramide
I morphine
J ibuprofen
K amitriptyline
L co-dydramol

A

Dexamethasone

Dexamethasone is used in the short term to reduce the raised intracranial
pressure caused by cerebral oedema around metastases.

394
Q

A 55-year-old patient with metastatic cancer from an unknown primary
requires medication to treat convulsions, vomiting and terminal
restlessness

A hyoscine butylbromide
B hyoscine, midazolam
C paracetamol
D haloperidol, midazolam
E ondansetron
F dexamethasone
G metoclopramide, phenytoin,
hyoscine
H metoclopramide
I morphine
J ibuprofen
K amitriptyline
L co-dydramol

A

Haloperidol, midazolam

Midazolam is used to treat both convulsions and terminal restlessness.

395
Q

A 26 year old man presents with severe shortness of breath & dry cough which he has had for 24 hours. He is very distressed. He has been an IV drug user. The x-ray shows peri-hilar fine mottling.

A Bacteroides fragilis

B Coxiella burnetii

C Escherichia coli (Gram –ve)

D Haemophilus influenzae

E Legionella pneumophila

F Mixed growth of organisms

G Mycobacterium tuberculosis

H Mycoplasma pneumoniae

I Pneumocystis carinii

J Staphylococcus aureus

K Streptococcus pneumoniae

A

Pneumocystis carinii

Pneumocystis carinii pneumonia (PCP) is associated with HIV/AIDS. IVDUs are an at risk of developing this. It generally presents with a fever, dry cough and SOB with weight loss and night sweats.

396
Q

A 28 year old man complains of generalised pruritus worse at night. It is most marked in his pubic area & his forearms. There appear to be a few thin ‘tracks; between his knuckles.

A Urticaria

B Scabies

C Dermatitis herpetiformis

D Lichen planus

E Insect bite

F Iron deficiency

G Primary biliary cirrhosis

H Primary proliferative polycythaemia

I Hodgkin’s disease

J Hyperthyroidism

K Hypothyroidism

L Chronic renal failure

A

Scabies

Contagious skin infection caused by Sarcoptes scabiei. This mite burrows under the skin causing intense allergic itching. This is typically worse when warm and at night. Can present with linear burrow tracks.

Immunosuppressed patients can develop crusted scabies as their skin becomes overrun by breeding mites.

Treat with Permethrin and anti-histamines.

397
Q

A 35 year old woman presents with purpura, fever, haemolytic anaemia, microvascular thrombosis, renal failure & mild stroke. Blood film shows fragmented red cells & clotting screen is normal.

A Amyloidosis

B Cushing’s disease

C Disseminated intravascular coagulation (DIC)

D Ehlos-Danlos syndrome

E Haemophilia

F Henoch-Schonlein syndrome

G Iatrogenic

H Idiopathic thrombocytopenic purpura (ITP)

I Paroxysmal nocturnal haemoglobinuria (PNH)

J Scurvy

K Thrombotic thrombocytopenic purpura (TTP)

A

Thrombotic thrombocytopenic purpura (TTP)

This rare coagulopathy results in extensive microscopic thrombi that result in end-organ damage. There is intravascular haemolysis of RBCs due to shear stress resulting from the thrombi (MAHA).

It is mainly due to inhibition of the ADAMTS13 enzyme, an MMP that activates vWF.

398
Q

A 45 year old man presents with a painless swelling in his scrotum. His left testis cannot be felt. The swelling transilluminates.

A Hydrocoele

B Varicocoele

C Epididymal cyst

D Seminoma

E Teratoma

F Chronic epididymitis

G Mumps orchitis

H Acute epididymitis

I Torsion of the testes

J Inguinal hernia

K Testicular gumma

L Scrotal haemotoma

A

Hydrocoele

399
Q

A 40 year old man presents with a painless swelling in his scrotum which is fluctuant & transilluminable. Both testes are easily palpable.

A Hydrocoele

B Varicocoele

C Epididymal cyst

D Seminoma

E Teratoma

F Chronic epididymitis

G Mumps orchitis

H Acute epididymitis

I Torsion of the testes

J Inguinal hernia

K Testicular gumma

L Scrotal haemotoma

A

Epididymal cyst.

400
Q

A 65 year old woman has a 3 day history of a painful blistering rash on her left outer thigh. She has recently been diagnosed with breast cancer.

A Erythema nodosum

B Erythema multiforme

C Henoch-Schonlein purpura

D Eczema

E Psoriasis

F Necrobiosis lipoidica

G Ringworm (tinea corporis)

H Shingles (herpes zoster)

I Urticaria

J Pemphigus

K Pemphigoid

A

Shingles (herpes zoster)

A localised band of painful blistering rash is associated with shingles. A re-emergence of VZV. This may have occurred as this lady is immunocompromised.

401
Q

A 65 year old woman has developed extensive blistering particularly involving her legs. The blisters are fragile & seem to spread under the skin surface when pressed. She feels generally unwell but has no other symptoms.

A Erythema nodosum

B Erythema multiforme

C Henoch-Schonlein purpura

D Eczema

E Psoriasis

F Necrobiosis lipoidica

G Ringworm (tinea corporis)

H Shingles (herpes zoster)

I Urticaria

J Pemphigus

K Pemphigoid

A

Pemphigus

A bullous skin disease associated with autoAg IgG to desmoglein in the desmosomes (intercellular connections between keratinocytes). Acantholysis can occur, this is NOT seen in bullous pemphigoid.

There are 2 main types of pemphigus: vulgaris and foliaceus

402
Q

A 35 year old man has developed a slowly progressive right-sided facial palsy with deafness & tinnitus. As well as facial asymmetry, he is unable to adduct his right eye. His father had been similarly affected.

A Stroke

B Brainstem tumour

C Multiple sclerosis

D Acoustic neuroma

E Otitis media

F Cholestertoma

G Bell’s palsy

H Ramsey-Hunt syndrome

I Parotid tumours

J Trauma

K Post-meningitis

L Sarcoidosis

A

Brainstem tumour

403
Q

Pain on breathing in & out, dyspnoea, coughing up blood as well, stony dull to percuss.

A Aortic dissection

B Anxiety

C PE

D Angina

E Pneumonia

F Ruptured AAA

G Pleural effusion

H Ca oesophagus

I TB

K GORD

A

PE

404
Q

The concordance of Type 2 Diabetes in identical twins is approximately 40%.

True or False

A

False

In Type 2 Diabetes the concordance in identical twins is nearly 100 %. It is type 1 diabetes that has a concordance of approximately 40%. Thus type 2 diabetes is undoubtedly more heritable than type 1 diabetes, which might be a surprise to some of you.

405
Q

A 43 year old male smoker presents to his GP complaining of dizziness and feeling light headed over the past week. He has had type 2 diabetes for 20 years and his HbA1c shows that this is poorly controlled.

  • Onset type 1 diabetes
  • Onset type 2 diabetes
  • Hyperosmolar hyperglycaemic state
  • Cataract
  • Gastroparesis
  • Autonomic neuropathy
  • Hypoglycaemia
  • Nephropathy
  • Peripheral neuropathy
  • Ischaemic heart disease
  • Ketoacidosis
  • Peripheral arterial disease
  • Retinopathy
  • Necrobiosis lipoidica
A

Autonomic neuropathy

This is diabetic autonomic neuropathy which typically occurs later in the disease course. Patients have a reduced response in HR and BP and postural hypotension occurs due to efferent sympathetic denervation (reduced vasoconstriction of peripheral vascular beds). Patients can feel week, faint, dizzy and light headed and may faint on standing. There may also be GI symptoms from vagal dysfunction. Bladder dysfunction and erectile dysfunction can also occur. Women can complain of a reduced libido and increased pain during sex

406
Q

A 57 year old lorry driver attends a regular appointment with his optician. On fundoscopy, dots & blots are noted, in addition to hard exudates around the macula.

A. Preproliferative retinopathy

B. Extradural haematoma

C. Horner’s syndrome

D. Diabetic background retinopathy

E. Subdural haemorrhage

F.Hypercholesterolaemia
G. Acute glaucoma

H. Meningitis

I. Conjuntivitis

J. Paraneoplastic syndrome

K.Macular degeneration
L. Oculomotor palsy

M.Migraine

A

Diabetic background retinopathy

This patient has a microvascular complication of diabetes – diabetic retinopathy. His macular involvement is an indication for macular laser therapy. Digital methods for photographing the fundus are preferred over fundoscopy. Microaneurysms, cotton wool spots, haemorrhages, hard/lipid exudates and neovascularisation may all be seen depending on the stage. Young onset T1DM is more associated with retinopathy, and additional risks include a longer duration of DM and poor glycaemic control compounded by renal disease and hypertension. There are also ethnic variations in risk.
Background retinopathy initially consists of just microaneurysms, which progresses with small haemorrhages (dots and blots and flames). There may also be hard exudates. However, where these changes occur within the macula, this is maculopathy and is potentially sight-threatening. Maculopathy can be classified, academically, into focal, fissue and ischaemic, though this is of limited clinical relevance. Pre-proliferative is associated with cotton wool spots, typically multiple (arguably, the odd one can be consistent with background). Proliferative is associated with new vessel formation, which can be quite subtle. The patient can also be asymptomatic but is at a risk of vitreous haemorrhage. Diabetic retinopathy needs to be screened for (either dilated fundoscopy in clinic or with retinal photography) at least on an annual basis. Treatment is with photocoagulation where a transpupillary laser is used to burn the retina, which reduces overall oxygen demand lessening the ischaemic stimulus. There are complications, which you can look up. Please have a look at some photographs of retinal images. Reading about it will not help you to identify a cotton wool spot, hard exudate or angiogenesis in real life. There are also variations of normal which you will be able to spot with the more images you look at, like racial pigmentation and drusen in the elderly. What would you expect to see on fundoscopy in a patient with age-related macular degeneration?

407
Q

Which one of the following is the rarest presentation of diabetic neuropathy?

  • Peripheral pain
  • Reduced or absent ankle reflexes
  • Carpal tunnel syndrome
  • Peripheral loss of sensation
A

Carpal tunnel syndrome

Much rarer presentations of DN include mononeuropathies like CTS as well as cranial neuropathies, diabetic truncal radiculoneuropathy and diabetic amyotrophy.

408
Q

Which one of the following statements is incorrect concerning diabetic retinopathy?

  • Flame haemorrhages are an indication for panretinal laser therapy
  • Lipid exudates can be seen in diabetic retinopathy
  • Cotton wool spots can be seen in non-proliferative diabetic retinopathy
  • Macular oedema can cause gradual vision loss
A

Flame haemorrhages are an indication for panretinal laser therapy.

Flame haemorrhages are a feature of non-proliferative diabetic retinopathy.

409
Q

Which one of the following statements is incorrect concerning T1DM?

  • T1DM is more common in Asians than European

s

  • Pancreatic beta cells are destroyed mostly by immune-mediated mechanisms
  • Lifelong insulin therapy is always required in T1DM
  • T1DM can present with blurred vision
A

T1DM is more common in Asians than Europeans.

Unlike T2DM the opposite is actually true in T1DM which is more common in Europeans than in Asians.

410
Q

Which one of the following statements is false?

  • A patient with polyuria, nocturia and a single random blood glucose of 12.1 has diabetes mellitus
  • Fructosamine provides a longer measure of blood glucose levels
  • The first line oral hypoglycaemic drug is metformin
  • Microalbuminuria is the earliest detectable sign of diabetic nephropathy
A

Fructosamine provides a longer measure of blood glucose levels.

Fructosamine is a glycated plasma protein which provides information on glucose levels over the previous 1-3 weeks.

411
Q

True or False

A palpable breast lump in a woman aged 40 years: is most likely to be a fibroadenoma

A

False

No, cysts/carcinoma are more likely. Think of fibroadenomas in age group 20-30.

412
Q

True or False

A palpable breast lump in a woman aged 40 years: May be ductal carcinoma in situ

A

False

DCIS is impalpable.

413
Q

True or False

A palpable breast lump in a woman aged 40 years: May be associated with a positive paternal family history

A

True

Positive FH is one of the RFs for development of breast cancer.

414
Q

True or False

Palpable breast cysts: Are most common in the fourth decade

A

False

It is most common in 20-40 age group.

415
Q

True or False

Paget’s disease of the breast: May lead to secondary sarcoma development

A

False

Paget’s disease (of the bone) can do so. Paget’s disease of the nipple is a unilateral eczema of the nipple due to destruction of the nipple by ductal spread of carcinoma.

416
Q

A 65-year-old woman presents with vertigo, vomiting and dysphagia.
On examination there is a left-sided Horner’s syndrome, and loss of
pain and temperature on the right side of the face.

A vertebrobasilar ischaemia
B benign positional vertigo
C vestibular schwannoma
D lateral medullary syndrome
E multiple sclerosis
F Menière’s disease
G auditory eczema
H acoustic neuroma
I vestibular neuronitis
J Ramsay Hunt syndrome
K migraine
L perilymph fistula
M hyperventilation

A

Lateral medullary syndrome

The posterior inferior cerebellar artery (PICA) supplies the dorsal lateral
medullary plate and parts of the posterior medial cerebellum. PICA
thrombosis results in lesions affecting cranial nerves, descending tracts
and the cerebellum that are known collectively as ‘lateral medullary
syndrome’.
Ipsilateral signs include:
– decreased pain and temperature sensation of the face (descending tract
and nucleus of nerve V)
– palatal, vocal fold paralysis (nerve IX, X involvement)
– Horner’s syndrome (sympathetic tract)
– cerebellar signs, e.g. ataxia, nystagmus.
Contralateral signs include:
– decreased pain and temperature sensation of the body (spinothalamic
tract)
– lateral medullary syndrome, eponymously known as Wallenberg’s
syndrome.

417
Q

A 20-year-old man presents with vertigo having suffered with symptoms
of flu for several days.

A vertebrobasilar ischaemia
B benign positional vertigo
C vestibular schwannoma
D lateral medullary syndrome
E multiple sclerosis
F Menière’s disease
G auditory eczema
H acoustic neuroma
I vestibular neuronitis
J Ramsay Hunt syndrome
K migraine
L perilymph fistula
M hyperventilation

A

Vestibular neuronitis.

Vestibular neuronitis is associated with a preceding infection/illness. The
vertigo is severe and presents abruptly with vomiting but no
deafness/tinnitus. The vertigo usually diminishes after a few days and in
many cases there is complete recovery within a couple of weeks. Some
individuals, however, develop benign paroxysmal positional vertigo
afterwards. There can also be a horizontal nystagmus.

Unlike labyrinthitis there are no auditory symptoms.

418
Q

A 60-year-old woman complains of bilateral proximal muscle weakness
in the legs and dysphagia. On examination she has a purple rash
on her cheeks.

A myasthenia gravis
B Charcot–Marie–Tooth disease
C polymyalgia rheumatica
D motor neuron disease
E Guillain–Barré syndrome
F syringomyelia
G alcohol-induced neuropathy
H Parkinson’s disease
I polymyositis
J multiple sclerosis
K bilateral stroke
L neurofibromatosis

A

Polymyositis

There are many non-neurological causes of weakness in the legs. Polymyositis
is an autoimmune disease characterized by non-suppurative
inflammation of skeletal muscle. In severe disease there can be respiratory
muscle weakness and also cardiac involvement. Creatine kinase (CK)
is usually markedly elevated. Electromyography (EMG) characteristically
shows fibrillation potentials. Dysphagia occurs in about 1/3 of patients.

419
Q

A 55-year-old man presents with bilateral progressive worsening
muscle weakness. There is marked wasting of lower limb muscles and
very brisk lower limb reflexes. Sensation is normal.

A myasthenia gravis
B Charcot–Marie–Tooth disease
C polymyalgia rheumatica
D motor neuron disease
E Guillain–Barré syndrome
F syringomyelia
G alcohol-induced neuropathy
H Parkinson’s disease
I polymyositis
J multiple sclerosis
K bilateral stroke
L neurofibromatosis

A

Motor Neuron Disease

Motor neuron disease (MND) is a progressive degenerative disease that
affects the upper and lower motor neurons (LMNs). There is characteristically
NO sensory involvement.
It is useful to be familiar with the following three common patterns of
involvement:
(i) Progressive muscular atrophy: anterior horn wasting lesion with
wasting often beginning in the distal muscles of the hand and then
spreading. Fasciculation is a common finding.
(ii) Amyotrophic lateral sclerosis: involvement of the lateral corticospinal
tract gives a progressive spastic teraparesis/paraparesis.
The presence of LMN signs, e.g. wasting, fasciculation, differentiates
this diagnosis from other causes of spastic weakness.
(iii) Progressive bulbar palsy: the lower cranial nerve nuclei and their
connections are primarily affected. Dysarthria and dysphagia are
common symptoms. This pattern of disease is more common in
women than in men.

420
Q

A patient complains of a hoarse voice, difficulty in swallowing and
choking when drinking fluids. Examination reveals visible weakness of
elevation of the palate, depression of palatal sensation and loss of the
gag reflex.

A V, VII, VIII, IX lesion
B right XII lesion
C left XII lesion
D right XI lesion
E IX lesion
F IV, V, VII lesion
G V lesion
H left XI lesion
I V, VII, VIII lesion
J IX, X lesion
K VI lesion
L IV lesion

A

IX + X lesion

Cranial nerve IX and X lesions rarely occur in isolation and can also
accompany nerve XI and XII lesions after infarction in the brain stem or
as a result of pathology around the jugular foramen.

421
Q

A 75-year-old man with poorly controlled diabetes is distressed by
wild flinging of his right arm and right leg.

A tardive dyskinesia
B senile dystonia
C myoclonus
D chorea
E akathisia
F dysmetria
G hemiballismus
H Tourette’s syndrome
I tics
J pill-rolling tremor
K torticollis
L asterixis

A

Hemiballismus

A very rare movement disorder, caused in most cases by a decrease in activity of the subthalamic nucleus of the basal ganglia, resulting in the appearance of flailing, ballistic, undesired movements of the limbs. Usually the underlying pathology is infarction/haemorrhage in the contralateral
subthalamic nucleus. Can be associated with non-ketotic hyperglycaemia of diabetes.

422
Q

Grimacing and involuntary chewing in a 75-year-old woman on
long-term treatment with neuroleptics.

A tardive dyskinesia
B senile dystonia
C myoclonus
D chorea
E akathisia
F dysmetria
G hemiballismus
H Tourette’s syndrome
I tics
J pill-rolling tremor
K torticollis
L asterixis

A

Tardive dyskinesia

This is repetitive, involuntary automated movements such as grimacing, lip smacking and excessive blinking.

Tardive dyskinesia is a movement disorder that usually appears only
after long-term treatment with typical antipsychotic medications and
other drugs with dopamine antagonist activity, e.g. metoclopramide.
Withdrawal of the responsible drug can lead to improvement but the
condition is irreversible in some patients. Atypical antipsychotic
medications such as clozapine and risperidone are less likely to cause
extrapyramidal side effects and tardive dyskinesia.

423
Q

A 45-year-old woman complains of a pain behind the right ear and
that her mouth is sagging on the right-hand side.

A muscular dystrophy
B Shy–Drager syndrome
C chronic fatigue syndrome
D neurofibromatosis
E myasthenia gravis
F melanoma
G Sturge–Weber syndrome
H syringomyelia
I pseudobulbar palsy
J Bell’s palsy
K cervical spondylisthesis

A

Bell’s palsy

In a Bell’s palsy (lower motor neuron lesion affecting cranial nerve VII)
all the muscles on one half of the face are affected. There may also be
loss of taste sensation on the anterior two-thirds of the tongue because
the facial nerve supplies sensory innervation to that area via the chorda
tympani.
Improvement and complete recovery are common. EMG can often be a
good predictor of outcome. In an upper motor neuron lesion (caused by a
cerebrovascular accident or CVA, for example) there is sparing of the
forehead as a result of bilateral innervation. The patient will be able to
raise both eyebrows.

424
Q

A 62-year-old woman presents with a stiff spastic tongue and ‘Donald
Duck’-like speech. On examination she had a brisk jaw jerk and was
prone to laughing inappropriately.

A muscular dystrophy
B Shy–Drager syndrome
C chronic fatigue syndrome
D neurofibromatosis
E myasthenia gravis
F melanoma
G Sturge–Weber syndrome
H syringomyelia
I pseudobulbar palsy
J Bell’s palsy
K cervical spondylisthesis

A

Pseudobulbar palsy

Pseudobulbar palsy is an upper motor neuron lesion caused by bilateral
lesions of the lower cranial nuclei and can be associated with emotional
lability. It is known to occur in multiple sclerosis, motor neuron disease
and after bilateral strokes affecting the corticobulbar pathways.

425
Q

A 27-year-old woman presents with wasting and weakness of the small
muscles of the hand. Examination reveals loss of pain and temperature
sensation over the trunk and arms with intact vibration sense.

A muscular dystrophy
B Shy–Drager syndrome
C chronic fatigue syndrome
D neurofibromatosis
E myasthenia gravis
F melanoma
G Sturge–Weber syndrome
H syringomyelia
I pseudobulbar palsy
J Bell’s palsy
K cervical spondylisthesis

A

Syringomyelia

Syringomyelia is the term used to describe the development of a fluidfilled
cavity (syrinx) within the spinal cord. The condition may be associated
with an Arnold–Chiari malformation.
The expansion of the syrinx compresses surrounding tracts, giving rise to
a particular pattern of symptoms/signs that relates to the position of the
tracts within the spinal cord.
The pain and temperature modalities are characteristically affected first
as a result of compression of decussating spinothalamic fibres anteriorly
in the ventral horns. This loss of pain and temperature sensation is often
described as occurring in a ‘cape’ distribution (over trunk and arms).
Wasting and weakness of hands and arms reflects involvement of
cervical anterior horn cells.
Sensory loss can result in Charcot’s joints (neuropathic joints damaged
through loss of sensation). Involvement of the sympathetic trunk can
give rise to Horner’s syndrome.
The condition is gradually progressive and there is no known curative
treatment. Patients with an Arnold–Chiari malformation may be treated
with decompression at the foramen magnum to slow progression.

426
Q

A 15-year-old boy is found to have several coffee-coloured patches on
his body. Slit-lamp examination reveals the presence of Lisch nodules.

A muscular dystrophy
B Shy–Drager syndrome
C chronic fatigue syndrome
D neurofibromatosis
E myasthenia gravis
F melanoma
G Sturge–Weber syndrome
H syringomyelia
I pseudobulbar palsy
J Bell’s palsy
K cervical spondylisthesis

A

Neurofibromatosis

This is a presentation of type I neurofibromatosis (also known as von
Recklinghausen’s disease). It is an autosomal dominant condition characterized by decreased production of neurofibromin.
Diagnosis requires two of seven diagnostic criteria.
(i) six or more café-au-lait spots 5 mm (in children 10) or 15mm in adults
(ii) axillary/inguinal freckling
(iii) two or more neurofibromas or even one plexiform neurofibroma
(iv) optic glioma
(v) two or more Lisch nodules (iris hamartoma detected with slit-lamp
examination)
(vi) osseous lesion such as sphenoid dysplasia/pseudoarthrosis
(vii) family history in first-degree relative.

427
Q

A 60-year-old woman walks with a drop foot and high-stepping gait
shortly after hip replacement.

A myopathic gait
B cerebellar ataxia
C femoral nerve injury
D sciatic nerve injury
E Alzheimer’s dementia
F Parkinson’s disease
G sensory ataxia
H Pick’s dementia
I Huntington’s disease
J Sydenham’s chorea
K psychogenic gait
L spastic gait

A

Sciatic nerve injury

Sciatic nerve is derived from the anterior primary rami of L4, L5, S1, S2
and S3, and divides into the common peroneal and tibial nerves about
two-thirds of the way down the thigh.
Sciatic nerve injury is an important postoperative complication of hip
replacement and can result from direct trauma or traction on the nerve. There is lack of sensation over the lateral leg below the knee (and the
entire sole of the foot) and inability to dorsiflex or evert the foot.

428
Q

A broad-based high-stepping gait in a known alcoholic.

A myopathic gait
B cerebellar ataxia
C femoral nerve injury
D sciatic nerve injury
E Alzheimer’s dementia
F Parkinson’s disease
G sensory ataxia
H Pick’s dementia
I Huntington’s disease
J Sydenham’s chorea
K psychogenic gait
L spastic gait

A

Sensory ataxia

Sensory ataxia results from a loss of proprioception caused by peripheral
neuropathy (in this case secondary to alcohol abuse). Romberg’s test is
used to identify proprioceptive sensory loss by demonstrating loss of
postural control in darkness.
The patient is asked to stand with the feet together and to close his or
her eyes. If the patient is unstable, the test is said to be positive. These
patients sway and fall because they rely heavily on visual cues to
maintain postural equilibrium. This test is not pathognomonic of
proprioceptive loss because patients with bilateral vestibular damage
and some patients with cerebellar disease require visual cues to keep
their balance.

429
Q

A broad-based unstable gait with veering to the right side.

A myopathic gait
B cerebellar ataxia
C femoral nerve injury
D sciatic nerve injury
E Alzheimer’s dementia
F Parkinson’s disease
G sensory ataxia
H Pick’s dementia
I Huntington’s disease
J Sydenham’s chorea
K psychogenic gait
L spastic gait

A

Cerebellar ataxia

Cerebellar disease affecting one hemisphere tends to cause the patient
to veer towards the side of the lesion. If cerebellar disease affects the
cerebellar midline vermis, there is truncal ataxia giving rise to difficulty
in standing and even sitting on a stool unsupported.

430
Q

A 75-year-old, right-handed woman is noted to ignore stimuli on the
left side of her body.

A left occipital lobe
B right cerebellar lobe
C medial longitudinal fasciculus (MLF)
D non-dominant temporal lobe
E caudate nucleus
F right occipital lobe
G substantia nigra
H frontal lobe
I left parietal lobe
J dominant temporal lobe
K left cerebellar lobe
L right parietal lobe
M bilateral nerve VI lesion

A

Right parietal lobe

This patient has sensory neglect suggesting a non-dominant hemisphere
lesion. As the patient is right handed, we know that the dominant lobe
must be on the left side, and hence the pathology is in the right
parietal lobe.

431
Q

A 34-year-old woman with multiple sclerosis is found to have bilateral
internuclear ophthalmoplegia.

A left occipital lobe
B right cerebellar lobe
C medial longitudinal fasciculus (MLF)
D non-dominant temporal lobe
E caudate nucleus
F right occipital lobe
G substantia nigra
H frontal lobe
I left parietal lobe
J dominant temporal lobe
K left cerebellar lobe
L right parietal lobe
M bilateral nerve VI lesion

A

Medial longitudinal fasciculus

Bilateral internuclear ophthalmoplegia is almost exclusively found in
multiple sclerosis and is caused by bilateral involvement of the MLF.
When the patient is asked to look towards the right, the left eye fails to
adduct and the right eye develops coarse nystagmus in abduction. This is
caused by the left MLF lesion.
When the patient is asked to look towards the left, the right eye fails to
adduct and the left eye develops coarse nystagmus in abduction. This is
caused by the right MLF lesion.

432
Q

A 52-year-old man with ischaemic heart disease requires treatment
for an acute migraine attack that has not responded to paracetamol.

A amitriptyline
B morphine
C timolol
D paracetamol
E prednisolone
F evacuation via burr hole
G propranolol
H high-dose aspirin and metoclopramide
I pethidine
J acetazolamide
K carbamazepine
L sumatriptan
M hydrocortisone

A

Aspiring and metoclopramide

Aspirin and metoclopramide provide relief for the symptoms of pain and
nausea/vomiting associated with migraine. The use of serotonin 5HT1
agonists such as sumatriptan are indicated if the patient fails to respond
to conventional analgesics. Remember that the 5HT1 agonists are contraindicated in patients with coronary artery disease, previous myocardial
infarction (MI) and uncontrolled hypertension.

433
Q

A 70-year-old woman requiring treatment for troubling trigeminal
neuralgia.

A amitriptyline
B morphine
C timolol
D paracetamol
E prednisolone
F evacuation via burr hole
G propranolol
H high-dose aspirin and metoclopramide
I pethidine
J acetazolamide
K carbamazepine
L sumatriptan
M hydrocortisone

A

Carbemazepine

Carbamazepine is commonly used to treat trigeminal neuralgia.
Phenytoin and gabapentin have also been used to treat this condition.
Cases refractory to medical treatment may be considered for surgery.

434
Q

A 29-year-old man with a skull fracture is suffering from very severe
pain overlying the laceration to his head. He has been admitted for
overnight observation.

A amitriptyline
B morphine
C timolol
D paracetamol
E prednisolone
F evacuation via burr hole
G propranolol
H high-dose aspirin and metoclopramide
I pethidine
J acetazolamide
K carbamazepine
L sumatriptan
M hydrocortisone

A

Paracetamol

Opiates such as morphine cause pupillary constriction and should not be given as analgesia in this case of head injury because it will interfere with
neurological observations.

435
Q

A 55 year old known epileptic arrives in A&E having suffered a fit whilst shopping. She is “post ictal” on arrival in A&E & breathing in an obstructed manner with O2 saturation of 92% (on air).

A Oral airway & oxygen

B Laryngeal mask airway (LMA)

C Endotracheal tube

D None

E Facemask

A

Oral airway and oxygen.

An oropharyngeal tube (Guedel) or nasophargyneal tube.

436
Q

Agitation, euphoria, blurred vision, slurred speech, ataxia, slate-grey cyanosis.

A Tricyclic antidepressants

B Benzodiazepines

C Opiate analgesics

D Salicylates

E Sympathomimetics

F Methanol

G Ethylene glycol

H Insulin

I Carbon monoxide

J Antipsychotics

K Volatile solvents

A

Benzodiazepines.

Presents with CNS depression, impaired balance, ataxia and slurred speech. Developing into respiratory depression and coma.

Particularly at risk when taken with other drugs such as alcohol.

Antidote is Flumezanil.

437
Q

Nausea, vomiting, tinnitus, deafness, sweating, hyperventilation, tachycardia.

A Tricyclic antidepressants

B Benzodiazepines

C Opiate analgesics

D Salicylates

E Sympathomimetics

F Methanol

G Ethylene glycol

H Insulin

I Carbon monoxide

J Antipsychotics

K Volatile solvents

A

Salicylates

Mild overdose (150mg/kg): N+V, dizziness

Moderate overdose (150-300mg/kg): N+V, tinnitus, headache, confusion, hyperventilation, tachycardia, fever, metabolic acidosis

Severe overdose (>300mg/kg): delirium, hallucinations, convulsions, coma, respiratory arrest

Management: activated charcoal, alkaline diuresis (IV sodium bicarbonate), haemodialysis.

438
Q

Drowsiness, hypotension, hypoventilation, arrhythmias, convulsions, dystonia & dyskinesias.

A Tricyclic antidepressants

B Benzodiazepines

C Opiate analgesics

D Salicylates

E Sympathomimetics

F Methanol

G Ethylene glycol

H Insulin

I Carbon monoxide

J Antipsychotics

K Volatile solvents

A

Anti-psychotics

439
Q

Coma, increased muscle tone & reflexes, myoclonus, convulsions, tachycardia, dilated pupils.

A Tricyclic antidepressants

B Benzodiazepines

C Opiate analgesics

D Salicylates

E Sympathomimetics

F Methanol

G Ethylene glycol

H Insulin

I Carbon monoxide

J Antipsychotics

K Volatile solvents

A

Tricyclic anti-depressants

Initial symptoms: tachycardia, drowsiness, confusion, dry mouth, N+V, urinary retention, agitation, headache. Cholinergic effects.

Severe complications: hypotension, dysrhythmias, hallucinations and seizures.

440
Q

A 42-year-old man with a history of subcutaneous heroin use presented to the hospital with slurred speech, diplopia, and dysphagia. The physical examination showed bilateral ptosis, a sluggish pupillary response to light, bilateral sixth-cranial-nerve palsies, and multiple skin abscesses on his arms and legs. There was also descending symmetrical muscle weakness. Shortly after admission, the dysphagia progressed, necessitating intubation for airway protection.

Diagnosis?

A

Wound Botulism.

This patient developed skin abscesses due to ‘skin popping’ as an IVDU. These abscesses have become infected with Clostridium botulinum. It is possible that the heroin injected carried spores of botulinum. Botulinum toxin irreversibly disrupts the release of ACh at the NMJ. This results in bilateral cranial neuropathies with a descending muscle weakness.

441
Q

A 55 year old man with pain, numbness and tingling in his hands and feet. He is a known type 1 diabetic that is controlled with insulin. His HbA1c was measured at 50mmol/mol (normal is <48mmol/mol) ; B12 was 500pg/mol (200-900) and his eGFR was 90.

O/E There was loss of sensation to pin prick in the glove and stocking distribution.

Which would you prescribe?

A) Codeine

B) Duloxetine

C) Hydroxocobalamin

D) Paracetamol

E) Pregabalin

A

Duloxetine.

This patient has diabetic peripheral neuropathic pain. Duloxetine is first line treatment. Where this fails amitriptyline or pregabalin should be tried.

The neuropathic pain may respond to opioid analgesics and paracetamol but specialist supervision should be used for the former.

Hydroxocobalamin is for B12 replacement, of which this patient is not deficient.

442
Q

Causes of peripheral neuropathy:

  1. Metabolic (5)
  2. Infiltrative
  3. Infection
  4. AI (3)
  5. Neoplastic (2)
  6. Hereditary
  7. Drugs (CAMPING)
A
  1. Metabolic
    - DM
    - Alcohol and drugs
    - B12
    - Hypothyroidism
    - Uraemia
  2. Infiltrative
    - amyloidosis
  3. Infectious
    - HIV
  4. AI
    - Vasculitis
    - CTD
    - Inflammatory demyelinating neuropathy: Guillain Barre, CIDP
  5. Neoplastic
    - Paraneoplastic
    - Myeloma paraproteinaemia
  6. Hereditary sensory motor neuropathy = Charcot Marie Tooth disease
  7. Drugs
    - Cytotoxics, Amiodarone, Metronidazole, Phenytoin, Isoniazid, Nitrofurantoin, Gold.
443
Q

A 34 year old woman present with weakness in her legs and blurred vision. O/E her legs are hypertonic and hyperreflexive with reduced power. There is also reduced sensation. Fundoscopy reveals a blurred optic disc.

What is the cause of the blurred vision?

A) Amaurosis fugax

B) Anterior uveitis

C) Papilloedema

D) Papillitis
E) Vitreous haemorrhage

A

Papillitis

Papillitis is also known as optic neuritis. It presents with blurred optic disc on fundoscopy with blurred vision and painful eye movements. The most common cause is MS. Indeed it is the presenting sign in 20-30% of MS cases.

Whilst papilloedema also results in a blurred optic disc it does NOT result in blurred vision or painful eye movements. None of the other answers result in a blurred optic disc.

444
Q

A 60 year old type 2 diabetic man presents with pain and parasthesia in his antero-lateral thigh. His diabetes is poorly controlled with metformin, his HbA1c is 60 mmol/mol (<48mmol/mol). His BMI is 30 kg/m2.

What is the next most appropriate step in his management?

A. Lose weight

B. Insulin
C. Statin
D. Aspirin

E. MRI Brain

A

Lose weight.

This man has meralgia paraesthetica. This is a mononeuropathy as a result of compression of the lateral femoral cutaenous nerve. Initial management involves reassurance of teh patient, advice to avoid tight garments and weight loss.

If pain continues then carbemazepine or gabapentin can be used.

445
Q

A 65 year old man who had a AAA repaird 2 days ago has diffuse abdominal pain. His pulse is 120bpm and his respiratory rate is 30 breaths per minute.

His blood tests are likely to show:

A) Normal lactate

B) High amylase

C) High bicarbonate

D) High Na

E) High Ca

A

High amylase

Mildly elevated amylase can occur in any abdominal abnormality. In acute pancreatitis it will be exceptionally high and fit with the clinical picture.

446
Q

A 55 year old known alcoholic male presents confused and with abdominal pain. O/E he has signs of cirrhosis, abdominal distension, asterixis and shifting dullness.

Which of the following is consistent with SBP?

A) Ascites neutrophils ≥25 cells/mm3

B) Ascites neutrophils ≥50 cells/mm3

C) Ascites neutrophils ≥100 cells/mm3

D) Ascites neutrophils ≥250 cells/mm3

E) Ascites neutrophils ≥500 cells/mm3

A

Ascites neutrophils ≥250 cells/mm3.

This is the laboratory threshold for SBP.

Management with IV antibiotics, IV albumin and repeat paracentesis of the ascites in 48h. Also consider pro-kinetics.

447
Q

An 85 year old man presented with perforated diverticular disease in the sigmoid colon. After resuscitation he had an emergency procedure.

A Ileal conduit

B Defunctioning loop ileostomy

C End ileostomy

D Parastomal hernia

E Prolapsed colostomy

F Hartman’s procedure: End colostomy

G Loop colostomy

H Subtotal colectomy & end ileostomy

A

Hartman’s procedure: end colostomy.

Hartman’s procedure is surgical resection of the rectosigmoid colon with closure of the rectal stump and formation of an end colostomy. It was used to treat colon cancer or diverticulitis.

448
Q

A 65 year old man undergoes elective excision of a rectal carcinoma & primary anastomosis, he is left with a stoma.

A Ileal conduit

B Defunctioning loop ileostomy

C End ileostomy

D Parastomal hernia

E Prolapsed colostomy

F Hartman’s procedure: End colostomy

G Loop colostomy

H Subtotal colectomy & end ileostomy

A

Defunctioning loop ileostomy.

This stoma results in faecal diversion to protect to newly formed distal anastamosis. It will later be reverted.

449
Q

Child with severe learning difficulties, cleft lip & palate, polydactyly, multiple congenital heart defects.

A Down’s syndrome

B Edward’s syndrome

C Patan’s syndrome

D Klinefelter’s syndrome

E Turner’s syndrome

F Fragile X syndrome

G Fragile X syndrome

A

Edward’s syndrome

450
Q

A 37 year old man has had half his ear bitten off in a fight & is admitted under the plastic surgeons. After being on the ward for a day & a half he becomes extremely agitated, claiming to see spiders & snakes crawling up the walls. Examination shows him to be tachycardic & sweaty but is otherwise unremarkable.

A Subdural haematoma

B Dementia

C Hepatic encephalopathy

D Diabetic ketoacidosis

E Urinary tract infection

F Alcohol withdrawal

G Phenytoin toxicity

H Meningitis

I Schizophrenia

J Ecstasy overdose

A

Alcohol withdrawal

451
Q

A 72 year old lady under long term therapy for rheumatoid arthritis.

A Low ACTH, low cortisol

B Low ACTH, high cortisol

C High ACTH, high cortisol

D Normal ACTH, high cortisol

E Normal ACTH, low cortisol

F High ACTH, low cortisol

G Low ACTH, normal cortisol

H High ACTH, normal cortisol

A

Low ACTH, Low cortisol

Exogenous steroids. Produces Cushingoid features as well as suppressing ACTH and thereby endogenous cortisol.

452
Q

An 18 year old woman with a family history of cortisol insufficiency, but with unimpaired aldosterone production.

A Low ACTH, low cortisol

B Low ACTH, high cortisol

C High ACTH, high cortisol

D Normal ACTH, high cortisol

E Normal ACTH, low cortisol

F High ACTH, low cortisol

G Low ACTH, normal cortisol

H High ACTH, normal cortisol

A

Normal ACTH, Low cortisol

Aldosterone can partially suppress ACTH to normal levels despite low cortisol.

453
Q

A 25 year old man developed a rash 4cms in diameter on the right side of his chest. Several days later some smaller discrete lesions appeared on his trunk.

A Eczema

B Scabies

C Psoriasis

D Pityriasis rosea

E Malignant melanoma

F Pemphigoid

G Discoid lupus

H SLE (systemic lupus erythematosis)

I Shingles

J Tynea

K Ringworm

L BCC (basal cell carcinoma)

M Chicken pox

A

Pityriasis rosea

It is benign but may inflict substantial discomfort in certain cases. Classically, it begins with a single “herald patch” lesion, followed in 1 or 2 weeks by a generalized body rash lasting up to 12 weeks, however usually around 6 - 8.

Can be associated with a URTI, fever, headache, nausea, fatigue.

454
Q

72 year old man with diabetes. He has been on insulin for 6 years. His daughter had noticed that he was increasingly vague. He was found unconscious. BP 160/90. Plasma urea, creatinine & electrolytes were normal.

A Peripheral neuropathy

B Nephropathy

C Retinopathy

D Cataract

E Autonomic neuropathy

F Onset type I diabetes

G Hyperosmolar coma

H Ischaemic heart disease

I Peripheral arterial disease

J Ketoacidosis

K Hypoglycaemia

L Gastroparesis

M Onset type 2 diabetes

A

hypo

455
Q

A 12 year old girl presents with pallor, dyspnoea, & pulse rate of 190. She is noted to have cardiomegaly & hepatomegaly.

A Kawasaki disease

B Hereditary angioedema

C Congenital nephritic syndrome

D Myocarditis

E Pericarditis

F Primary pulmonary hypertension

G Juvenile rheumatoid arthritis

H Acute rheumatic fever

I Congestive heart failure

J Toxic synovitis

K Aortic stenosis

A

CCF

456
Q

A 59 year old man presents to the rheumatology outpatient department with haemoptysis, weight loss & painful swelling of the wrists. He has smoked since the age of 14.

A Carcinoma of bronchus

B Bronchiectasis

C Pleural empyema

D Fibrosing alveolitis

E Cirrhosis

F Inflammatory bowel disease

G Congenital heart disease

H Mesothelioma

I Hypertrophic pulmonary osteoarthropathy

J Congenital

K COAD

L RA

M Carcinoma of caecum

A

Hypertrophic pulmonary osteoarthropathy

This disease presents with clubbing and periostitis of the small joints of the hand (DIP). Also painful, swollen joints of the long bones. Associated with lung diseases such as non-small cell lung Ca.

457
Q

A 60 year old lady with recent onset unilateral headache made worse by combing her hair. Her thyroid function was in the hypothyroid range & her ESR was 60mm/hr.

A Temporal arteritis

B Meningitis

C Encephalitis

D Subarachnoid haemorrhage

E Sinusitis

F Migraine

G Tension headache

H Raised intracranial pressure

I Severe hypertension

J Analgesic rebound headache

K Trigeminal neuralgia

A

Temporal arteritis

Raised ESR!

458
Q

A 15 month old toddler developed a maculo-papular rash 10 days after receiving the MMR vaccine, followed by a swelling at the angle of the left jaw. His mother telephoned the surgery in a state of panic one morning because he is having convulsions & a high temperature.

A Temporal arteritis

B Meningitis

C Encephalitis

D Subarachnoid haemorrhage

E Sinusitis

F Migraine

G Tension headache

H Raised intracranial pressure

I Severe hypertension

J Analgesic rebound headache

K Trigeminal neuralgia

A

Encephalitis

This child has developed Mumps from the MMR vaccine. As a result he has parotitis, a rash and encephalitis. He may develop orchitis.

459
Q

A 30 year old nurse presented with frontal headache after an acute viral illness. The pain was constant, affecting the right side of her head only. On examination there was marked tenderness on the right side of her face, over the maxillary area.

A Temporal arteritis

B Meningitis

C Encephalitis

D Subarachnoid haemorrhage

E Sinusitis

F Migraine

G Tension headache

H Raised intracranial pressure

I Severe hypertension

J Analgesic rebound headache

K Trigeminal neuralgia

A

Sinusitis

460
Q

80 year old presents with symptoms of dysphagia. He has been a lifelong smoker. On examination there is a reducible mass over lateral aspect of the neck.

A Hodgkin’s lymphoma

B Lymphadenopathy 20 to infection

C Lymphadenopathy 20 to malignancy

D Carotid body tumour

E Thyroid cancer

F Thyroiditis

G TB lymphadenitis

H Thyroid goitre

I Salivary gland tumour

J Parotitis

K Mumps

L Thyroglossal cyst

M Pharyngeal pouch

N Glandular fever

A

Pharyngeal pouch.

A pharyngeal pouch (aka Zenker’s diverticulum) is a diverticulum of the mucosa of the pharynx.

This can present in the more elderly with dysphagia, a lump in the neck, regurgitation, cough and halitosis.

Can be identified with a Ba swallow.

461
Q

A 70 year old man presents with poor stream & nocturia. On examination, he has a lemon tinge to his skin, ascites, a palpable bladder & enlarged prostate gland. His blood pressure is 170/95.

A Carcinoma of the bladder

B Carcinoma of the kidney

C Carcinoma of the prostate

D Acute pyelonephritis

E Testicular torsion

F Acute epididymo-orchitis

G Testicular tumour

H Inflamed hydatid de Morgani

I Acute tubular necrosis

J Chronic renal failure

K Ureteric colic

L Hydrocoele

A

Chronic renal failure.

This patient is uraemic.

462
Q

A 51 year old manic depressive man, who has taken an unknown number of extra lithium tablets & has a plasma lithium level of 4.5mmol/l.

A Alakaline diuresis

B Intravenous acetylcysteine

C Haemodialysis

D Intravenous naloxone

E Oral desferrioxamine

F Dicolbolt edetate

G Hyperbaric oxygen

H Intravenous pralidoxime

I Ethanol

J Sodium calcium edetate

A

Haemodialysis

463
Q

Erythromycin

What is the frequent side effect for each of the following drugs or class of drug taken from the list above?

A Cold toes & fingers

B Constipation & dry mouth

C Deafness

D Dry cough

E Goitre

F Indigestion & GI bleed

G Nausea & vomiting

H Swelling of feet & ankles

I Peripheral neuropathy

J Tremor

A

N+V

464
Q

A 49 year old man of northern European origin presents to an endocrinologist with symptoms & signs of diabetes mellitus, & also complains of long-standing fatigue & arthralgia. On extensive further investigation he is noted to have cardiomyopathy, testicular atrophy & hepatomegaly. The patient also mentions that his skin seems to have darkened somewhat over the past decade, despite no increase in sun exposure.

Which drug would combat the poisoning or overload?

A Flumazenil

B Naloxone hydrochloride

C N-acetylcysteine

D Protamine sulphate

E Dicobalt edetate

F Desferrioxamine

G Vitamin K

H Penicillamine

I Ethanol

J Pralidoxime mesilate

K Methionine

L Dimercaprol

A

DFO

This patient has haemochromatosis.

465
Q

A 35 year old woman has a 10 year history of low retrosternal dysphagia & painless regurgitation of food in the mouth.

A Achalasia of cardia

B Carcinoma of oesophagus

C Cerebrovascular accident

D Thyroid goitre

E Myasthenia gravis

F Inflammatory oesophageal stricture

G Plummer-vincent’s syndrome

H Carcinoma of bronchus

I Hiatus hernia

J Pharyngeal pouch

A

Achalasia

466
Q

A 50 year old obese lady with weight loss, tiredness, pruritus vulvae & recurrent boils.

A Diabetes mellitus

B Myxoedema

C Thyrotoxicosis

D Primary hyperparathyroidism

E Cushing’s Syndrome

F Addison’s disease

G Phaeochromocytoma

H Hypopituitarism

I Hyperprolactinoma

J Acromegaly

K Diabetes insipidus

A

DM

467
Q

A 35 year old lady with weight loss, poor appetite, abdominal pains & irregular periods. On examination she had white patches on her face & dark colouring in the palms of her hands.

A Diabetes mellitus

B Myxoedema

C Thyrotoxicosis

D Primary hyperparathyroidism

E Cushing’s Syndrome

F Addison’s disease

G Phaeochromocytoma

H Hypopituitarism

I Hyperprolactinoma

J Acromegaly

K Diabetes insipidus

A

Addison’s.

This patient not only has Addison’s but it has caused vitiligo hence the white patches on the face.

468
Q

A 71 year old lady presented with depression & personality disorder which was first noticed during long term psychiatric care. On examination there was marked alopecia & loss of facial contours.

Treatment

A Combined FSH/LH

B Surgery

C GnRH subcutaneous infusion pumps

D Thyroxine

E Oral sugar

F Gliclazide

G Alfacalcidol

H Hydrocortisone

I Spironolactone

J Propanolol

K Octreotide

L Carbimazole

M Ethyloestradiol

A

Thyroxine

469
Q

A lady came to her GP due to problems trying to conceive. She says her periods have been unpredictable for the past 2 years. She’s been suffering from double vision recently. Other than that she’s been fine. On examination her eyes appeared to bulge out & had swelling periorbitally.

Treatment

A Combined FSH/LH

B Surgery

C GnRH subcutaneous infusion pumps

D Thyroxine

E Oral sugar

F Gliclazide

G Alfacalcidol

H Hydrocortisone

I Spironolactone

J Propanolol

K Octreotide

L Carbimazole

M Ethyloestradiol

A

Carbimazole.

If she is trying to conceive should she be on PTU?

470
Q

40 year old woman • Hb: 110 g/L
• MCV: 60

The most likely cause is:

A. Iron deficiency

B. Beta thalassaemia heterozygosity

A

Beta thalassemia

A very very low MCV suggests Thalassemia rather than IDA.

471
Q

50 year old man. Hypercalcaemia. Low PTH. Backache.Normal ALP

The most likely cause is:

A. Malignancy

B. Multiple myeloma

A

Myeloma.

Myeloma presents with a normal ALP because osteoblasts are suppressed.

472
Q

A 67 year old woman presents to A+E with lethargy, fever and left sided temporal

headaches. There is no obvious pulsation of the temporal artery.

A. Primary percutaneous coronary

intervention

B. High dose oral steroids

C. Coronary artery bypass graft

D. IV phenytoin

E. Oral leukotriene receptor antagonist

F. IV magnesium sulphate

G. 1mg IM glucagon

H. 50mL of 50% IV glucose

I. Continue oxygen via facemask and

nebulised short acting β-agonists and

steroids

J. Oral glucose

K. IV lorazepam

L. Oral diazepam

M. IV benzylpenicillin

N. Streptokinase

A

High dose oral steroids.

This is temporal arteritis for which the treatment is steroids.

473
Q

A 32 year old man presents with SOB. He feels that it is similar to the asthma he

had as a child. He has also developed hay fever. On examination he has a

palpabale purple rash on his legs.

A. Behҫet’s disease

B. Chrug-Strauss syndrome

C. Giant cell arteritis

D. Kawasaki’s disease

E. Microscopic polyangitis

F. Polyarteritis nodosa

G. Polymyalgia rheumatica

H. Takayasu’s arteritis

I. Wegener’s disease

A

B. Churg-Strauss

• Can be triphasic

– Rhinitis and allergies

– Eosinophilia

– Systemic Vasculitis

  • pANCA
  • Steroids
474
Q

A 29 year old man complains of recurrent painful ulceration in his mouth and on

his genitals. The ulceration is clearly seen on examination. Swab cultures are

negative for herpes simplex virus.

A. Behҫet’s disease

B. Chrug-Strauss syndrome

C. Giant cell arteritis

D. Kawasaki’s disease

E. Microscopic polyangitis

F. Polyarteritis nodosa

G. Polymyalgia rheumatica

H. Takayasu’s arteritis

I. Wegener’s disease

A

Behҫet’s disease

  • Recurrent oral and genital ulceration
  • Also iritis, skin lesions, thrombophlebitis
  • Pathergy reaction
  • Turkey and eastern Mediterranean
  • HLA-B5, M
475
Q

A 5 year old boy is brought to the GP as he has had a fever for the past 5 days.

On examination you find cervical lymphadenopathy, odema and desquamation

of his legs, oral mucosal erythema and a rash on his back.

A. Behҫet’s disease

B. Chrug-Strauss syndrome

C. Giant cell arteritis

D. Kawasaki’s disease

E. Microscopic polyangitis

F. Polyarteritis nodosa

G. Polymyalgia rheumatica

H. Takayasu’s arteritis

I. Wegener’s disease

A

Kawaski’s disease

  • Acute febrile systemic vasculitis
  • Children especially japanese boys
  • Must have fever for 5 days and 4 of;

cervical lymphadenopathy, oral mucosal erythema, conjunctivis without exudates, rash, extremity changes

  • May have thrombocytosis
  • Anti-inflammatories and immunoglobulins
  • Coronary aneurysms
476
Q

A 30 year old male presenting with lethargy. Bilirubin 50µmol/l (unconjugated)

ALT 20, AST 80, ALP 40, LDH elevated

a. Gilbert’s Disease
b. Hepatitis A
c. Alcoholic liver disease
d. Hepatitis C
e. Biliary tract obstruction
f. Haemolytic anaemia
g. Hepatocellular carcinoma
h. Crigler Najjar Syndrome
i. Short term alcohol abuse
j. Hepatitis B
k. . Paget’s Disease
l. Carcinoma of the pancreas

A

Haemolytic anaemia

  • An unconjugated hyperbilirubinaemia
  • Elevated AST but normal ALT and ALP, and elevated LDH
  • AST is not purely a hepatic enzyme; it is also found in e.g. heart,

erythrocytes

  • LDH is also found in e.g. heart, muscle, erythrocytes
  • Hence a patient with haemolytic anaemia would see rises in

these enzymes. The clinical picture of lethargy is also consistent

477
Q

A 25 year old female admitted to hospital with nausea/vomiting and severe hypotension, most marked when she stands up.Blood results show glucose 3mmol/l, Na 119mmol/l, K 7.2mmol/l

A: Carbimazole

B: Surgery

C: Octreotide

D: IV hydrocortisone

E: Levothyroxine

F: IV prednisolone

G: Gliclazide

H: Desmopressin

I: Propylthiouracil

J: Liothyronine

K: Metyrapone

L: Bendroflumethiazide

A

IV hydrocortisone.

This patient had Addison’s, the treatment for which is IV hydrocortisone.

478
Q

A 25 year old alcoholic presents with severe vomiting and epigastric pain. ABG

demonstrates pH 7.51, pO2 12kPa, PCO2 5.2 kPa, bicarbonate 36 mmol/l

Arterial blood gases

a. Uncompensated metabolic acidosis
b. Uncompensated respiratory acidosis
c. Uncompensated respiratory alkalosis
d. Uncompensated metabolic alkalosis
e. Partially compensated metabolic acidosis
f. Type I respiratory failure
g. Partially compensated respiratory acidosis
h. Partially compensated metabolic alkalosis
i. Fully compensated respiratory acidosis
j. Type II respiratory failure
k. Fully compensated metabolic acidosis

A

Uncompensated metabolic alkalosis

  • History is suggestive of pancreatitis
  • Severe vomiting has caused loss of stomach acid, so alkaline pH
  • High bicarbonate confirms this is a likely metabolic alkalosis
  • There doesn’t appear to be any effort to compensate as the PCO2 is within the normal

range

479
Q

A 25 year old man presented yesterday in severe pain with a distended abdomen. An AXR done upon admission shows grossly distended large bowel. He is in theatre now as conserva>ve management over the last 24 hours has failed.

A. Tracheostomy

B. Loop colostomy

C. End ileostomy

D. Gastrostomy

E. De-func>oning colostomy

F. Nephrostomy

G. Loop ileostomy

H. End colostomy

I. Cystostomy

J. Urostomy

K. De-func>oning ileostomy

L. None of the above

A

End ileostomy.

End ileostomies are performed as part of an operation whereby the large bowel is removed and the end of the ileum is brought out as a stoma. This can be a temporary or a permanent measure. This is effectively protecting the distal large bowel that is diseased.

480
Q

Which of these is a relative contraindication to thrombolysis?

A. History of haemorrhagic stroke

B. Recent head injury

C. Ischaemic stroke in the past 3 months

D. Clotting disturbance

E. Possible dissecting aortic aneurysm

A

Recent head injury

**Absolute Contraindications **

  • Active bleeding
  • History of haemorrhagic stroke
  • Ischaemic stroke in previous 3 months
  • Clotting disturbance
  • Possible dissecting aortic aneurysm

**Relative contraindications: **

  • Recent head injury
  • PMH of peptic ulcer disease
  • Cerebral neoplasm
  • Liver disease
  • Systolic BP >200mmHg
481
Q

A stable COPD sufferer continues to be breathless despite treatment with short acting

bronchodilators. An FEV1 measurement is <50% predicted.

What is the next appropriate treatment step?

a. Add inhaled beclometasone
b. Add montelukast
c. Add ipratropium (short acting muscarinic antagonist)
d. Add oral prednisolone
e. Add salmeterol
f. Add tiotropium (long acting muscarinic antagonist)

A

Add tiotropium

  • Look at NICE guidelines for more detail, but in short:
  • First line inhaled therapy is with short acting bronchodilators
  • If this does not control symptoms, then base step on FEV1
  • If FEV1 >50% predicted, add either long acting B2 agonist (LABA) or long acting

muscarinic antagonist (LAMA)

-If <50% predicted, either LABA + inhaled steroid, or LAMA

482
Q

A 55 year old male presents with a few months history of lethargy, weight loss and LUQ

pain. Examination reveals massive splenomegaly. A peripheral blood film is
leukoerythroblastic. What is the most likely diagnosis?
a. Chronic myeloid leukaemia
b. Myelofibrosis
c. Waldenstroms macroglobulinaemia
d. Polycythaemia rubra vera
e. Chronic lymphocytic leukaemia
f. Myeloma

A

Myelofibrosis

-Massive splenomegaly in haematological context can be seen in CML, myelofibrosis,

CLL. Also in malaria, leishmaniasis. Splenomegaly : 2Hs & 2Is

-Give away is a leukoerythroblastic blood film (presence of primitive WBCs and primitive

RBCs – nucleated erythrocytes). Seen in myelofibrosis

-In myelofibrosis question may also mention teardrop poikilocytosis

483
Q

A 42 year old male presents with a 3 day history of abdominal pain, absolute constipation

and distension. After basic management has been instigated what is the most

appropriate next step?

a. MRI
b. Abdominal ultrasound
c. Diagnostic laparoscopy
d. Abdominal x-ray
e. CT abdomen
f. Colonoscopy

A

AXR

  • History indicates most likely intestinal obstruction
  • As vomiting is not a prominent feature but constipation is, suggests a large bowel

obstruction

-A plain abdominal radiograph is an appropriate next modality to confirm that there is

indeed intestinal obstruction (note difference between small and large bowel

obstruction appearances)

-A contrast CT scan could be done after this if necessary to further visualise obstruction

484
Q

A 57 year old lorry driver attends a regular appointment with his optician. On fundoscopy, dots & blots are noted, in addition to hard exudates around the macula

A Diabetic background retinopathy

B Hypercholesterolaemia

C Macular degeneration

D Preproliferative retinopathy

E Horner’s syndrome

F Conjuntivitis

G Subdural haemorrhage

H III nerve lesion

I Migraine

J Acute glaucoma

A

Diabetic background retinopathy.

Background (mild) non-proliferative DR: at least one microaneurysm.

Moderate non-proliferative DR: microaneurysms or intraretinal haemorrhages ± cotton wool spots, venous beading, intraretinal microvascular abnormalities (IRMAs).

Severe to very severe non-proliferative DR (sometimes referred to as pre-proliferative disease), as above: a minimum number of these features are required in a minimum number of retinal quadrants to define severe or very severe disease.

Non-high-risk proliferative DR: new vessels on the disc (NVD) - or within one disc diameter of it or new vessels elsewhere (NVE).

High-risk proliferative DR: large NVD or NVE (defined by comparing to the optic disc surface area) or presence of pre-retinal haemorrhage. In advanced disease, there may also be an accompanying retinal detachment.

485
Q

Weber’s disease

A Ataxia

B Congestive cardiac failure

C Gastroenteritis

D Hereditary haemorrhagic telangiectasia

E Impotence

F Jaundice

G Leprosy

H Pericarditis

I Stroke

J Tuberculosis

K Vasculitis

A

Stroke

Weber’s syndrome (superior alternating hemiplegia) is a form of strokecharacterized by the presence of an oculomotor nerve palsy and contralateralhemiparesis or hemiplegia.

486
Q

An 8 year old boy presents with constipation. His abdomen is also slightly distended. He is normally fit and his diet is balanced and healthy.

A. Aganglionosis

B. DehydraDon

C. Colorectal carcinoma

D. Hypokalaemia

E. Environmental

F. Furosemide use

G. Hypothyroidism

H. Fibroid

I. Diabetes

J. Anal fissure

K. Opiate use

L. Hypercalcaemia

A

Aganglionosis

Aka Hirchsprung’s disease.

Absence of ganglion cells in the GI tract

  • Therefore impaired/absent nervous control

Usually diagnosed as infant

487
Q

65 year old man is referred by his GP with altered bowel habit and rectal bleeding over the past 6 weeks. The blood is bright red and

is not associated with any pain. He reports tenesmus and weight loss.

A. OGD

B. Colonoscopy

C. Barium Swallow

D. Flexible Sigmoidoscopy

E. Rigid Sigmoidoscopy

F. CT KUB

G. CT CAP

H. USS

I. Abdo X-Ray

J. Proctoscopy

K. Barium enema

L. None of the above

A

Flexibly sigmoidoscopy

Colorectal Ca

  • Predisposing Factors: IBD, polyps, FAP, smoking, low fibre diet
  • Genetics: one 1st deg rela5ve 1:17, two first deg relatives 1:10
  • Tx: Must stage first – TNM
           Chemoradiation/Surgery    

• Flexi Sig – visualisation of rectum and sigmoid, biopsies

488
Q

A 40 year old banker presents with left upper quadrant pain radiating to

the back. He is hypotensive and tachycardic. What is the single best

marker of prognosis?

A Serum amylase

B ALT/AST

C Venous oxygen

D HDL

E Serum urea

A

Serum urea

489
Q

A 35 year old woman presents with a lump on her neck. It has uneven borders, is smooth, slightly fluctuant and is not fixed to the skin

or deeper structures.

A. MRI

B. FBC

C. Fine needle aspiraUon

D. USS

E. Sialogram

G. Angiography

H. X-ray

I. Barium swallow

J. Excision biopsy

K. Reassure patient

A

Reassure patient.

Lipoma

  • Benign fatty lump
  • Occurs wherever fat can expand (not scalp/
    palms)
  • Symptomatic only with pressure
  • Malignant change is RARE
490
Q

A 66 year old retired Marines officer presents with a large scar from a burn sustained in a war with a painless 3 cm area of irregular ulceraOon within the region of scarring. It is surrounded by a raised edge and has bloodstained discharge from the base. There is no regional lymphadenopathy.

A Anthrax

B Chancrous ulcer

C Gummatous ulcer

D Arterial ulcer

E Neuropathic ulcer

F Squamous cell carcinoma

G Tuberculous ulcer

I Pyoderma gangrenosum

H Venous ulcer

J Marjolin’s ulcer

A

Marjolin’s ulcer

An aggressive ulcerating SCC that occurs in an area of previously traumatised, chronically inflamed or scarred skin.

491
Q

A 54yr old man with Stage 3 CRF comes to renal outpatient clinic. You see that his calcium is low, and his alk phos is high.

What test would you do to confirm the suspected pathology?

a) abdo ultrasound
b) DEXA scan
c) lumbar spine x ray
d) FBC, U&E, LFT, urine dipstick

A

DEXA scan

Renal Osteodystrophy

  • High phosphate due to inadequate excretion
  • Low calcium due to low calcitriol production and poor renal reabsorption.
  • Increased PTH
  • Increased bone resorption
  • Osteoporosis-like picture on DEXA
492
Q

Chloramphenicol side effects

A
  1. Anaemia
  2. Aplastic anaemia
  3. Grey baby syndrome - accumulation of toxic chloramphenicol metabolites due to baby’s reduced metabolism.
493
Q

Doxycycline SFx

A
  1. Photosensitivity reaction
  2. GI upset
  3. Tooth discolouration
494
Q

Erythromycin SFx

A
  1. GI discomfort - N+V
  2. Skin rashes
495
Q

Imipenem SFx

A
  1. GI upset
  2. skin rash
  3. Seizures
496
Q

Gentamicin SFx

A

Aminoglycosides = A MEAN GUY

Punch in the stomach - Renal toxicity. Reversible

Knocks your head off - CN VIII toxicity = hearing loss and vertigo. Irreversible.

497
Q

A 12 year old boy is a member of a family with a history of renal disease, with males more severely affected than females. He is found to have auditory nerve deafness, corneal dystrophy, and ocular lens dislocation. A urinalysis shous microscopic haematuria.

A. Alport’s syndrome

B. Autosomal dominant PKD

C. Haemolytic uraemic syndrome

D. IgA nephropathy

E. Autosomal recessive PKD

F. Tuberous sclerosus

G. Wegener’s granulomatosis

I. Lupus nephritis

J. Hepatitis B

K. Post-streptococcal GN

L. Henloch-Scholein purpura

M. Minimal change GN

N. Goodpasture’s syndrome

O. Amyloidosis

A

Alport’s syndrome

X linked disease. Mutation in type IV collagen. Major structural component of the basement membranes in the kidney, ear and eye.

Triad: Haematuria + several ocular abnormalities + SNHL

498
Q

Interpet the following results:

Bil: 25umol/L (<17); Alb: 40 g/L (35-51); ALT: 5560 IU/L (<40); AST 3410 U/L (<40); ALP 280 U/L (35-51); GGT: 470 U/L (11-42); INR 2.7 (0.8-1.2)

A. Paracetamol overdose

B. Alcoholic liver disease

C. Cholestasis

D. Hepatitis C

A

Paracetamol

ALT and AST have very high indicating high liver damage. Acute marker of liver function is affected (INR reduced) but chronic measure is normal (albumin). Hence this is acute.

499
Q
A